Spencer on an alternate view of CO2 increases

This interesting essay by Dr. Spencer is reposted from his blog, link here:

Global Warming Causing Carbon Dioxide Increases: A Simple Model

May 11th, 2009 by Roy W. Spencer, Ph. D.

Global warming theory assumes that the increasing carbon dioxide concentration in the atmosphere comes entirely from anthropogenic sources, and it is that CO2 increase which is causing global warming.

But it is indisputable that the amount of extra CO2 showing up at the monitoring station at Mauna Loa, Hawaii each year (first graph below) is strongly affected by sea surface temperature (SST) variations (second graph below), which are in turn mostly a function of El Nino and La Nina conditions (third graph below):

simple-co2-model-fig01

Click for larger images

simple-co2-model-fig02

simple-co2-model-fig03

Click for larger image

During a warm El Nino year, more CO2 is released by the ocean into the atmosphere (and less is taken up by the ocean from the atmosphere), while during cool La Nina years just the opposite happens. (A graph similar to the first graph also appeared in the IPCC report, so this is not new). Just how much of the Mauna Loa Variations in the first graph are due to the “Coke-fizz” effect is not clear because there is now strong evidence that biological activity also plays a major (possibly dominant) role (Behrenfeld et al., 2006).

The direction of causation is obvious since the CO2 variations lag the sea surface temperature variations by an average of six months, as shown in the following graph:

simple-co2-model-fig04

So, I keep coming back to the question: If warming of the oceans causes an increase in atmospheric CO2 on a year-to-year basis, is it possible that long-term warming of the oceans (say, due to a natural change in cloud cover) might be causing some portion of the long-term increase in atmospheric CO2?

I decided to run a simple model in which the change in atmospheric CO2 with time is a function of sea surface temperature anomaly. The model equation looks like this:

delta[CO2]/delta[t] = a*SST + b*Anthro

Which simply says that the change in atmospheric CO2 with time is proportional to some combination of the SST anomaly and the anthropogenic (manmade) CO2 source. I then ran the model in an Excel spreadsheet and adjusted an “a” and “b” coefficients until the model response looked like the observed record of yearly CO2 accumulation rate at Mauna Loa.

It didn’t take long to find a model that did a pretty good job (a = 4.6 ppm/yr per deg. C; b=0.1), as the following graph shows:

simple-co2-model-fig05

Click for larger image

The best fit (shown) assumed only 10% of the atmospheric CO2 increase is due to human emissions (b=0.1), while the other 90% is simple due to changes in sea surface temperature. The peak correlation between the modeled and observed CO2 fluctuation is now at zero month time lag, supporting the model’s realism. The model explained 50% of the variance of the Mauna Loa observations.

The best model fit assumes that the temperature anomaly at which the ocean switches between a sink and a source of CO2 for the atmosphere is -0.2 deg. C, indicated by the bold line in the SST graph, seen in the second graph in this article. In the context of longer-term changes, it would mean that the ocean became a net source of more atmospheric CO2 around 1930.

A graph of the resulting model versus observed CO2 concentration as a function of time is shown next:

simple-co2-model-fig06

If I increase the anthropogenic portion to 20%, the following graph shows somewhat less agreement:

simple-co2-model-fig07Click for larger images

There will, of course, be vehement objections to this admittedly simple model. One will be that “we know the atmospheric CO2 increase is manmade because the C13 carbon isotope concentration in the atmosphere is decreasing, which is consistent with a fossil fuel source.” But has been discussed elsewhere, a change in ocean biological activity (or vegetation on land) has a similar signature…so the C13 change is not a unique signature of fossil fuel source.

My primary purpose in presenting all of this is simply to stimulate debate. Are we really sure that ALL of the atmospheric increase in CO2 is from humanity’s emissions? After all, the natural sources and sinks of CO2 are about 20 times the anthropogenic source, so all it would take is a small imbalance in the natural flows to rival the anthropogenic source. And it is clear that there are natural imbalances of that magnitude on a year-to-year basis, as shown in the first graph.

What could be causing long-term warming of the oceans? My first choice for a mechanism would be a slight decrease in oceanic cloud cover. There is no way to rule this out observationally because our measurements of global cloud cover over the last 50 to 100 years are nowhere near good enough.

And just how strenuous and vehement the resulting objections are to what I have presented above will be a good indication of how politicized the science of global warming has become.

REFERENCES

Michael J. Behrenfeld et al., “Climate-Driven Trends in Contemporary Ocean Productivity,” Nature 444 (2006): 752-755.

5 1 vote
Article Rating
282 Comments
Oldest
Newest Most Voted
Inline Feedbacks
View all comments
Indiana Bones
May 12, 2009 11:34 am

Slightly OT but germane to regulation of CO2 is an excerpt from an inter-agency memo authored by Office Management Budget:
“The amount of acknowledged lack of understanding about the basic facts surrounding [greenhouse gases] seem to stretch the precautionary principle to providing regulation in the face of unprecedented uncertainty,”
This reported today in the Wall Street Journal as EPA Administrator Lisa Jackson appears to about face on “Cap N’ Trade.” But without $646 billion in carbon trade revenue – how will the Administration pay for programs?
http://online.wsj.com/article/SB124214922088511421.html

Joel
May 12, 2009 11:56 am

Is it the physical properties of oceans that alter C02 absorption or is it biological? I could be backwards, but I’m having trouble understanding how a warmer ocean (regardless of why it’s warmer now) absorbs less CO2 than a cooler ocean, especially if the absorption is through marine life. Wouldn’t warmer temps increase plant life and therefore decrease C02? Either way it will be a glorious day if it is proven that this relationship is the main mechanism for CO2 fluctuations!!

Douglas DC
May 12, 2009 12:02 pm

Ahem, I shall get the criticisms over: Heretic! Unbeliever, Infidel!! Ok, that said this may be a very good way of seeing what Co2 actually is in our atmosphere.I’ve held that the Co2 content is from largely Oceanic heating.-Now that things are cooling off for whatever the reason,could we be seeing some stabilization or even decrease?-Due mainly to the Oceanic absorption of Co2…

John in NZ
May 12, 2009 12:02 pm

Fantastic. I love it.
It will be interesting to see comment from others.

Adam from Kansas
May 12, 2009 12:03 pm

Well at least the world’s factories are contibuting their (small) amount of an increase in a gas that is plant food and will cause plant-life to flourish.
I don’t know if we crossed a magic threshold that could be with a certain CO2 level + lots of rain or not, last year the mullberry and hackberry trees in our backyard grew some jumbo-sized leaves that were quite a bit bigger than usual, our neighbor’s young walnut tree had leaves that looked like they were hit with a jumbo ray, and there’s early signs some trees in our yard could have huge leaves this year as well.
This is why I didn’t like it when I saw a picture of these things that are designed to take CO2 out of the air, the CO2 is helping out with the rain to create good-sized plant growth that’s more than normal behind our house and around a few other spots in our neighborhood.

Ron de Haan
May 12, 2009 12:05 pm

How much clearer do you want to have your science served!
Great job, great conclusions.

Jim Papsdorf
May 12, 2009 12:06 pm

Fantastic !!!
It is great to see how your 90-10 Model serves to hang the AGW Alarmists on their own petards !!!!

jmrSudbury
May 12, 2009 12:11 pm

I wonder about the carbon cycle. The http://www.uwsp.edu/geO/faculty/ritter/geog101/textbook/earth_system/carbon_cycle_NASA.jpg picture shows that the amount to and from the oceans is only 50% more than the amount to and from land vegetation. Both are much higher than the fossil fuel and cement industries. Is this not a large component of the puzzle that this simple model excludes?
John M Reynolds

John Galt
May 12, 2009 12:19 pm

The ice core data shows long-term rises in CO2 following temperature increases and the short-term data shows CO2 lags surface temperate rises by 6 months. Now how can CO2 be causing the increases?

Stephen Wilde
May 12, 2009 12:28 pm

Here is something relevant which I prepared some time ago:
1) Take a container of, say, 10 litres, full of natural CO2 molecules (N) only.
2) Have an intake on one side and an outlet on the other.
3) Insert 9 molecules of N and one molecule of human CO2 (H) into the intake.
4) Allow time for mixing and remove 10 randomly selected molecules from the outlet.
5) Due to the size of the initial volume the chance of removing H is infinitesimal so the contents will then have an H molecule with all the rest being N molecules.
6) Repeat many many times and eventually the proportion of H molecules in the container will rise to 10% which is commensurate with the proportion of H amongst the molecules being put in.
What will have happened up to this point is that outgoing N molecules will have been replaced by incoming H molecules until 10% have been replaced so it’s now 90% N and 10% H but the total volume has remained the same.
Then this goes to the heart of the issue:
7) Add 1 litre of N to the container which will reduce the proportion of H then continue the process as before until the 10% proportion of H is restored.
8) Now we will have a larger total volume but still 90% N and 10% H
9) Note that the increase in volume came entirely from new N. The H molecules only got there by replacing molecules of N.
10) It would be tempting to say that the 10% increase in volume was caused by the introduction of the 10% H but in fact it was not.
Thus it is unsafe to suggest that in the real world the increase in total atmospheric CO2 was caused by anthropogenic input, even if the volume of H appears to be similar to the amount of increase in the total volume.

Mike from Canmore
May 12, 2009 12:30 pm

Anthony:
Sometime in the last year, you posted an article, (by Joe D’Aleo I believe – could be wrong as I’m going by memory). I tried to find it. No luck. Do you still have it? It would be nice to read that along side the above.
Thanks in advance.

Robert Wood
May 12, 2009 12:35 pm

I like this article, it explores something I’ve often had doubts about.
Also “the natural sources and sinks of CO2 are about 20 times the anthropogenic source” is a fact I use with agnostics

Steve Keohane
May 12, 2009 12:37 pm

The CO2 balloon has major leaks. Temperature doesn’t seem to respond to the monotonic increasing of atmospheric concentrations of this trace gas. Understanding whatever it is that consistantly is overwhelming CO2 should go a long way towards understanding climate. The anthro contribution to the increasing levels of CO2 is miniscule, 3-4%. It seems we are wasting a lot of time and resources on a non-problem that certainly isn’t driving our climate.

Cassanders
May 12, 2009 12:44 pm

A very interesting approach.
A question from a simple mind.
Are some of the CO2- extrapolations employed by IPCC assuming a 1% annual CO2 increase?
With the current rate , http://www.esrl.noaa.gov/gmd/ccgg/trends/
the annual rate of increase is in the 0.5-0.75% range, and apparently decreasing (as the total CO2 concentration is increasing).
Cassanders
In Cod we trust

Erik
May 12, 2009 12:45 pm

I don’t think you’re actually saying 10% from an Anthropogenic source. b=.1 would represent 10% if a=.9, but since you have a=4.6 you’re actually saying that the Anthopogenic contribution is closer to 2 percent (2.13%). Increasing b to .2 means it’s a 4.17% contribution. a=4.6 would be a normalized to 97.87%. Your weighting variables are percentages only when your equation is normalized.

Ray
May 12, 2009 12:46 pm

Anthropogenic CO2 is still rising while both temperature and global CO2 are decreasing. Why are we blamed for this again? Oh, yeah, that’s because Fat Albert said so…

May 12, 2009 12:46 pm

Al prophesying
CO2 choking
cows farting
planets circling
Sun moving…
Oh Gosh!, barycenter!
sunspots disappearing
jet stream downing
cosmic rays falling
clouds growing
la nina dancing
Oh Gosh!, my head dizzying

Ed Scott
May 12, 2009 12:49 pm

Obama Administration Memo Warns of Harm to Economy if Greenhouse Gases Regulated through Clean Air Act
http://blogs.abcnews.com/politicalpunch/2009/05/obama-adminis-2.html
If the EPA goes forward with a finding of endangerment for all six greenhouse gases, the document warns, “it could be establishing a relaxed and expansive new standard for endangerment. Subsequently, EPA would be petitioned to find endangerment and regulate many other ‘pollutants’ for the sake of the precautionary principle (e.g., electromagnetic fields, perchlorates, endocrine disruptors, and noise).”

Kazinski
May 12, 2009 12:51 pm

It is an interesting theory, and there certainly are mechanisms that can explain the cause and effect relationship. But the real test will be the models ability to predict CO2 rate of change going forward. The Achilles heal of the climate models is of course they seem much better at predicting the past than they are at predicting the future, since this model has a six month lag, it should only take a few years to vet the model and make sure it has staying power.

Michael D Smith
May 12, 2009 12:53 pm

I wasn’t aware that anyone thought 100% of increases must be anthropogenic. That seems pretty illogical given the biosphere and the cyclical nature of… nature. There I go thinking logically again…
Roy, I’ve done a fair amount with this already, and I have a method to backsolve any variable so I can zero in on the parameters to match the curve (or change the model) to match it better than what you’re showing. It’s a lot more powerful than excel’s backsolver. If you want to email me a copy of your sheet I’ll show you what I mean and will email back the results…
I use it to match oscillation frequency / amplitude until I get best match with actual data (least squares), then project forward oscillations to get temperature estimates… Works with CO2. dCO2/DT is pretty drastic and very fast, leading me to think that the amount exchanged is huge (no matter what the temperature is at the moment) and the time constant is very short (i.e, the notion that residence time of CO2 is more than a few years is pure fantasy). 6 month lag peak to peak depends somewhat on the temp oscillation frequency (since if there was not an oscillation neither curve would have peaks)… Let me know if you’re interested… It should be possible to get a time constant out of the data (that too could be modeled to get the best fit TC)

Gene Nemetz
May 12, 2009 1:01 pm

Mr. Roy W. Spencer,
You have an easy to understand style of writing. Thank you for that,
A fellow Yooper

Richard deSousa
May 12, 2009 1:03 pm

It makes sense to me but watch the AGW promoters attack Dr. Spencer with ad homs. They are getting desperate as more and more scientists and lay people are abandoning the side of the AGWers.

Flanagan
May 12, 2009 1:08 pm

Well, there’s nothing new here. CO2 desorbing from oceans is one of the most classical positive feedbacks in global circulation models. End of the story.
REPLY: Which is the problem of alarmism in the first place, you never look beyond that, nobody wants to dig any deeper. “Man made CO2 is the cause, end of story”. As another psoter said: Get some integrity “Flanagan”. Look deeper instead of replaying talking points if you are capable. – Anthony Watts

Skeptic Tank
May 12, 2009 1:17 pm

Even the AGWers acknowledge that increased temperatures cause an increase in CO2 to be released from warmer oceans and thawing permafrost. So, that aspect of the argument is “settled”. It actually result in a negative feedback to their overall argument.

Rob
May 12, 2009 1:18 pm

HOW STRONGLY DOES THE SUN INFLUENCE CLIMATE?
Since the middle of the last century, the Sun is in a phase of unusually high activity, as indicated by frequent occurrences of sunspots, gas eruptions, and radiation storms. Researchers at the Max Planck Institute for Solar System Research (MPS) in Katlenburg-Lindau (Germany) and at the University of Oulu (Finland) have come to this conclusion after they have succeeded in reconstructing the solar activity based on the sunspot frequency since 850 AD. To this end, they have combined historical sunspot records with measurements of the frequency of radioactive isotopes in ice cores from Greenland and the Antarctic. As the scientists have reported in the renowned scientific journal, Physical Review Letters, since 1940 the mean sunspot number is higher than it has ever been in the last thousand years and two and a half times higher than the long term average. The temporal variation in the solar activity displays a similarity to that of the mean temperature of the Earth. These scientific results therefore bring the influence of the Sun on the terrestrial climate, and in particular its contribution to the global warming of the 20th century, into the forefront of current interest.
http://earthobservatory.nasa.gov/Newsroom/view.php?id=25149
Sun warms oceans, oceans outgas CO2, simple, the sun becomes highly active then CO2 starts to increase. Less cloud more cloud, the sun is still the driver as it was at the end of the little ice age. 220ppm as average CO2 level is rubbish, Beck disproved that number.

George E. Smith
May 12, 2009 1:21 pm

Well that is an interesting suggestion from Dr Spencer. The change from 10% MMCO2 (I don’t use five syllable words) to 20% may notreally be a worse fit than Roy says. The model (in red) seems to have a pesky curvature that is the real cause of the lack of fit.
Maybe the source of that too large curvature needs to be examined Roy; and you might do an even better fit. This would be a case where smooth curve fitting is appropriate since the real data curve is not very noisy.
I’m not sure my mind immediately grasps why the 6 month correlation delay goes away; but is that something you expected to happen Roy ?
Of course if you look at AlGorre’s book pages 66/67 where he has an 800 year delay instead of six months; then one could blame our present rising CO2 on the mediaeval Warm period which was 800 years ago.
One thing you can say about the warm ocean/more CO2 emission thesis, is that it is pretty basic physical chemistry so it seems irrefutable.
I too am a skeptic as to the C13 signature. Biologists tell me there are several kinds of plant CO2 metabolism, that have different C13 signatures; and in any case if fossil fuels are ancient plant material why wouldn’t they both have the same fingerprints.
Likewise the C14 signature is also a cosmic ray influence, so once again not definitive.
In any case, if we discovered a source of “fossil” fuel (petrochemical) that for some reason also contained 3% of dissolved Argon; and we started burning that stuff, and just ignoring the Argon, which would vent to the atmosphere; then we would expect over time, the atmospheric Argon would increase.
Likewise, releasing a source of carbon that is deficient in C13 would lead to the atmospheric C13 diminishing; but it would do exactly that even if the total atmopsheric CO2 never changed one iota.
So a changing isotopic ratio is not by itself evidence that an INCREASE in CO2 must be all fossil sourced. The C13 content would be expected to decline whether the total CO2 is going up or down or sideways; since we know we are releasing carbon from a source that is deficient in C13.
So I’m with you; whether C13 or C14; it is not conclusive that the atmospheric CO2 INCREASE is fossil carbon; just that fossil carbon is being introduced in some amount.
George

May 12, 2009 1:22 pm

Jim Papsdorf (12:06:42) :
It is great to see how your 90-10 Model serves to hang the AGW Alarmists on their own petards !!!!
Reminds me of the analysis by Rind and Lean that at most 10% of the warming can be ascribed to solar activity. Looks more and more like the oceans hold the key.

George E. Smith
May 12, 2009 1:28 pm

“”” Joel (11:56:17) :
Is it the physical properties of oceans that alter C02 absorption or is it biological? I could be backwards, but I’m having trouble understanding how a warmer ocean (regardless of why it’s warmer now) absorbs less CO2 than a cooler ocean, especially if the absorption is through marine life. Wouldn’t warmer temps increase plant life and therefore decrease C02? Either way it will be a glorious day if it is proven that this relationship is the main mechanism for CO2 fluctuations!! “””
Joel; there almost certainly is a biological takeup change along with a temperature change; and I have no idea which way and how much; but the effect that Roy is talking about is purely Physical Chemistry. Substance (a) has a certain solubility in substance (b), and not surprisingly the value of that solubility happens to be a function of temperature; and in this case CO2 just happens to be more soluble in colder water.
Let’s face it; there aren’t a whole lot of zoogremlins swimming around in your favorite Champaigne, but if you warm it up, it will outgas more CO2.
George

Jack Wedel
May 12, 2009 1:28 pm

You showed a plot for an 80-20 split, and increased model departure from actual. What happens when you run a 95-05 apportionment? Is the curve flattened further, but lower than actual? Perhaps a constant is needed to correct the deficiency.

May 12, 2009 1:33 pm

This demonstrates that a bottle filled with warm water warms while a bottle filled with air warms not….so much.

Wondering Aloud
May 12, 2009 1:33 pm

Joel
Carbon Dioxide like most gases is less soluble in water as temperature increases. If the water is warmer more of the CO2 is released in the air. Warm beer or whatever will give you more fizz than an icy clod one.

Dave in Delaware
May 12, 2009 1:40 pm

I have always found Dr Spenser’s explanations to be well thought out and easy to follow. Thank you again Dr Spenser.
In a related thought –
Discussions of Thermohaline Circulation, the ocean conveyor belt, tend to focus on present time down welling of the conveyor in the North Atlantic.
I have often wondered, but found little published, on what happens when and where the conveyor resurfaces in the Pacific and Indian Oceans. We are told that the ocean conveyor cycle takes 800 to 1800 years to complete a circuit. To my mind that says that we have been seeing the ‘return’ of deep ocean waters that disappeared down into the conveyor from roughly the time of the Medieval Warm Period (MWP).
Possible implications from ocean conveyor upwelling
* Since the conveyor starts in the cold North Atlantic, it should on average have more dissolved CO2 than the warmer Pacific and Indian Oceans, hence it would serve as a net source of CO2 released to the atmosphere in those regions.
* On top of that, the MWP atmospheric CO2 was likely as high or higher than present, so the dissolved CO2 that went into the conveyor from 800 years ago was likely higher than say the early Industrial Revolution / Little Ice Age atmospheric CO2 levels. That returning CO2 would tend to drive up present day atmospheric levels, and may have been adding CO2 to the atmosphere slowly over the past 200 years.
* Carbon/CO2 is also added to the deep levels of the conveyor by deposition from upper ocean organic matter. That added organic matter has a d13C (C13 isotope) content similar to petroleum derived from fossil fuels. The implication being, you can’t tell ‘old ocean’ CO2 from current man-made CO2 based on the C13 content.
The IPCC is only able to account for about half of the CO2 in the atmosphere if the driver were all man-made CO2. The up trend in atmospheric CO2 does not correlate well with man-made CO2 generated since the start of the Industrial Revolution and the natural atmospheric exchange of CO2 is more than able to absorb the relatively small man-made contribution on an annual basis. If real driver for increased atmospheric CO2 is the oceans, perhaps the ‘returned’ CO2 from the ocean conveyor is the source.
regards,
DJH P. E. (retired)

C. Boncelet
May 12, 2009 1:40 pm

Erik–ascribing a percentage to SST and to anthro is more complicated than simply looking at the ratio of a/(a+b). It’s a*SST/(a*SST+b*anthro). It depends on the relative units of SST and anthro.
(It appears Spencer may have missed this as well.)

May 12, 2009 1:42 pm

Forbidden relations:
Jupiter/Saturn/Sol
Sol/GCR/clouds
Sol/ozone/jet stream/Aleutians/PDO/Nino
CO2/breathing trees/exhaling men/beverages
CH4/Cows/barbeques

Jerker Andersson
May 12, 2009 1:43 pm

“What could be causing long-term warming of the oceans? My first choice for a mechanism would be a slight decrease in oceanic cloud cover. There is no way to rule this out observationally because our measurements of global cloud cover over the last 50 to 100 years are nowhere near good enough.”
Some studies indicate rapid changes in earths albedo right before the super el nino 1997/98.
Se link below.
http://www.bbso.njit.edu/science_may28.html
I dont know how accurate that data is though but if it is correct it seems to me that the jump upwards in temperatures in the mid 1990 and beginning of 2000 was caused by less cloudcover during a few years.
Since it is obvious that the CO2 increase is sensitive to global temperature changes one could calculate how much CO2 would raise at different temperatures. Based on the Mauna loa and RSS global temp data I estimate the CO2 would roughly stop increasing if the yearly global temperature would drop to -0.3 to -0.4 (RSS). That would be equal to the global temperatures near 1900.

Ed Scott
May 12, 2009 1:46 pm

SPPI Monthly CO2 Report
April 2009 | Volume 1 | Issue 4
http://scienceandpublicpolicy.org/images/stories/papers/originals/april_09_co2_report.pdf
———————————————————–
Chuck it yet again, Schmidt!
A personal note by Christopher Monckton of Brenchley
http://scienceandpublicpolicy.org/images/stories/papers/commentaries/chuck_yet_again_schmidt.pdf

Claude Harvey
May 12, 2009 1:46 pm

Two questions pop out of this paper:
1) It concludes that 10% of observed increases in CO2 are the result of human activity and 90% are from natural sources. The figures I see comparing current total atmospheric CO2 with that of the four previous and comparable global temperature peaks of the past 450,000 years indicate CO2 content is currently 36% higher than it was during those past peaks. Although that variance does not rule out “natural”, it certainly raises questions.
2) We’ve recently seen a paper on this site indicating that ocean temperatures have actually declined over the past 100 years, yet we know atmospheric CO2 has risen during that period. Isn’t this contrary to Spencer’s logic?

pyromancer76
May 12, 2009 1:47 pm

It is always helpful to get us readers back to basics, Dr. Spencer — the foundations of science. Thank you for this contribution and I read it the first time on your blog.

ak
May 12, 2009 1:48 pm

“Are we really sure that ALL of the atmospheric increase in CO2 is from humanity’s emissions? After all, the natural sources and sinks of CO2 are about 20 times the anthropogenic source, so all it would take is a small imbalance in the natural flows to rival the anthropogenic source. ”
The thing I’m wondering about is, with a residence time of 5 years in the atmosphere, when does anthropogenic CO2 stop being anthropogenic and become natural? If there’s more CO2 now than 100 years, which is clearly seen, there is that much more CO2 to be within the various sinks.

bsneath
May 12, 2009 1:48 pm

I strenuously and vehemently object. I refuse to believe any analysis that lacks hundreds, or even better thousands of interactive and codependent variables, with positive feedback loops and tipping points. I further question the reliability of any analysis that is not dependent upon future federal funds, with the full understanding that such funds are contingent upon the model showing that AGW is real and present danger to mankind. I simply cannot accept any data or evidence that is contrary to my preconceived and strongly held beliefs that we must protect our fragile planet’s ecosystems, habitats, wetlands and species (words with which I was indoctrinated beginning with my “Weekly Reader” in elementary school) from the ravages of evil corporations, developers, suburbanites and especially Deniers!
(So how did I do?)

Stephen Skinner
May 12, 2009 1:52 pm

Just to throw in a thought. Could the rise in CO2 be in step with the rate at which vegetation is being cleared? Land clearance has been relatively in step with population growth and along with the advances in technology the physical effect of population growth over recent decades is probably augmented.
As far as what is driving SST I wonder if direct solar heating is overstated and there is much more exchanging of heat between land, sea and air. The focus of attention appears to be primarily the amount of sea surface exposed to the sun, but equally the same sea surface is exposed to the same area of sky with whatever heat it contains or extracts. Infra red images of the oceans imply that the shallows or coastal areas are the hottest with these warm waters being ‘dragged’ out and away from the coasts. I expect to see temperature difference on land because of the variety of surfaces but apart from the affect of the earths rotation I would expect to see more uniformity in SST as water is much the same over most of the planet including it’s height if it is warmed solely by direct solar radiation.

Jan Breslow
May 12, 2009 1:54 pm

In Spencer’s book entitled: “Climate Confusion?”, there is a chapter on climate that rebukes the steady state “blanket theory”. The model he presents is anything but steady state, including absorbing and radiating heat during the day and only radiating at night, as well as conducting heat from the equator (where it is hottest) to the poles and into the stratosphere where it is radiated into space. Spencer talks about water playing the major role with evaporation at ground level and sublimation at higher altitudes transferring tremendous amounts of energy. In this context I read an essay listed on the ICECAP site by Jules Klabfeld arguing that there are simply too few CO2 molecules in the atmosphere to transfer a significant (measureable) amount of heat compared to the other molecules in the atmosphere and to water, which as part of the heat transfer cycle undergoes phase transitions involving many more molecules than what is at any one moment in gaseous form. Jules Klabfeld’s arguments seem reasonable to me, but is there anyone out there with basic knowledge of thermodynamics who might comment. If anyone thinks this is a reasonable argument, can it be put into the context of how much heat needs to be transferred in the non-steady state theory of climate and what fraction CO2 would be capable of transferring?
Klabfeld Essay: http://icecap.us/images/uploads/CHALLENGE_7b_1.cwk_(WP)(2).pdf

Tarnsman
May 12, 2009 2:03 pm

Joel (11:56:17) :
“Is it the physical properties of oceans that alter C02 absorption or is it biological? I could be backwards, but I’m having trouble understanding how a warmer ocean (regardless of why it’s warmer now) absorbs less CO2 than a cooler ocean, especially if the absorption is through marine life. Wouldn’t warmer temps increase plant life and therefore decrease C02? ”
Where do the baleen whales go to fat up? The tropics or the polar seas? Answer: the polar seas. Why? Because that is where the greatest concentrations of plankton are to be found. Why? Because the colder water in the polar seas carries more nutrients and CO2 than the warmer water of the tropics. The polar seas are the richest marine environments in terms of marine flora and fauna. So the answer to your question is: No, warmer temperatures would cause a decrease in marine plant life (less CO2 in the water) and thus reduce their mitigation of CO2 levels through photosynthesis.

crucilandia
May 12, 2009 2:03 pm

That was awesome. Thank you

John
May 12, 2009 2:05 pm

Spencer’s simple model is very interesting, but as he implies, the simplicity allows for questions.
One question has to do with sulfates and black carbon. There was global dimming though 1990, global brightening since, due evidently mainly to the fall of the FSU and the huge drop in SO2 emissions as industry went bust there and in eastern Europe, and secondarily due to acid rain programs in the US and Europe). It would be good if Spencer could somehow accomodate these changes, which should affect SSTs, and which certainly could cause warmer SSTs after Pinatubo’s effects diminished. Even if cloud cover didn’t change, reduced sulfate would mean more sunlight penetrates to the surface, because in low humidity, sulfate forms a light colored aerosol. It was only around 2000 or so that increased Chinese emissions caused worldwide SO2 drops to level off and reverse a little.
Secondly, volcanos should be included in the model. After Pinatubo, temps decreased and cloudiness increased for a year or two, which caused vegetation to actually grow better for a couple of years, which sucked a bit more CO2 from the air — which is why in 1992 only about 0.4 ppm of CO2 increase was recorded at Mauna Loa.
Third question: posts on WUWT discuss the PDO, and the fact that it has been in a warm phase until recently — could this be a cause of the increase in SSTs, instead of a hypothesized change in cloud cover?
Spencer could easily be right about the cloud cover, but I’d like to ask him to address these questions to see how robust his argument is.

urederra
May 12, 2009 2:06 pm

(11:56:17) :

Is it the physical properties of oceans that alter C02 absorption or is it biological? I could be backwards, but I’m having trouble understanding how a warmer ocean (regardless of why it’s warmer now) absorbs less CO2 than a cooler ocean, especially if the absorption is through marine life. Wouldn’t warmer temps increase plant life and therefore decrease C02? Either way it will be a glorious day if it is proven that this relationship is the main mechanism for CO2 fluctuations!!

from here…
http://www.elmhurst.edu/~chm/vchembook/174temppres.html

As the temperature increases, the solubility of a gas decrease as shown by the downward trend in the graph .
More gas is present in a solution with a lower temperature compared to a solution with a higher temperature.
The reason for this gas solubility relationship with temperature is very similar to the reason that vapor pressure increases with temperature. Increased temperature causes an increase in kinetic energy. The higher kinetic energy causes more motion in molecules which break intermolecular bonds and escape from solution.

May 12, 2009 2:07 pm

Cassanders (12:44:55) :
for both prophet and priest are profane; yea, in my house have I found their wickedness, saith the LORD.
Only the prophet speaketh lies about CO2. Don’t believe him! CO2 is the gas we all God’s children exhale, and green trees breath. HE is profane!

Michael
May 12, 2009 2:15 pm

If 1930’s temperature is the point where oceans turn from a sink to a source, then I recommend that temperature as the zero baseline for graphs of surface temperature anomalies since CO2 is the devil de jour.

Dave in Delaware
May 12, 2009 2:26 pm

…. and an apology for my incorrect spelling of Dr. Spencer’s name.

paul maynard
May 12, 2009 2:27 pm

Joel’s Question
I don’t think anybody answered it.
I think it is Henry’s law that a liquid, e.g water can hold less gas in solution as its temperature rises and vice versa.
WUWT published a thread a while back in which Dvaid Archibald used this effect to explain the Vostok Ice Core record that showed CO2 increasing some 800 years after temps rising as a result of outgassing of CO2 from the sea which holds over 50 times the amount of CO2 dissolved in solution as there is in the atmosphere.
Paul

Joseph
May 12, 2009 2:28 pm

Roy,
In your model equation, what values were used for the variable “Anthro”?
What would happen if the Anthro component were simply omitted? Can a value be found for the “a” coefficient that results in the model fitting the observations even better?

Ron de Haan
May 12, 2009 2:38 pm

It’s not the fact that warmists have an opposite opinion that pisses me off.
It’s the lack of sound arguments and open discussion, the manipulation, the lying, the cherry picking, the arrogance, the style and the political agenda that makes me mad.
This kind of articles shuts them up all right.

Fernando
May 12, 2009 2:57 pm

Eager to read the comment by Ferdinand Engelbeen

May 12, 2009 3:06 pm

Our nice friend George said:
“Let’s face it; there aren’t a whole lot of zoogremlins swimming around in your favorite Champagne, but if you warm it up, it will outgas more CO2.”
Let’s also face that if you allow your champagne to warm you clearly cannot be trusted with champagne.

JamesG
May 12, 2009 3:31 pm

I like to mention the Nevada FACE facility where they measure background CO2 continuously in a desert environment and which for 10 years has shown little or no trend (though a lot of scatter). Nobody ever comments about it but I see it as important evidence. Presumably these scientists are doing the background measurements correctly otherwise the whole experiment would be pretty pointless.
http://www.unlv.edu/Climate_Change_Research/NDFF/co2_treatment.htm
I’ve been wondering why Kealing never chose a desert environment; a short drive from his office, after having already reported in his talks that he got low background readings in such environments. I mean why a volcano in the Pacific? What on earth was he thinking about? Even today on the ML website they say it’s the cleanest air in the world which is nonsense: It’s a volcano for Pete’s sake – and they admit they need to correct for it’s emissions! However that’s a side issue. It just makes me wonder about Kealing’s decision-making mechanism. But maybe he was a surfer.
No doubt Engelbeen will pop in and link his website and mention that the Antarctic station agrees with ML.

Ivan
May 12, 2009 3:38 pm

Interesting feature of Spencer’s analysis is that it implies Segalstad/JAworowski kind of criticism of IPCC interpretation of ice core records. If XX century rise in CO2 concentration up to 380 ppmv was mainly natural process than it is highly unlikely that highest concentration in previous 650 000 years could be only 280/290 ppmv, as “consensus science” assume. The most basic IPCC cornerstone simply must be wrong (as Segalstad and Jaworowski conjecture).

Nick Stokes
May 12, 2009 3:42 pm

Well, some more simple arithmetic. We burn nearly 10 Gigatons of C per year. The atmospheric increase is about 5-6 Gt. The rest is thought to go into the ocean.
But if the ocean is a nett source of CO2 to the air, then where does it all go?

Ray
May 12, 2009 3:44 pm
May 12, 2009 3:49 pm

Rob (13:18:07) :
Since the middle of the last century, the Sun is in a phase of unusually high activity
There is good evidence that this is not the case. A starting point for you might be this: http://www.leif.org/research/Napa%20Solar%20Cycle%2024.pdf before we go to the next [more technical] level.

May 12, 2009 3:52 pm

As Al Gore would say…. “Did they ever fit together?”

JamesG
May 12, 2009 4:00 pm

To Nick Stokes, re “Where does it all go”
The earth is obviously greening:
http://earthobservatory.nasa.gov/Features/GlobalGarden/
Now where do you suppose trees gets their carbon from? Maybe you can use some simple arithmetic.

Philip_B
May 12, 2009 4:05 pm

If Dr Spencer is right then we should see a sharp fall in the rate of CO2 increase in the next year.
If you look at the annual mean growth rate of CO2 at Manu Loa (link below) then there does seem to be decrease over the last 3 or 4 years. Although year to year variability is high and is increasing.
BTW, the year to variability can’t be explained by the anthropogenic CO2 theory and is serious problem for it.
http://www.esrl.noaa.gov/gmd/ccgg/trends/

May 12, 2009 4:09 pm

The long-term record shows that the CO2 changes trail the temperature changes by roughly 800 years. When I have tried to discuss this with True Believers, I’m told that there is a feedback process under which the changes are amplified by the increased release of dissolved CO2 from the higher temperatures, and I’m just too simple to understand the science, which has already dealt with fools like me, so shut up.
Well, here’s what I don’t understand. After 90 thousand years of cold temperatures, when the CO2 levels are at their lowest, the temperatures start rising, and continue to rise for that eight century interval before the CO2 levels begin to go up. Similarly, after ten thousand years of interglacial, when the CO2 is at its highest, the temperatures start dropping, and continue to do so for eight centuries before the CO2 levels follow.
How is it that CO2 levels that have been stable for thousands of years cause 800 years of temperature changes in the first place?

Nick Stokes
May 12, 2009 4:17 pm

JamesG
More simple arithmetic – the plant biomass is about 500 Gt carbon. This Spencer theory would imply an increase of about 10 Gt/year, about 2%. If that has been going on for a while, we’d notice. To put it another way, we’ve burnt in total over 300 Gt. That would be a 60% increase.
Your link says about 6% increase in productivity over 25 years (which isn’t quite the same).

RW
May 12, 2009 4:21 pm

“Global warming theory assumes that the increasing carbon dioxide concentration in the atmosphere comes entirely from anthropogenic sources, and it is that CO2 increase which is causing global warming.”
These things are not ‘assumed’.
“is it possible that long-term warming of the oceans (say, due to a natural change in cloud cover) might be causing some portion of the long-term increase in atmospheric CO2?”
Why, of course. But we know that this portion is very small. How we know this should be common knowledge among anyone interested in climate science, but let’s run through it one more time:
Observation 1: concentration of atmospheric CO2 is rising steeply. It is 40% higher than it was at any point in the 800,000 years or so before 1800.
Observation 2: not only is the concentration rising steeply, but its isotopic composition is changing. After 800,000 years or so of little variation, the proportion of CO2 molecules containing a 13C atom began dropping sharply about 200 years ago.
Observation 3: In the oceans, a similar drop in 13C is observed, and the quantity of CO2 in the oceans is found to be rising.
Conclusion: the oceans cannot be the source of the extra CO2, because if they were, then a) the amount of CO2 in them would be falling, not rising; and b) the δ13C in the oceans would have to be rising, not falling. Roy’s ideas fail to explain the observations, and must therefore be discarded.
Instead, we can see that obviously, the extra CO2 must be coming from a reservoir with a lower δ13C than both atmosphere and oceans. Who might know what that could be?
If, despite the isotopic evidence, you want to believe that warming has caused the rise in CO2, then you can see from ice cores that the relation between CO2 and temperature that held for 800,000 years or so was that a 1°C rise in temperature corresponded to about an 8ppm rise in CO2. Therefore, to explain the ~110ppm rise in CO2 since 1800, we require a temperature rise of 14°C. Hands up if you believe it’s got 14°C warmer globally since 1800.
“The best model fit assumes that the temperature anomaly at which the ocean switches between a sink and a source of CO2 for the atmosphere is -0.2 deg. C… it would mean that the ocean became a net source of more atmospheric CO2 around 1930.”
The oceans are a net sink of CO2. Therefore, the model fails.
“And just how strenuous and vehement the resulting objections are to what I have presented above will be a good indication of how politicized the science of global warming has become.”
How outrageously unscientific, to pre-emptively claim that all possible objections must be political. Strenuous and vehement objections are the only sensible scientific response to a deeply flawed analysis. Politics is irrelevant.

May 12, 2009 4:24 pm

JamesG
In his autobiography Keeling says he took the job there because it was more fun and would be outdoors (he was a keen outdoors man) and better than the alternative offered-a job in an office based environment . Nothing more scientific than that, as he had no experience whatsoever of co2 analysis before taking up the job.
tonyb

May 12, 2009 4:46 pm

It’s interesting. I was looking for a way to comment on Dr. Spencer’s blog but there was no link. I had a few questions for him, one related to data source and the other was why didn’t he do a multivariate regression to find the ratio between natural and AGW rather than hand match the temps.
Either way, it’s an interesting analysis and if we trust the long term temperatures it might be worthwhile to do the regression.

May 12, 2009 4:53 pm

Or maybe CO2 and temperature are so loosely connected that the relationship won’t show for hundreds of years. How did CO2 and temperature get so closely coupled — Just because Al Gore says?

Chris Knight
May 12, 2009 4:55 pm

Joel
Your question may have been answered.
1) First the carbon dioxide needs to get into the ocean.
2) Second, the ocean needs to be cool enough to retain it for long enough to allow photosynthetic plankton to assimilate it.
3) Then the rest of the plankton food chain needs to really incorporate the carbon products into the biomass.
1) is not often considered thoroughly – there is not sufficient ocean surface area for effective diffusion of carbon dioxide from the atmosphere into the ocean to support the bulk of photosynthetic life there. The key source is precipitated rain water, saturated with carbon dioxide, mainly in temperate and subpolar zones. In tropical zones, both on land and in the ocean, the respiration of the biomass accounts for much of the circulating carbon.
2) Tropical oceans, and tropical landmasses recycle much of the carbon dioxide reaching the surface dissolved in rain directly back into the atmosphere. Subtropical desert regions consolidate carbon dioxide into carbonate minerals which fix sand dunes and mineral-rich lake deposits.
3) Biomass has two parts – the organic and the inorganic. The organic bit -protein, carbohydrate, lipid etc., and the inorganic stuff – shells, bones, dental stuff, maybe I can include resin and some resistant plant products here too, stuff that is not easily biodegradable. The organic stuff is readily recycled, but the inorganic stuff is either fossilized, or incorporated into sediments. Some scientists think that even the inorganic stuff is recycled into petroleum products, as long as there is a source of carbon and hydrogen, and the temperature and pressure are high enough.

E.M.Smith
Editor
May 12, 2009 5:02 pm

Nicely done.
I, too, would suspect that you need a 95-5 run with an offset to flatten the curve. Potentially, there is also an algae bloom vs sunlight cloud modulation distractor…
FWIW I got sent down the C12 / C13 isotope ratio rat hole too. The Result:
http://chiefio.wordpress.com/2009/02/25/the-trouble-with-c12-c13-ratios/
Enjoy.
BTW, one of the simplest things wrong with the C12/13 ratio is that each fossil fuel has a unique ratio and it varies by deposit of “stuff”; so unless you know where all the past oil, coal, and gas came from and what it’s ratio was then, you must “guess”.
Another? How much CO2 comes from mid ocean ridges? Is it all the same ratio? How much from methane seepage from clathrates? With what ratio? What is the impact of ocean floor deposits and plate subduction on the CO2 ratios produced in the volcanos above those plates? With what time lag? Was the CO2 ratio different in the past when there was different “stuff” on the ocean floor? By how much? What’s down there now? How much oil, coal, gas is consumed by bacteria each year everywhere on the planet? With what ratios? …
You get the idea. What we don’t know swamps what we do. But that still doesn’t seem to stop folks from calculating fantasies to 2 decimal points…
Oh, and there are two major plant metabolisms (called C3 and C4 for the number of carbons in a key step). They consume C12 / C13 in different ratios. How much C4 metabolism biomass has existed on the planet at all times of interest for production of soil carbon (thus determining the ratio of releases from all soils…)? (Hint: Grasses are the major – but not only- C4 plant type. They are only about 6 million years old…)
It’s not a simple problem and the answer that “we know” is just hubris.

J. Peden
May 12, 2009 5:07 pm

Flanagan (13:08:37) :
Well, there’s nothing new here. CO2 desorbing from oceans is one of the most classical positive feedbacks in global circulation models. End of the story.
Exactly, any positive feedback from additional CO2 seems to be over. Shouldn’t we already have had a temperature “runaway” without any additional anthropogenic CO2 whatsoever?

pft
May 12, 2009 5:14 pm

When I first started looking into AGW for myself I was trying to disprove a colleague who was a skeptic. But when I read articles or blogs that supported the AGW theory, especially of the alarmist variety, I realized something was not right as the arguments in favour were fuzzy, not very clear, more like an illusion one must accept as a matter of faith. You could not grab hold anything substantial that would give clarity to how man alone was causing this warming, which did not seem substantial or unusual from a historical perspective, and how the problem could spiral out of control- other than the models say so.
But articles like Spencers exist in abundance on the the other side of the debate, and they encourage debate. They make no claims to certainty and do not insist the science is settled, and their articles tend to lift the fog on this fuzzy science, and are not hidden behind subscription walls. The science as they present it is logical and makes sense and relatively free of assumptions that are not supported by evidence or a healthy understanding of the process (eg. Climate models seem to minimize solar forcing, clouds and precipitation efficiency effects on climate due to a low level of understanding and lack of accurate historical data or observations).
Unfortunately, the alarmists will always denounce any articles that rock their world as being funded by Big Oil scientists, like Spencer is accused of, and fail to acknowledge there is a profit motive and political motive on their side. And since the debate is closed, this article like many others will simply be ignored.
Then there is peer review. Articles tend to not get published which challenge the establishments consensus. Peer review is a form of censorship. Einstein never had his great theories subject to peer review until he went up against Bohr and challenged the Quantum Theory as being incomplete. This theory had achieved consensus status in Copenhagen in 1927 and the science was said to be settled.
His first paper was rejected in 1935, Einstein had never heard about peer review before this and was furious the paper had been to others without his permission or knowledge who they were. In the past peer review took place after publishing, although before publishing scientists would send their drafts to other scientists they respected for comments. He then sent it in to a journal which did not do peer review and got published.
I have also discovered this corrosive influence is not limited to Climate Science but is pervasive in all science and even social sciences, including economics and history. Thats OT, so thats all I will say about that.

Gary
May 12, 2009 5:20 pm

Monster,
The primary driver of climate change is the interaction of the earth’s orbital parameters: eccentricity, tilt, and wobble which were proposed by Milankovich more than a century ago and verified in the 1970-80s by the CLIMAP and SPECMAP projects. These cycles produce the large-scale glacial-interglacial oscillations. What happens on the shorter terms (millennial, century, and decade) can be attributed to other causes.

E.M.Smith
Editor
May 12, 2009 5:29 pm

JamesG (15:31:32) : I’ve been wondering why Kealing never chose a desert environment; a short drive from his office, after having already reported in his talks that he got low background readings in such environments. I mean why a volcano in the Pacific? What on earth was he thinking about? […] It just makes me wonder about Kealing’s decision-making mechanism.
Let me think… I can put a CO2 station where I will spend years of my life in the boring stinking empty hot desert, or on the side of the mountain in a tropical paradise with fabulous vacation accommodations, floor shows, fantastic rum drinks, lush tropical forests to die for, and views that can’t be beat from the best beaches in the world.
Decisions decisions… (I don’t think it’s his “decision-making mechanism” that has issues 😉
Me? I’d have chosen South Island New Zealand … Then again, I like both beaches AND skiing… and roast leg of lamb, with those wonderful light red wines the Kiwis make … but the mind wanders… Oh, and the air would be really really clean and well mixed there, yeah, that’s it, well mixed.

Robert Kral
May 12, 2009 5:34 pm

I have recently seen dire predictions about higher atmospheric CO2 acidifying the oceans and disrupting the ability of shellfish to deposit calcium carbonate in their shells efficiently. No doubt this is the beginning of the fall-back position when temperature changes have obviously failed to follow CO2 levels.
RW, your points may be valid (or not) but I am struck by the way you so confidently assume that the whole business comes down to the variability of CO2. First, your line of argument completely fails to deal with the established facts that the world has been both much warmer and much colder than it is now, before the emergence of humans and after. Since everything that we’re observing now is clearly within the bounds of natural variability, why is it necessary to invoke human causation? In fact, why is it not specious to do so before carefully eliminating all possible natural causes?
Second, I know from my own line of work that when you try to explain the behavior of highly complex systems in terms of a single variable you’re going to be wrong. It’s called “excessive reductionism”, and it’s an unhealthy phenomenon, especially when you don’t really know what all the other variables are.

3x2
May 12, 2009 5:36 pm

Joel (11:56:17) :
Is it the physical properties of oceans that alter C02 absorption or is it biological? (…)

Both? We know that colder Oceans will hold more C02 and that once the Oceans warm C02 is released. At the same time, within SST/Biological limits, Planktonic life will bloom due to higher temps and more free C02 (food). The question, in my mind at least, would be how much of a buffer does that Biological system create? You would see the physical out gassing with warmer SST’s buffered to some extent by biological growth and re-absorption. The question would concern the ratio of biological buffer v. Physical “Coke-fizz”.
Keep in mind also that the entire biosphere blooms on various time scales. Our observed Oceanic Planktonic bloom, although possibly short lived, is transferred more permanently through various stages into Herring and then to Herring eating Whales. The “carbon” trapped and released would create harmonics in the C02 record. As far as I can see there is a (short time scale) biological multiplier at work here in that the average lifespan of the Carbon trapped by Plankton may be months but by the time Herring are the beneficiaries that lifespan is now years. By the time our specimen Herring is eaten by a Whale the “carbon” is trapped and released on decadal scales. The same must be true for land based life also.
This may be one reason why purely “Physical” models of C02 and climate have proved so inadequate. Climate is not just some 2D “energy transfer” model. Apart from the complications of “energy transfer” when applied to water vapour (!) you can add some further “complications” when including biological systems (!).
As there have been a few posts concerning Dr Spencer personally , I would like to say that I enjoy reading his work and thoughts. To me he is what “science” should be about. He seems to question his own data and conclusions as often as he does those of others and that should be how science works. Do surface temps govern clouds or do clouds govern surface temps? I may not be a climate scientist but I understand
exactly what kind of questions he is asking and attempting to answer. Unfortunately he tends toward real data and we can’t have that in Nintendo science.

May 12, 2009 5:41 pm

Flanagan (13:08:37) : Look, CO2 it is just a tiny part of the atmosphere, only a 3.85 per TEN THOUSAND (provided we believe in Mauna Loa-a Volcano-figures).
So, it is a trace gas, which you and me, we humans, EXHALE , and plants/trees breath. It is a small part of the air, which DOES NOT can hold heat as compared with water (3,227 times less).
Our planet earth, I don´t know if you ever look above to the sky, IT IS NOT CLOSED, as within a crystal box, it is OPEN TO SPACE, there it goes the warm by a process which makes baloons fly up, ya know? , that´s called convection. So, when air gets some heat it goes up there and loses it.
Read: http://www.giurfa.com/gh_experiments.pdf

Richard M
May 12, 2009 5:42 pm

I posted this OT in another thread and got no replies. Hopefully, I’ll get a response here.
—————————
Maybe someone can help me understand something that has been bothering me for sometime.
Why don’t we have CO2 measuring capabilities at every NOAA weather site?
It seems to me we know from Beck’s work that local CO2 amounts vary significantly from location to location. This is even used by warmists to discredit Beck’s work. So, if there are local differences that can be up to thousands of PPM then it seems like that should also impact the local temperatures. It would also seem like forecasting weather would be severely hampered without CO2 monitoring since the effects of radiation are immediate.
Of course, if we had this capability then I can think of several ways to demonstrate the validity of the CO2 effects on temperature. So, why haven’t climate scientists been up in arms demanding that we have this data?
I realize that the satellite that was lost could have provided some of this information. And, other satellites are available. But it seems to me very local CO2 monitoring stations where the temp is monitored right next door would be invaluable to understanding AGW.
Why aren’t we putting the dollars budgeted for improved models into collecting this data. Can anyone tell me where I’m off base?

Rathtyen
May 12, 2009 5:48 pm

Try an experiment of going outside on a warm, cloudy day. When exposed to the sun, you can feel the heat. When a cloud passes over, the shadow induces a change in the heat intensity you feel, making it noticeably cooler (ie not necessarily cool, but cooler). Unlike permanent shade, this isn’t the cooler air, it is the result of a partial blocking of the sun.
Given what we know of the solar cycles, a strong sun during a solar maximum has more power and likely results in less cloud cover overall. It would then be expected to have a ocean heating effect. The slightly warmed oceans are therefore more chemically active, both releasing more CO2 and increasing organic decay rates. I would also presume the increased surface warming would trigger greater ocean current activity, churning up more CO2.
Once the opposite occurs, a solar minimum reduces the direct warming effect and triggers greater cloud cover, resulting in increased shade. This causes a marginal cooling, reducing CO2 releases, slowing decay, and reducing ocean current movement.
As these are slow processes resulting in marginal changes, the effects become more noticeable over longer periods of time (ie the time duration of the Solar Maximum or Minimum counts), as well as being contingent on the intensity. Timing would also count with regards to other cyclical events, whether solar or Terran, as these would either amplify or offset each other.
Mt understanding is that at present, the Solar Cycle, Pacific Decadal Oscillation, Inter-Decal Oscillation and Southern Oscillation Index have all recently swung from “warm” to “cool” (with the others I’m not sure, although the Indian Ocean patterns are still casting a dry effect on Australia despite multi-year La Ninas). Its when they align that things appear to get interesting, and if we are entering such a period of alignment (and it seems difficult to determine what level of inter-causality exists), then we should expect an ocean cooling effect and, if Dr Spencer is correct, a drop in CO2 levels.

E.M.Smith
Editor
May 12, 2009 5:52 pm

Oh, and speaking about “what we don’t know about the CO2 in the ocean” there is also this:
http://environmentalresearchweb.org/cws/article/research/37370
from January of this year. A quote or two:
Fish ‘gut-rocks’ solve ocean puzzle
For decades marine scientists have been perplexed by the increase in alkalinity with depth in the top 1000 m of the ocean surface when chemistry suggests this should only take place lower down. But now, a team from the UK, US and Canada reckons excretion of a highly soluble form of carbonate from fish intestines could go some way to solving the mystery.
“Our most conservative estimates suggest three to 15% of the oceans’ carbonates come from fish, but this range could be up to three times higher,” said Rod Wilson of the University of Exeter, UK.

Notice the large ranges? 3% to 15% but maybe 3 x that…
Now what was that you were saying about “it had to come from people” based on some hypothetical ocean CO2 accounting? We haven’t a clue…
“We also know that fish carbonates differ considerably from those produced by plankton,” said Wilson. “Together these findings may help answer a long-standing puzzle facing marine chemists, but they also reveal limitations to our current understanding of the marine carbon cycle.”
The carbonate the fish excrete is high in magnesium and more soluble than the forms of carbonate produced by plankton. As a result it can dissolve at higher levels of the ocean.
Together with colleagues from the University of Miami, University of Ottawa in Canada, University of British Columbia, Canada, and the University of East Anglia, UK, Wilson estimated the total biomass of bony fish in the world’s oceans as between 812 million and 2050 million tonnes, leading to a total carbonate production of around 110 million tonnes.

Again with the 812 to 2050 … kind of a wide range, eh what? Yet we end up with a single nice 110 million tonnes answer… at least it has an ‘around’…
Now here’s a little thought for you:
We’ve reach “Peak Fish” some decade or two ago. (As of now some 30% of all fish eaten is aquacultured so nobody cared much about the Peak Fish crisis…) The fastest we can harvest them from the ocean. Do you think that maybe hauling billions of pound of fish out of the ocean might reduce the quantity of carbonate pellets the fish in the ocean can excrete?
Do you think that might leave more CO2 in the ocean to outgas?
Do you think that might raise the CO2 level in the air?
Wether that it A Good Thing or A Bad Thing I’ll leave for another day… “Just think what you will know tomorrow” – Men In Black.

May 12, 2009 6:01 pm

Ed Scott, thanks for those great links:
click1
click2
In the first link, Gavin Schmidt has been gelded by Monckton. Did Schmidt actually believe that he could get away with his prevarications online?? Altering graphs produced by someone else is thoroughly dishonest. It is fraud, isn’t it? It appears that people check their ethics at the door when they enter GISS/RealClimate territory.
Also, regarding human CO2 emissions: click. That puts the minuscule human CO2 emissions in perspective. Hardly alarming.
Finally, WUWT trumps RealClimate once again. We have a genuine Viscount on our team! How cool is that?

George E. Smith
May 12, 2009 6:02 pm

“”” Flanagan (13:08:37) :
Well, there’s nothing new here. CO2 desorbing from oceans is one of the most classical positive feedbacks in global circulation models. End of the story. “””
Well it might also be one of the false “feedbacks” as well.
A general principle of GHG “global warming” is that a GHG such as water vapor or even CO2 absorbs surface emitted long wave infrared radiation which is in the 13.5-16.5 micron wavelength range for CO2, and covers a much larger spectral range in the case of water vapor. That captured energy is transferred to the normal atmospheric gases through molecular collision processes; which apparently happen sooner than spontaneous re-emission from the capturing molecule (at low altitudes and higher pressures. The net result is a heating of the atmospheric gases themselves (N2, O2, Ar). Those ordinary atmospheric gases couldn’t care less what species of absorbing molecule crashed into them to convey that heating energy to them. As a result of that heating of the atmosphere, the atmosphere being physical matter above zero K emits a thermal radiation spectrum that depends on the material (atmosphere) temperature, generally following in some fashion the black body radiation spectrum. So as a result of atmospheric heating the amount of long wave radiation emitted from that material increases probably as the 4th power of the temperature (K). That radiation is generally emitted in an isotropic radiation pattern; so about half of it can be expected to head downwards towards the earth; about 705 of which is actually the oceans. The remaining 30% which lands on material which is other than water results in no further CO2 emission to the atmosphere; so no positive feedback there.
The 70% that does strike the oceans or any other water gets absorbed in the top 10 microns of the water surface, resulting in rather prompt increase in evaporation due to the extra surface energy.
The amount of heating of the oceans due to that long wave IR can’t be very large, because only a few microns of the surface absorbs it; and the resulting water evaporation also carries a lot of latent heat back into the atmosphere.
So if you are looking for GHG caused atmospheric warming to warm enough ocean water to disgorge a bunch more CO2; my guess is you will look for a long time.
So I think your concept of a positive feedback effect from further emission of CO2 from long wave IR heated ocean water, is all wet. Now incoming sunlight which penetrates deep in the oceans and has a much higher irradiance level that the atmospheric IR emissions; will in fact warm a lot of water and cause increased CO2 emissions.
Not all em radiations are created equal, and the thermal effects of long wave IR are quite different from those due to solar spectrum radiation when it comes to the effect on water. Notice that the thermal radiation from the atmosphere should carry no fingerprints of whatever molecular species captured that energy in the first place, because that atmospheric radiation is coming from ordinary N2, O2, and Ar molecules.
I agree that if the ocean isn’t warming (well the MMGWCC alarmists claim it is warming so fast it is going to flood all of us), then it wouldn’t be outgassing CO2 at an increased rate.
The fact that CO2 in the atmosphere and the oceans is decreasing is only evidence that a source of depleted C13 carbon is being burned; it is not evindence that that source is the origin of the increased CO2 in the atmosphere.
In a past life I used to grow epitaxial layers of the mixed alloy GaAs(1-x)P(x) single crystal on GaAs substrates; from the vapor phase. We fed Hydrogen containing a percentage of Arsine (AsH3) through a capillary, into a flask (several litres), and then out of that flask into another capillary; the capillaries were actually coiled up inside the flask to save space for one reason. after going through typically three such capillary flask combinations the gas mixture was fed into the reactor where the vapor deposition took place. After growing an initial layer of GaAs on the substrate; the fow of Arsine bearing hydrogen was cut back, and a new source of hydrogen containing phosphine (PH3) replaced it, so we started feeding phosphine into the flasks which already were stabilised with a certain percentage or Arsine; so the ratio of Arsine to phosphine in the first tank, slowly changed building up phosp[hine at the expense of arsine. That changing mixture headed into the next capillary, and started changing the mixture in there, and so on to the third flask. The capilary/flask combination behavbes like an RC time constant integrator, and the three of them in series resulted in a near Gaussian leading edge with a prescribed rise time, creating a smooth change from zero phosphine to around 45% or so to 55% Arsine, that resulted in a transition from pure GaAs deposited crystal to GaAs0.6P0.4 crystal, that being the recipe for the then brightest red LED material.
At no time did the total hydrogen flow change; it was maintained constant; but the output composition of reagent gases was smoothly changed from one mixture to another. And yes I used ordinary elecric filter circuit theory to design the three pole Gaussian filter that shaped the transition process.
So don’t try fooling this old codger that a changing atmospheric composition and amount means the source of the composition change is also the source of the extra amount. Yes it could be; but no it doesn’t have to be, because you can change the composition without changing the amount one iota.
George

stumpy
May 12, 2009 6:06 pm

Co2 clearly follows temperature in the earths history, it is clear that solar activity in the past drove SST and co2 was a result of this warming due to outgassing of the sea, which is essentially a large co2 reserve. We know that slightly elevated TSI triggered the Medieval Warm Period, and over the last centuary TSI was at a similar level, hence observation would indicate that the sea should be warming (the amplification required could be due to variation in cloud cover) and therefore co2 should be released. It makes sense and matches observation and as far as I am concerned that is science, explaining observation. Your above empirical model goes a long way towards demonstrating this is still happening and makes perfect sense. There are of course numerous papers that support this, many papers estimate the co2 lifetime at around 7 years, co2 rises at the same rate in each hemisphere etc…
Yet question the co2 dogma and you are called a “denier” or a fool!

J.Hansford
May 12, 2009 6:14 pm

RW…. Anthony already said the point about C13 is redundant because of the problems with the measurement of C13.
Your first point was based solely on the C13 count…. Not very ingenious of you.
Your second point pertains to Ice cores… Once again there are serious problems in the measurement and reliability of CO2 content that remains in ice cores…. The degree of resolution that you are trying to attributing to that remaining CO2 and your use of it as a Temperature Proxy is way beyond what that CO2 measurement can provide, hence your obviously incorrect 14C degree figure and therefor, your point.
Furthermore. If the consensus of the Scientific community can base such high stock in such ambivalent data…. Politics can be the only explanation.

May 12, 2009 6:45 pm

E.M.Smith: The carbonate the fish excrete is high in magnesium ..
Oh my God…! Where does this CO2 nonsense has take us to?
Fish excrete!!! That´s really funny LOL
O’ Evil prophet, all the damage you have made!
You are no longer a serious person, prophet, you are a clown!

tokyoboy
May 12, 2009 6:59 pm

I sincerely hope Dr. Spencer publishes this analysis in a peer-reviewed journal, to make a further strong weapon for pitting ourselves against AGW believers.

Just Want Truth...
May 12, 2009 7:13 pm

C. Boncelet (13:40:15) :
I think you overlooked this line :
“…this admittedly simple model.”

rbateman
May 12, 2009 7:27 pm

CO2 is simply along for the Climate Change ride.
Anthropogenic CO2 is piggybacked onto Natural CO2.
CO2 is a prisoner of the Earth’s Climate, BioChemistry and GeoChemistry.
If you try to bury it, it will bubble back up.
if you don’t bury it, the plants will eat it, including eating your expensive carbon-fiber.
If the plants don’t eat it, the oceans will suck it up and calcify it.
Hey, you’re dragging it around too!
Would you care to supersize your carbonated drink to go along with your carbon-based lifeform burger?

Paul Vaughan
May 12, 2009 7:52 pm

Re: Adolfo Giurfa (18:45:15)
I was just thinking about how hopelessly-conflicted some of these threads get when you succeeded in putting all into some much-needed context — Nice work Adolfo!

DaveE
May 12, 2009 7:53 pm

I admire that Dr. Spencer says, “here is my hypothesis, here are my data and methods, criticise and improve”
I think there are some who could learn from that.
I have a problem with the whole MMCO2 thing however.
If temperatures hadn’t changed, surely CO2 concentration would have changed little in the atmosphere due to the vapour pressure trying to maintain a similar proportional concentration between oceans & atmosphere.
I’ve forgotten so much, so if I’ve got it all wrong, correct me please.
Dave.

Joel Shore
May 12, 2009 8:08 pm

Smokey:

In the first link, Gavin Schmidt has been gelded by Monckton. Did Schmidt actually believe that he could get away with his prevarications online?? Altering graphs produced by someone else is thoroughly dishonest. It is fraud, isn’t it? It appears that people check their ethics at the door when they enter GISS/RealClimate territory.

Oh, give me a break, Smokey! He didn’t “alter the graphs”…He simply simply copied that part of the graph that was relevant, leaving off a little plug along the top border of the graph for Monckton’s website address and he also did not copy Monckton’s caption below the graph. There is no law that says that if you show a graph that someone else has shown you also have to include their caption. (Of course, if Gavin had misrepresented the graph because Monckton had said something very different in the caption than what Gavin implied Monckton was saying, Monckton might have a legitimate complaint. But he doesn’t make any such complaint.)
This is a completely manufactured controversy over nothing and you, a supposedly “skeptic” have fallen for it hook-line-and-sinker! You appear to be about the least skeptical person on the entire planet when it comes to things that agree with your preconceptions.

DaveE
May 12, 2009 8:16 pm

Forgot to mention.
I’ve not forgotten so much that I’ll fall for the ‘saturated oceans’ ploy
DaveE

Joel Shore
May 12, 2009 8:23 pm

As for Spencer’s ideas, as RW has explained, they are in clear contradiction with what we know from the data … which is that the oceans are in net absorbing CO2, not emitting it.
Furthermore, there is no self-consistency here with other data that has been presented … For example, if you want to believe that the oceans have not warmed for the last few years (as a blog post here from several days ago asserted) and you want to believe Spencer’s post here then why have the CO2 levels continued to rise significantly?
Also, if you want to believe that during past glacial – interglacial cycles, the initial CO2 change lagged the initial temperature change by about 800 years (as seems likely), then you have to conclude that even to get the release of CO2 that was seen in those cycles takes a long time…and that effect, as has been noted, is at most about 15ppm rise in CO2 levels for every 1 C of global temperature change, which would imply a fairly small contribution to the >100ppm rise in CO2 levels since the industrial revolution given the temperature rise that we’ve seen.
In fact, the rise in CO2 levels from ~280ppm to ~385ppm is almost entirely due to our emissions. (And, in fact, it would be about twice as large if the oceans and biosphere were not acting as a sink for CO2.) The year-to-year temperature fluctuations due to El Nino and La Nina and the like do cause fluctuations in the rate at which the CO2 level rise but the overall warming of the oceans has not been significant enough to contribute anything but a small fraction of the overage change in CO2 levels seen since the beginning of the industrial revolution (and, even then, it is not the oceans releasing CO2 in net…it is them just absorbing less of what we emit).

Pamela Gray
May 12, 2009 8:30 pm

Joel, the rise in CO2 is a partially modeled data set, not an actual measured data set. The AIRS satellite was supposed to provide the definitive verification of the modeled data set (the seasonally and cyclically adjusted pump amount minus the sink amount). So far, that has not been the case. The actual data sets (modeled versus measured) do not match and have not been compared over a long enough cyclic period to verify the modeled data set even if they did match. You state a case without actual observation-based calibrated measured proof.

Joel Shore
May 12, 2009 8:32 pm

Smokey said:

Altering graphs produced by someone else is thoroughly dishonest. It is fraud, isn’t it?

By the way, when you actually materially alter what someone else had graphed or predicted so that it is no longer an accurate representation of what they predicted but you claim it is their prediction, this is in fact very dishonest. And, that is exactly what Monckton did by making a plot that he claimed to show the IPCC predictions when they were in fact not the IPCC predictions but rather an exaggerated version of them. Strangely, you don’t seem particularly bothered by this!
So, in some sense, you and Monckton are “hoisted by your own petard”!

deadwood
May 12, 2009 8:33 pm

And just how strenuous and vehement the resulting objections are to what I have presented above will be a good indication of how politicized the science of global warming has become.
I have noticed a bit more testiness in the good doctor’s writings of late.

Fuelmaker
May 12, 2009 9:07 pm

Great work Dr. Spencer.
My first serious doubts of AGW arose about ten years ago when a brilliant chemical engineer offhandedly asserted that the theory was obviously wrong because CO2 increases were simply the effect of rising ocean temperature, not the cause.
Joel:
Already mostly answered, but clearly the dominant process in CO2 absorption is physical. There is not nearly enough plankton in most ocean water to metabolize much of the CO2, certainly not on a daily basis. And the temperature is never the limiting factor in photosynthesis. As noted elsewhere here, the tropics are limited by the low concentrations of CO2 in the water. The polar regions are relatively lush and are limited by light, not temperature.
jmrSudbury (12:11:38) :
Beware of complex models. Finding a better fit by adding factors that have no physical basis or are unknown creates all sorts of incorrect conclusions. I wouldn’t trust the precision of a Geology 101 textbook chart very much, it neglects the daily changes. I saw somewhere that ML CO2 varies 10 ppm every day! This is exactly what would be expected from the daily heating of the top surface of the ocean water.
Absorption by vegetation is minor compared to the amount of overall absorption, and it is certainly does not explain where the half of the CO2 from fossil fuel and cement (now 7 GT/yr) has gone or why it varies so much year to year.
Claude Harvey (13:46:47) :
While total ocean heat is essential to an energy balance of the earth. Only the surface exchanges CO2 with the atmosphere, so surface temperature is the only temperature that matters. Also, the fact that it is cooling, very slightly, does not matter, just that it is still above average. Perhaps most importantly, the water upwelling and outgassing may be from hundreds of years ago when the LIA water was much colder and absorbed a whole lot more CO2 just before it went down the conveyor.
Jan Breslow (13:54:29) :
The thermodynamics are actually pretty straightforward. The argument that the concentration is too low is a misunderstanding of the dynamics. Interactions (collisions) between molecules happen at such a fast rate that very small concentrations of IR absorbers (which by definition emit at the same wavelengths) can efficiently transfer heat as if the gas were pure. Every time a photon is emitted by CO2 changing its vibrational state (which cools it), it almost instantaneously is reheated by collision with a N2 or O2 molecule.
Nick Stokes (15:42:19) :
What is your point? Dr. Spencer has simply shown that without any hand waving, it appears that 90% of the increase is due to temperature. In other words, if SST cools to normal, instead of the ocean only absorbing 50% of CO2 emissions, it would absorb 95%.
Nick Stokes (16:17:55) :
Dr. Spencer never said it went into vegetation. Obviously it is coming out of the ocean or being sequested in the deep ocean, actually both happen each year so you could probably improve the model if you separated the average SST into absorbing (cooling) and outgassing (warming).

May 12, 2009 9:10 pm

A fine example of Occam’s razor:
“The simplest explanation for a phenomenon is most likely the correct explanation.”
So what’s simpler – this model or the multitude of climate models used by the IPCC?
No apologies needed Dr. Spencer, I think you are spot on.

May 12, 2009 9:22 pm

pft (17:14:44) :
Einstein never had his great theories subject to peer review until he went up against Bohr . . . His first paper was rejected in 1935.

That must have been one gutsy peer reviewer.

Justin Sane
May 12, 2009 9:22 pm

If AGW is causing CO2 levels to rise at Mauna Lao shouldn’t we expect to see an absolute drop in CO2 levels…
(A) due to the world depression reducing energy consumption significantly the last 9 months.
(B) the Pacific SST dropping for the last few years now
The fact that these these reductions don’t show up in the Mauna Loa CO2 record tells me that the AGW CO2 component is minuscule compared to the non-AGW component of CO2!
So until we can control the non-AGW component of CO2 we’ll just have to live with it whether it causes GW or not.

Carl Wolk
May 12, 2009 10:13 pm

I’m not buying it. There are several inconsistencies.
If Bristlecone Pines can be considered as a proxy for CO2 levels, then the hockey-stick millennial CO2 reconstruction is confirmed. If we reject the reconstruction, we must also reject a key piece of evidence against Michael Mann’s millennial reconstruction.
So, assuming we accept the hockey-shape CO2 reconstruction along with Loehle’s millennial temperature reconstruction, we could not also attribute the modern CO2 increase to increasing oceanic temperatures. If this were the case, why are historic CO2 levels so flat when oceanic temperatures were so variable?

K-Bob
May 12, 2009 10:14 pm

It looks like the Realclimaters have come out of the woodwork and are providing posts that are full of “Schmidt”. Apparently the Gavinizer has run out things to blog about recently. Welcome! I know I prefer to hear healthy debate on these issues. It certainly seems that blogs about the CO2 record seem to cause the greatest debate. The constant increase of CO2 in the atmosphere is the one measurable item that agwers hang their hats on ……… and are clinging to dear life on to.

May 12, 2009 10:23 pm

I agree in your general assertion that not all climate change is man-made but disagree with the reality of the figure you generated at 10%.
I believe the more critical issue is pollution in general and not specifically CO2 emissions but as you pointed out this issue has been politicized by both sides and rational debate is rare.

pft
May 12, 2009 10:53 pm

Mike McMillan (21:22:05) :
“That must have been one gutsy peer reviewer.”
Thats one of the problems with peer review, in most cases the reviewer is anonymous to the writer of the article. On the Quantum mechanics issue, Einstein was thought to be behind the times-too old.

Paul Vaughan
May 12, 2009 10:55 pm

Eric Lightborn (22:23:24)
“I believe the more critical issue is pollution in general and not specifically CO2 emissions but as you pointed out this issue has been politicized by both sides and rational debate is rare.”

Nice to see a truthful, non-partisan comment in the climate debate.

May 12, 2009 11:03 pm

I found this a useful (and for me at least easy to read) piece when looking at CO2 and ocean carbon sinks – it also gives an explanation of Henry’s Law:
http://motls.blogspot.com/2007/11/ocean-carbon-sink-henrys-law.html
I hope it helps with the debate

pft
May 12, 2009 11:17 pm

RW (16:21:26) :
“Conclusion: the oceans cannot be the source of the extra CO2, because if they were, then a) the amount of CO2 in them would be falling, not rising ”
You seem to have bought the IPCC mantra hook line and sinker.
How accurately do we know the CO2 levels in oceans relative to the amount of CO2 it releases into the ocean. What is the uncertainty? Over 200 GtonsC/yr are emitted into the atmosphere via natural processes against only 8 GtC/yr from man. The oceans hold 38,000 GtonsC. A 1% error is 380 GtC, half as much C as in the entire atmosphere. It is not possible to measure changes in ocean C content -3 Gtons C per year (1/13,000 of the oceans C content), with enough accuracy to rule out the oceans role in changes to atmospheric CO2 if the uncertainty in this number is accounted for. Isotope measurement has even greater uncertainty.
ASSUMPTIONS are the heart of IPCC science. For example, assuming the surface layer of the oceans to be in equilibrium leads IPCC to conclude that the measured increase in CO2 is from man’s emissions, without increases due to background effects or warming of the ocean.
The IPCC use the MLO record to represent global CO2, since they calibrate CO2 measurements from other sources to make them all agree, on the assumption that atmospheric CO2 is well-mixed. This contradicts observations of CO2 gradients in latitude and longitude.
IPCC ignores the global flow of CO2 through the atmosphere and across and through the surface layer of the ocean, and then into and out of the Thermohaline Circulation. CO2 is absorbed near 0ºC at the poles, more so in the cold waters of the Antarctic, and returned about one millennium later to the atmosphere at the prevailing tropical temperature. Much of it returned to the atmosphere near the MLO as the MLO sits in the outgassing plume of the Eastern Equatorial Pacific, and CO2 circulates with measurable gradients around the globe.
IPCC does not model the oceans temperature-dependent exchange of about 90 gigatons of carbon per year, even though it dwarfs mans emission of 8 gigatons per year. The outgassing of CO2 is a positive feedback of global warming.
IPCC considers the ocean to absorb mans CO2 at a few gigatons per year, or 50% of its emission rate. It reports natural CO2 outgassed from the ocean as being exchanged with the atmosphere at about 90 gigatons per year
( 100% of the oceans emission rate). IPCC offers no reasonable explanation for the accumulation of mans CO2 (50%), but not natural CO2 (0%).
IPCC’s conclusions are wrong that CO2 is long lived (do the math on the exchanges and the total CO2 in the atmosphere), that it is well mixed, and that it is a forcing, meaning that it is not a feedback.
IPCC assumed cloud cover to be a constant (due to lack of understanding and lack of historical data and accurate historical observations) and not a dynamic feedback and thus ignored what is likely the dominant feedback in the climate system, the negative feedback of cloud albedo.
“If, despite the isotopic evidence, you want to believe that warming has caused the rise in CO2, then you can see from ice cores that the relation between CO2 and temperature that held for 800,000 years or so was that a 1°C rise in temperature corresponded to about an 8ppm rise in CO2.”
The Vostok record, especially the 450,000 year reduction, includes the dominant period of ice ages. The warm periods are the interglacial maxima which are geologically brief, and even instantaneous if you consider that the sampling interval of ice core samples is 1.3 millennia. The present interglacial is within a few degrees of the ceiling interpreted from the previous four maxima. It also suggests that CO2 levels lagged temperature by as much as 800 years.
CO2 levels as measured via chemical analysis with an accuracy of 3% have shown CO2 levels in the 19th century and even in the 1940’s to be at or above levels of CO2 today. The argument to discard these levels is not very convincing if you accept that CO2 is well mixed in the atmosphere. Also, there is some uncertainty in measurements of gaseus inclusions in ice, since the assumption the gas is representative of global atmosphere at the time is not verifiable. Furthermore, an untested assumption that the air in bubbles in ice is 90 to 200 years younger than the ice in which the bubbles are entrapped was made in order to explain the concentrations
of GHG in air bubbles from ice deposited in the 18th and 19th century were similar to those of the present atmosphere. This way 100-200 year old ice is considered to contain air bubbles from 20th century air.
Your own assumption that CO2 and temperature is linear is not true, and even IPCC agrees. An 8 ppm increase at 200 ppm has a much greater effect on temperature than an 8 ppm increase at 360 ppm. The effect of doubling CO2 from 180 to 360 ppm (180 ppm) would be similar to a doubling CO2 from 360 ppm to 720 ppm (360 ppm).

p.g.sharrow "PG"
May 12, 2009 11:54 pm

A cooling hydrosphere will scrub a lot more CO2 out of the atmosphere and result in lowering of the CO2 detected during this period.
Plant life on this planet is an almost steady state carbon cycle and can not cause the changes observed.
Only a planet with huge, dirty (with salts) oceans can have a oxygen enriched atmosphere. CO2 is very easy to scrub with sea water as the CO2 goes into solution with water easily and then combines with light metal ions to make carbonates that percipitate out. Solar radiation in the upper atmosphere breaks down H2O into hydrogen that leaks into space and oxygen that sinks toward the surface.
Global cooling causes lowering of CO2 in the atmosphere.
Global warming causes raising of CO2 in the atmosphere.
CO2 has almost nothing to do with the temperature in the atmosphere, it is an effect not the cause. CO2 is a very poor green house gas and comprises only 1/30 of 1 percent of the atmosphere, and carbon is the most important element for life on this planet. The amount of carbon in circulation is the limiting nutritant in all life on this planet
IT IS THE SUN.
The total output of the sun causes the goldylocks zone to move in or out and the planet to cool or warm depending on the amount of energy it encounters, and the amount of CO2 to lower or raise with the temperature changes.
As a farmer and an inventor and designer of industrial fume scrubbers I believe I have some insight in the creation and maintaince of an atmosphere.

Robin
May 13, 2009 12:24 am

“Global warming theory assumes that the increasing carbon dioxide concentration in the atmosphere comes entirely from anthropogenic sources”
No it doesn’t.
It does assume that some of the increasing carbon dioxide concentration in the atmosphere comes from anthropogenic sources.
It turns out that the oceans and the terrestrial biosphere are net carbon sinks, so the increase is entirely due to anthropogenic sources. But when the oceans get warm enough to outgass, creating a further positive feedback, this will be entirely consistent with global warming theory.

Zer0th
May 13, 2009 12:30 am

Pardon me Roy (is that the cat that chewed your new shoes?)…
Excel spreadsheet, you say… how on earth can that be right? Try again on a super-duper-computer.

Nick Stokes
May 13, 2009 12:40 am

Fuelmaker:
My point is simple. Follow the C. We burn about 10 Gt Carbon per year, and 5-6 new Gt are found in the atmosphere, where we put it. Dr S’s case is that that new C in the air came from the ocean, not from the obvious suspect (the C we put there). OK, then where did ours go?
Dr S didn’t say into vegetation, but JamesG, to whom I was replying, did. The question remains,
Now you seem to be saying that our C goes into the ocean, even while other C is coming from the ocean to replace it. How does that work?
Actually, there is of course some exchange of atoms, so after a time not all the C in the air are the actual atoms that we burnt. But the overall flux of C follows the concentration gradient, from the air into the sea. And we drive that by burning fossil fuels.

Robin
May 13, 2009 12:41 am

“a change in ocean biological activity (or vegetation on land) has a similar signature…so the C13 change is not a unique signature of fossil fuel source.”
However the C14 from biological activity in the ocean is not depleted as it is from a fossil fuel source. So by simply looking at all Carbon Isotopes, and not just C13, it can be shown that the increase in atmospheric carbon dioxide concentration is from the combustion of fossil fuels.
That is without the plethora of information showing that the ocean is taking up carbon dioxide, not releasing it, including studies that track the anthropogenic carbon in the oceans, and the oceanic acidification that is apparent everywhere.

RW
May 13, 2009 1:05 am

pft: the amount of CO2 in the oceans is rising. Therefore, they cannot be the source of the extra CO2 in the atmosphere. Simple as that. 13C measurements both of the current atmosphere and in ice cores are easily accurate enough to show that a dramatic change in the composition of atmospheric CO2 began about 200 years ago, after 800,000 years of almost no variation.
If you want to believe that the oceans are the source of CO2, you have to believe that fossil fuel CO2 is just disappearing, that oceanic CO2 is somehow changing its isotopic composition of its own accord, and that temperatures have risen many degrees since 1800.
“The IPCC use the MLO record to represent global CO2, since they calibrate CO2 measurements from other sources to make them all agree, on the assumption that atmospheric CO2 is well-mixed. This contradicts observations of CO2 gradients in latitude and longitude.”
You had a lot to say about the IPCC there, much of it apparently under the impression that the IPCC itself carries out science. It does not. This one statement in particular is not remotely true. Where on earth did you get this idea from?

Nick Stokes
May 13, 2009 1:13 am

I note the oddity of this post’s assertion that ocean warming is the source of the regular annual rise in CO2, when just a few days ago we had another of many posts asserting that the oceans are cooling.
But I’d also endorse RW’s point above. It’s true that in the past, CO2 levels have risen, when the oceans warmed at the end of an Ice Age. But it is usually about 100 ppm or so, in response to a warming of at least 5C. Here we have a bigger rise since industrialisation, with far less ocean warming, raising CO2 to levels higher than any post-ice age period in the last million years.

Alan Wilkinson
May 13, 2009 1:40 am

There’s too much curvature discrepancy for me. The SST data seems to force the curvature whereas the emissions data is relatively linear as is the Mauna Loa data.
As far as I can see a detrended SST anomaly to provide the short-term variability with the emissions anomaly to provide the long-term trend would provide a much better match.
That wouldn’t support Spencer’s hypothesis. Does anyone have the data?

Malcolm
May 13, 2009 1:43 am

The simple model encapsulates perfectly the notion that the main driver of CO2 changes is natural forcing.
However, it may be more appropiate when adding an anthropogenic element to highlight that this may be largely due to changes in land usage, the impact of aerosols and UHI rather than man-made emissions of CO2; after all we know there is no HOT SPOT in the troposphere, the ghostly AGW radiative driver, to force increases in global temperature.
So to enhance this model we need to breakdown and add-up the natural and anthropogenic temperature elements to improve its ability to do hind-casts.
Say:
delta[CO2]/delta[t] = Natural (Oceans + Atmosphere (clouds + particles + gasses) + Land + Solar) + Anthro(Land usage + Aerosols + UHI + Greenhouse gas emissions)
One aspect of Roy W. Spencer’s model that is worthy of a mention is that it takes the scaremongering out of the equation.

Bill DeMott
May 13, 2009 2:03 am

There is a basic reason why this analysis by Dr. Spencer cannot be published in a scientific journal. As alluded to in at least one posting above, his analysis either assumes that the law of the conservation of matter does not apply or that there are order of magnitude errors in our estimates of the amount of fossil fuels that is burned and/or the change in atmospheric CO2.
What mass balance (conservation of matter) estimates tell us is that the rate of increase in atmospheric CO2 is about half the rate of CO2 release from burning fossil fuels. In other words, over the long term, about 50% of the CO2 released by combustion is absorbed by nature and 50% is accounted for by the increase in atmospheric CO2. Since we have not observed massive increases in terrestrial plant matter (a potential sink on land), scientists have concluded that oceans are a net sink for CO2. That is, over a time scale of years and decades, the oceans are absorbing more CO2 than they release.
If we use Dr. Spencer’s results and assume that the oceans are releasing CO2 and that this release is much greater than fossil fuel burning, we can calculate that the rate of increase in CO2 in the atmosphere should be an order of magnitude higher than observed in the Mano Loa data. If Dr. Spencer were to submit his findings to a scientific journal, the discussion section of his manuscript would need to explain these discrepancies between his results and what is known about world fossil fuel consumption, the resulting CO2 release and the increase in atmospheric CO2. The alternative would be to assume that CO2 from fossil fuel (magically) disappears or that the oceans (magically) absorbs CO2 from fossil fuel while releasing CO2 from other sources.
I assume that Dr. Spencer must be aware of these issues. Dr. Spencer said that the purpose of his analysis was to “stimulate debate.” However, without further explanation it is will just cause confusion among the naive public while being ignored by scientists.

Don Keiller
May 13, 2009 2:28 am

A very interesting model.
I have always been suspicious of the claim that the the delta 13C signal proved that the observed increase in atmospheric CO2 was anthroprogenic.
In fact CO2 relaesed via both fossil fuel combustion and that cycled through biological processes is 13C deleted to a similar amount (approx -30). This is because the photosynthetic enzyme (RUBISCO) discriminates between 12C and 13C so that 13C is depleted in organic (fixed) carbon. This will also happen for fossil fuels because its carbon was “fixed” by the same biological process and both 12C and 13C are stableover geological time.
How can one distinguish between “fossil” and “modern” 13C depletions?
The reference below is useful
ROBINSON, J. J., AND C. M. CAVANAUGH. 1995. Expression of form
I and form II ribulose-1,5-bisphosphate carboxylase/oxygenase
(Rubsico) in chemoautotrophic symbioses: Implications for the
interpretation of stable carbon isotope ratios. Limnol. Oceanogr.
40: 1496–1502.

Gilbert
May 13, 2009 2:36 am

Carl Wolk (22:13:31) :
If Bristlecone Pines can be considered as a proxy for CO2 levels, then the hockey-stick millennial CO2 reconstruction is confirmed. If we reject the reconstruction, we must also reject a key piece of evidence against Michael Mann’s millennial reconstruction.
So, assuming we accept the hockey-shape CO2 reconstruction along with Loehle’s millennial temperature reconstruction, we could not also attribute the modern CO2 increase to increasing oceanic temperatures. If this were the case, why are historic CO2 levels so flat when oceanic temperatures were so variable?
Mann used the Bristlecone Pines as a “temperature” proxy. Temperature measurements from adjacent weather stations demonstrate that this is in error. It has been theorized that the variation in the rings is due to CO2, but there doesn’t appear to be any confirmation studies. Irrelevant anyway.
The historic CO2 reconstruction by Callandar and continued by Keeling is also problematical.
http://www.biokurs.de/treibhaus/180CO2_supp.htm
The hocky-stick is junk science at its best.
Great job by Dr. Spencer!!!
It really did bring out the AGW trolls.

Mike
May 13, 2009 3:33 am

A newcomer to this stuff.
Is there a network of reliable (i.e. properly audited for accuracy) CO2 measuring sites throughout the world? The one that seems to be mentioned always is near an active volcano. Anyone know why? Surely the atmosphere isn’t homogeneous – is it?

JamesG
May 13, 2009 3:59 am

Nick Stokes
You say “We burn nearly 10 Gigatons of C per year. The atmospheric increase is about 5-6 Gt. The rest is thought to go into the ocean.”
But look at this up to date link here:
http://www.guardian.co.uk/environment/2009/feb/18/trees-tropics-climate-change
“Over the world’s tropical forests, this extra “carbon sink” effect adds up to 4.8bn tonnes of CO2 removed each year – close to the total carbon dioxide emissions from the US.”
4.8bn tonnes (or 4.8Gt) is the missing carbon sink that you thought we should assume went into the oceans. So the ocean isn’t a net sink after all. The missing sink was the biosphere all the time. But I now wonder how correct the “10 Gigatons of C per year” calculation is. Bear in mind that the IPCC seems to think that deforestation adds 20% to carbon emissions, which is the opposite of the truth, I don’t give much hope for their other numbers.

Lindsay H
May 13, 2009 4:15 am

Its an amusing debate, some great insights being posted !!
Roy is using similar methods to that of the IPCC where it arrived at its climate sensativity conclusions,
So it should be good enough for Roy to make a claim about co2 being 90% natural, but I bet he does’nt get a mention in the next IPCC report!!
At least we are starting to learn a lot more about the oceans now with the ARGO floats starting to produce usefull information.
If co2 is being absorbed in polar waters and released in tropical waters would there be an observable difference in co2 concentrations between say antartic waters and the tropics. Equally there should be an observable difference between northern hemisphere concentrations where 90% of MMco2 is released and the southern hemisphere.
yet there does not seem to be much difference
http://cdiac.ornl.gov/trends/co2/graphics/JubanyMonthly08.jpg
according to the Italian Jubany monitoring station.
I have some difficulty in understanding how 9-10 gigatons of carbon can be diffused through the atmosphere to the south pole so quickly.

May 13, 2009 4:27 am

Spencer confuses the temperature modulation of the sink speed (3ppm/y/K) with temperarture sensitivity of the equilibrium sink (10 ppm/K)
The units are completely differerent. In electronics Its comparing a temperature resistor with a temperature sensitive battery.

Nick Stokes
May 13, 2009 4:29 am

Mike,
Yes, there are many such sites. Here is the site for the Scripps Institute network – 10 stations across a full range of latitudes. You can look at their results – pretty consistent, and mostly a long way from any volcanoes.

JimB
May 13, 2009 4:38 am

Apologies if this has been posted…I’m travelling and have limited access, so can’t read all the comments to check.
Pretty serious coverage on Hot Air this morning of Senate hearings between Sen. Barrasso and Lisa Jackson regarding the “Smoking Gun OMB memo”, including Youtube clip.
http://hotair.com/archives/2009/05/12/video-epa-memo-says-greenhouse-effect-not-proven/
JimB

Bill D
May 13, 2009 4:42 am

One more time–atmospheric CO2 is going up at 50% of the rate that CO2 is being added from fossil fuel combustion. If we assume that Spencer is right, and fossil fuels only account for 10% of the increase in CO2, then the rate of increase of CO2 in the atmosphere needs to be about 10X higher or we have radically over estimated the amount of fossil fuels that are being consumed. The CO2 needs to be going somewhere. If it is not going into the ocean, where is it going? Forests are being cut down in the tropics and are, perhaps, increasing in the Northern Hemisphere, but that would not account for the massive amount of CO2 that Spencer speculates is being released from the ocean–enough to dwarf the amount of CO2 being released from fossil fuels.
Spencer’s speculations need to take into account the data on CO2 fluxes. Someone should explain the error in my reasoning or you should show why the increase in CO2 in the atmosphere is understimated.

JamesG
May 13, 2009 4:44 am

Also Nick,
“I note the oddity of this post’s assertion that ocean warming is the source of the regular annual rise in CO2, when just a few days ago we had another of many posts asserting that the oceans are cooling.”
This might have been fair comment if you hadn’t already accepted the illogicality of a warming sea being a net carbon sink. It appears that you only look for discrepancies in the skeptic storyline when the IPCC storyline contains far more. The cooling, by the way, is only over the last 6 years.
Furthermore, you might have noticed on this site that the Mauna Loa measurements had been showing suspicious drops until “errors” were corrected. In reality, like the other corrections of outgoing radiation, radiosonde, satellite and ocean buoys, all of which were adjusted towards the hypothesis, the real problem is that too many people are so certain of the hypothesis that they just don’t believe what their instruments are telling them. But the raw, unadjusted data are all consistent. As Douglass noted though, some of the “corrections” make the data inconsistent.

Malcolm
May 13, 2009 4:47 am

There is no such thing as “conservation of matter”.
Modern physics has shown, even in supposedly closed systems such as Earth’s atmosphere, that the only player is energy-mass equivalence. That is why Earth’s radiation budget is so important in determining climate.

Allan M R MacRae
May 13, 2009 4:52 am

BACKGROUND:
My paper was posted Jan.31/08 at
http://icecap.us/index.php/go/joes-blog/carbon_dioxide_in_not_the_primary_cause_of_global_warming_the_future_can_no/
Carbon Dioxide in Not the Primary Cause of Global Warming: The Future Can Not Cause the Past
Despite continuing increases in atmospheric CO2, no significant global warming occurred in the last decade, as confirmed by both Surface Temperature and satellite measurements in the Lower Troposphere. Contrary to IPCC fears of catastrophic anthropogenic global warming, Earth may now be entering another natural cooling trend. Earth Surface Temperature warmed approximately 0.7 degrees Celsius from ~1910 to ~1945, cooled ~0.4 C from ~1945 to ~1975, warmed ~0.6 C from ~1975 to 1997, and has not warmed significantly from 1997 to 2007.
CO2 emissions due to human activity rose gradually from the onset of the Industrial Revolution, reaching ~1 billion tonnes per year (expressed as carbon) by 1945, and then accelerated to ~9 billion tonnes per year by 2007. Since ~1945 when CO2 emissions accelerated, Earth experienced ~22 years of warming, and ~40 years of either cooling or absence of warming.
The IPCC’s position that increased CO2 is the primary cause of global warming is not supported by the temperature data. In fact, strong evidence exists that disproves the IPCC’s scientific position. This UPDATED paper and Excel spreadsheet show that variations in atmospheric CO2 concentration lag (occur after) variations in Earth’s Surface Temperature by ~9 months. The IPCC states that increasing atmospheric CO2 is the primary cause of global warming – in effect, the IPCC states that the future is causing the past. The IPCC’s core scientific conclusion is illogical and false.
There is strong correlation among three parameters: Surface Temperature (“ST”), Lower Troposphere Temperature (“LT”) and the rate of change with time of atmospheric CO2 (“dCO2/dt”). For the time period of this analysis, variations in ST lead (occur before) variations in both LT and dCO2/dt, by ~1 month. The integral of dCO2/dt is the atmospheric concentration of CO2 (“CO2”).
______________________________
See also Roy Spencer’s (U of Alabama, Huntsville) take on this subject at
http://wattsupwiththat.wordpress.com/2008/01/25/double-whammy-friday-roy-spencer-on-how-oceans-are-driving-co2/
and
http://wattsupwiththat.wordpress.com/2008/01/28/spencer-pt2-more-co2-peculiarities-the-c13c12-isotope-ratio/
_______________________________
Earlier work by Kuo et al reached similar conclusions about the lag of CO2 after temperature.
Coherence established between atmospheric carbon dioxide and global temperature
ref. Kuo C, Lindberg C & Thomson DJ, Nature 343, 709 – 714, 22 February 1990.
Summary:
The hypothesis that the increase in atmospheric carbon dioxide is related to observable changes in the climate is tested using modern methods of time-series analysis. The results confirm that average global temperature is increasing, and that temperature and atmospheric carbon dioxide are significantly correlated over the past thirty years. Changes in carbon dioxide content lag those in temperature by five months.
____________________________
Keeling et al reached some similar conclusions in 1995.
Interannual extremes in the rate of rise of atmospheric carbon dioxide since 1980
ref. C. D. Keellng*, T. P. Whorf*, M. Wahlen* & J. van der Plicht†, Nature Vol 375 . 22 June 1995
OBSERVATIONS of atmospheric C02 concentrations at Mauna Loa, Hawaii, and at the South Pole over the past four decades show an approximate proportionality between the rising atmospheric concentrations and industrial C02 emissions1. This proportionality, which is most apparent during the first 20 years of the records, was disturbed in the 1980s by a disproportionately high rate of rise of atmospheric CO2, followed after 1988 by a pronounced slowing down of the growth rate. To probe the causes of these changes, we examine here the changes expected from the variations in the rates of industrial CO2 emissions over this time2, and also from influences of climate such as EI Nino events. We use the 13C/12Cratio of atmospheric CO2 to distinguish the effects of interannual variations in biospheric and oceanic sources and sinks of carbon. We propose that the recent disproportionate rise and fall in CO2 growth rate were caused mainly by interannual variations in global air temperature (which altered both the terrestrial biospheric and the oceanic carbon sinks), and possibly also by precipitation. We suggest that the anomalous climate-induced rise in CO2 was partially masked by a slowing down in the growth rate of fossil-fuel combustion, and that the latter then exaggerated the subsequent climate-induced fall
________________________________
Veizer (GAC 2005) is very interesting, and was quite controversial when published – see also Veizer and Shaviv (2003):
Pages 14-15: The postulated causation sequence is therefore: brighter sun => enhanced thermal flux + solar wind => muted CRF => less low-level clouds => lower albedo => warmer climate.
Pages 21-22: The hydrologic cycle, in turn, provides us with our climate, including its temperature component. On land, sunlight, temperature, and concomitant availability of water are the dominant controls of biological activity and thus of the rate of photosynthesis and respiration. In the oceans, the rise in temperature results in release of CO2 into air. These two processes together increase the flux of CO2 into the atmosphere. If only short time scales are considered, such a sequence of events would be essentially opposite to that of the IPCC scenario, which drives the models from the bottom up, by assuming that CO2 is the principal climate driver and that variations in celestial input are of subordinate or negligible impact….
… The atmosphere today contains ~ 730 PgC (1 PgC = 1015 g of carbon) as CO2 (Fig. 19). Gross primary productivity (GPP) on land, and the complementary respiration flux of opposite sign, each account annually for ~ 120 Pg. The air/sea exchange flux, in part biologically mediated, accounts for an additional ~90 Pg per year. Biological processes are therefore clearly the most important controls of atmospheric CO2 levels, with an equivalent of the entire atmospheric CO2 budget absorbed and released by the biosphere every few years. The terrestrial biosphere thus appears to have been the dominant interactive reservoir, at least on the annual to decadal time scales, with oceans likely taking over on centennial to millennial time scales.
_______________________________
I think there are perhaps four cycles in which CO2 lags T:
1. A cycle of thousands of years, in which CO2 lags T by ~hundreds of years (Vostok ice cores, etc.)
2. A cycle of ~70-90 years (Gleissberg), in which CO2 lags T by ~5-10 years (this is contentious – Ernst Beck’s direct-measurement CO2 data supports, ice core data does not, and there is the important question of how much humanmade CO2 affects this cycle).
3. The cycle I described in my paper of 3-5 years (El Nino/La Nina), in which CO2 lags T by ~9 months.
4. The seasonal “sawtooth” CO2 cycle, which ranges from ~18 ppm in the North to ~1 ppm at the South Pole.
It is clear that T precedes CO2 in cycles 1, 3 and 4. For Cycle 2 we have conflicting and perhaps inadequate data…
Regards, Allan

Allan M R MacRae
May 13, 2009 5:02 am

Bill DeMott (02:03:40) :
There is a basic reason why this analysis by Dr. Spencer cannot be published in a scientific journal. As alluded to in at least one posting above, his analysis either assumes that the law of the conservation of matter does not apply or that there are order of magnitude errors in our estimates of the amount of fossil fuels that is burned and/or the change in atmospheric CO2.
______________________________
Bill – this is Ferdinand Englebeen’s mass balance argument, and it is an interesting one. However, it is possibly wrong. Richard Courtney makes a strong counter-argument. This debate has been gong on between them for some time – you can probably find it by searching ClimateAudit or wattsup.
To put it into perspective, the annual natural variation in CO2 in the North is ~18 ppm, versus the annual average growth of CO2 in the atmosphere of ~2 ppm. Is it possible that nature does not even see man’s contribution?
Note that since 1958 there have been 12-month periods in which CO2 did not increase at all.
Annualized Mauna Loa dCO2/dt has “gone negative” a few times in the past (calculating dCO2/dt from monthly data, by taking CO2MonthX (year n+1) minus CO2MonthX (year n) to minimize the seasonal CO2 “sawtooth”.)
These 12-month periods are (Year-Month ending):
1959-8
1963-9
1964-5
1965-1
1965-5
1965-6
1971-4
1974-6
1974-8
1974-9
Has this not happened recently because of increased humanmade CO2 emissions, or because the world has, until recently, been getting warmer?

Frank K.
May 13, 2009 5:10 am

Bill DeMott (02:03:40) :
“What mass balance (conservation of matter) estimates tell us is that the rate of increase in atmospheric CO2 is about half the rate of CO2 release from burning fossil fuels.”
Where did you get this from? Please post a link or prove this. What are the error bars on this estimate?
By the way, remember from your undergraduate thermodynamics that matter is always conserved in chemical reactions: fuel+air –> products of reaction (which includes CO2, H2O, CO, …). I’m not sure why you would invoke “mass conservation” in this case…

Allan M R MacRae
May 13, 2009 5:14 am

Mike (03:33:53) :
A newcomer to this stuff.
Is there a network of reliable (i.e. properly audited for accuracy) CO2 measuring sites throughout the world?
___________________________________
Have fun Mike.
Regards, Allan
Barrow Alaska
ftp://ftp.cmdl.noaa.gov/ccg/co2/in-situ/brw/brw_01C0_mm.co2
Mauna Loa
ftp://ftp.cmdl.noaa.gov/ccg/co2/in-situ/mlo/mlo_01C0_mm.co2
American Samoa
ftp://ftp.cmdl.noaa.gov/ccg/co2/in-situ/smo/smo_01C0_mm.co2
South Pole
ftp://ftp.cmdl.noaa.gov/ccg/co2/in-situ/spo/spo_01C0_mm.co2

anna v
May 13, 2009 5:22 am
hunter
May 13, 2009 5:49 am

If AGW was real science, this provocative analysis would be welcomed as a great way to test the durability and reliability of AGW.
Instead, since any questioning of AGW gets in the way of powerful financial and political agendas, the discussion will not happen.

May 13, 2009 6:40 am

Those new flash ads bring my browser to a very slow pace, forces me to view them, can’t scroll, can’t switch windows.
They are detestable.

Pamela Gray
May 13, 2009 6:40 am

Earth-bound stations measure outgassing rate in situ. They do not measure resultant global atmospheric CO2 levels. There are no stations that measure sinks (since by definition they want to measure CO2 it would be a waste of money to measure where CO2 is not). All other current statistics about how much CO2 we have in the atmosphere circling and swirling around the globe in globby loopy mists are modeled extrapolations. They are estimates based on for example, gas station receipts and rate of CO2 plant absorption in green houses. In addition, the data has not been collected long enough to cover multidecadal oscillations. AIRS did show that CO2 increased during the time the satellite was measuring this gas, but there were many other variables that increased at the same time. Correlation does not and did not point to cause and affect One thing AIRS has discovered so far is that CO2 is not well mixed in the atmosphere. If the modelers got that one wrong, they should be spending their time performing other tests of falsification instead of standing on their bandwagon picking their noses.

João Oliveira
May 13, 2009 7:10 am

Oh! We are safe! Just tax Mauna Loa!!!

anna v
May 13, 2009 7:27 am

Another relevant thread is the old measurements of CO2 by chemical methods,
http://wattsupwiththat.com/2008/07/25/beck-on-co2-oceans-are-the-dominant-co2-store/
If it is relevant to have temperatures measured at the surface in a grid, it is relevant to measure CO2 also, the way we measure rainfall next to the temperature sensors, since such value is placed on CO2 and its effect on the weather. We would then see, as we see with humidity for example, whether temperatures are higher when CO2 is higher. After all this famous green house effect will be stronger the denser the air ( more molecules ppm).

oms
May 13, 2009 8:44 am

Lindsay H (04:15:10) :

If co2 is being absorbed in polar waters and released in tropical waters would there be an observable difference in co2 concentrations between say antartic waters and the tropics. Equally there should be an observable difference between northern hemisphere concentrations where 90% of MMco2 is released and the southern hemisphere.
….
yet there does not seem to be much difference

I have some difficulty in understanding how 9-10 gigatons of carbon can be diffused through the atmosphere to the south pole so quickly.

Water evaporates from the oceans in some places while rivers feed water into the oceans in other places; yet we observe that sea level is roughly the same everywhere. Hard to believe that all this water can flow around the world so quickly!

AnonyMoose
May 13, 2009 8:49 am

JFA: See if you can add NoScript to your browser, so you can better control what uses your browser. My browser isn’t having difficulty with these ads so I told NoScript to allow everything on this page, but it may help you. (I’m not using NoScript as an ad blocker, just as a security tool. If someone makes their site too hard to navigate without JavaScript then they have to hope that I feel it worth the trouble to allow JS.)

AnonyMoose
May 13, 2009 9:03 am

A graph which shows both temperature and CO2 changes might be helpful. On this, you can see that temperature rises before CO2 rises, and temperature drops before CO2 drops. The analysis by Spencer uses statistics to confirm that this happens, but with the two on the same graph you can see it yourself.
http://www.woodfortrees.org/plot/esrl-co2/isolate:60/mean:12/scale:0.2/plot/hadcrut3vgl/isolate:60/mean:12/from:1958

AnonyMoose
May 13, 2009 9:04 am
Shawn H
May 13, 2009 9:10 am

Perhaps, I am missing something elementary here, but when Spencer uses the 90%-10% split ocean-anthropogenic isn’t he saying that changes in ocean temp *determine* 90% of the variation of in CO2 concentrations, not that 90% of the increased CO2 in the atmosphere came directly from the oceans.
As example, assume animals A and B live on an island. If animal A only eats animal B, then we could say that the population of A is determined by the population of B. Even if we (anthropogenically) parachuted As in by the thousands, they would still be ultimately be determined by the population of B(excess As simply die off).
Cheers, 🙂

Phil.
May 13, 2009 9:25 am

The problem with Spencer’s ‘model’ is that he appears to have made a numerical error!
According to his graphs the present dCO2/dT ≃ 2ppm/yr
His best fit SST term is 4.6*SST, as the present SST anomaly is ~0.21 (from his graph) this gives dCO2/dT (from SST)≃ 0.97 or about half of the total this should mean that about half should be due to Anthro.
However he says that the Anthro term is 0.1*Anthro, since Anthro ≃ 4.2 dCO2/dT (from Anthro)≃ 0.42.
Clearly this is about 20% not 10% but clearly the equation as written doesn’t give the result as given in the graph, to do so would need a value for b of ~0.25 (about 50%). It’s possible that his equation for the Anthro term uses different units than presented in his graph but even so it’s inescapable that that term must yield about half the dCO2/dT not 10%.
So Spencer needs to go back to his spreadsheet and correct his math (and correct the description in his blog).
His model has flaws anyway, for example the SST term should at least have a dependence on [CO2] as well.

Steve Keohane
May 13, 2009 9:33 am

The CO2 cycle is very interesting, thank you Dr. Spenser for your time on this. I am still puzzling over this AIRS video, I’ll post a link below. This video shows that the highest annual concentrations of CO2 occur in April, approx. 40-60 deg. N. In the western hemisphere, it looks like Alaska and Canada are the big emmiters. This makes no sense to me, as things are just beginning to thaw from winter, from non-industrial, low-density habitation areas. The only thing I can imagine is a loss of snow and ice releasing CO2 from under the same. I suppose all the underwater and underground species have been respirating all winter, and that might be released.
http://tinypic.com/m/2l9dea/1%5DView My Video

AnonyMoose
May 13, 2009 9:33 am
lgl
May 13, 2009 9:48 am

AGW-ers:
The ocean is a net sink, no doubt, but the the interesting bit is what would the concentration have been without our emissions? It bothers me a little that the ocean becomes a source during strong ninos, and that there is not a widening gap between CO2 increase and temp graphs as the emissions increase.
counter-AGW-ers:
How do you explain this: http://en.wikipedia.org/wiki/File:Co2-temperature-plot.svg
and this: http://www.ferdinand-engelbeen.be/klimaat/klim_img/law_dome_1000yr.jpg
Temperature should have been 12 deg higher to explain the CO2 level, and you have to choose between a warm MWP or man-made CO2. You can’t have both MWP as warm as today and ocean as the source.

gianfabricio
May 13, 2009 11:23 am

It seems, for me, a even more dangerous scenario that the warmer seas become a CO2 producer. If the hypotheses is correct, the antropogenic production will surely break the natural inbalance.
10% or 20% are very significant figures in a system.

May 13, 2009 12:41 pm

The Monster (16:09:52) :
The long-term record shows that the CO2 changes trail the temperature changes by roughly 800 years. When I have tried to discuss this with True Believers, I’m told that there is a feedback process under which the changes are amplified by the increased release of dissolved CO2 from the higher temperatures, and I’m just too simple to understand the science, which has already dealt with fools like me, so shut up.

There is NO evidence that the rising CO2 causes any feedback. This is AGW bluster at its worst. This contains what you need to rebut it.
PaulHClark (23:03:16) : I found this a useful (and for me at least easy to read) piece when looking at CO2 and ocean carbon sinks – it also gives an explanation of Henry’s Law [Lubos Motl URL]
I’m very sorry to say Lubos has got it wrong AFAICT. He usually has my respect. The oceans hold a huge amount of CO2 and we therefore have every reason to suspect that atmospheric CO2 levels are very sensitive to small SST changes. Have a look at my page on CO2 and decide for yourself. The whole CO2 issue is an ever-changing mobile flux that is heavily dependent on the oceans and the biosphere and far less dependent on us to date. It is a strange coincidence that the rising CO2 rate is about half the emissions rate – but it is a coincidence that has mesmerised the whole of AGW.
Here are some useful figures (gigatons of carbon) to help get things in proportion:-
Atmosphere 700… Ocean “surface” 700… Biosphere 70… Total ocean 40,000… Fossil fuel reserves 12,000…
Annual natural flux 230… Annual human CO2 emissions 6…

theatreofinconveniences
May 13, 2009 1:57 pm

The simple question is, would it help if there were no anthropogenic sources of CO2?
Another question would be – Will the adverse effects of global warming and climate change stop manifesting themselves if Al Gore went ahead and said there is no need to worry about fossil fuels since they have insignificant contribution to global CO2 levels?

Paul Vaughan
May 13, 2009 2:17 pm

JFA in Montreal (06:40:33)
“Those new flash ads bring my browser to a very slow pace, forces me to view them, can’t scroll, can’t switch windows.
They are detestable.”

I found what *seems to be a solution to multi-minute hangs: I discovered that I can relieve the (seemingly-permanent-at-times) hangs by grabbing-&-dragging the scroll-knob (right margin). (Attempting to scroll via mouse-wheel seems to fail.)

Gerald Machnee
May 13, 2009 2:45 pm

RE: Phil. (09:25:32) :
**His model has flaws anyway, for example the SST term should at least have a dependence on [CO2] as well.**
Where does that come from? An AGW model?
SST does not depend on CO2. CO2 is more likely to be related to an increase in SST.

Carl Wolk
May 13, 2009 2:48 pm

Gilbert:
I am not an “AGW troll.” I find it disconcerting that if I doubt one argument proposed by one skeptical scientist, I am assumed to agree with AGW theory. I am what you would call a “skeptic,” but I think he is wrong on this issue. “Skeptics” have begun (especially on this blog) to leap onto any theory that would contradict AGW theory; this is not skepticism.
For Spencer’s proposed mechanism to make sense, we have to reject ice-core data in light of the medieval warm period. Instead we turn to Beck’s reconstruction. I find it odd that the chemical measurements show such rapid, and changing CO2 content, yet as soon as Mauna Loa begins to take measurements the slope turns very flat and does not change from a general linear rise. Are there any modern CO2 chemical measurements that we could use to compare against Mauna Loa data?
So really, this debate is focused on the wrong place. Spencer’s proposition is impossible if we accept ice-core data. It might be correct if we accept Beck’s chemical data.

JamesG
May 13, 2009 3:41 pm

The certainty that some people have here about the mass balance is belied by the sheer guesswork involved. When you look at the calculations you realize you can make up any story you like because you can fit a bus in the error bars.
I’ve seen estimates of anthropogenic emissions from 7 Gt to 27 Gt all from seemingly reputable sources. Whether 7 or 27 though the assumed lost-in-sink percentage of that is always assumed to be 50%. If that seems ludicrous it’s because it is. The 50% is clearly just a dogma that doesn’t depend on any actual numbers.
It’s widely acknowledged though that our emissions are roughly about 3% of the total CO2 flux. New scientist shows a graphic of the carbon exchange:
http://www.newscientist.com/data/images/ns/cms/dn11638/dn11638-4_738.jpg
ie 440 Gt going in and out from land and 260 Gt going in and out from sea. Humans cause 26 Gt from fossil fuels and 6 Gt from land use. Well that +6 is wrong for a start because the earth is known to be greening. Likely it should be -6. And of course the fossil fuel part may be as low as 7 Gt. But anyway, just look at the sheer size of the fluxes compared to our tiny contribution.
So in order for for us to affect things, bearing in mind the huge likelihood of greater than 3% error in any of the flux numbers, we have to prove that the increase in the air is from man due to us disturbing the balance of nature. I find the argument surreal at this point: The system can’t cope with an extra 3%? There is no real evidence of this except that CO2 is increasing and so are temperatures. Which is, of course a silly circular argument because the temperature might cause the CO2 rise. The other evidence rests on iffy isotope ratio ideas. The carbon 14 argument was abandoned even by the IPCC and the carbon 13 argument was exposed as wrong-headed by Spencer. We also have the ice-cores telling us one thing about past CO2 but other geological evidence (ignored by the IPCC) telling us the opposite.
So all that are left are those iffy hand calculations with massive error bars and the a priori assumption that the increase is not natural. So no, it’s not obvious that the ocean is a net sink. Not unless you are either totally innumerate or just plain want to believe it.

Frank K.
May 13, 2009 4:50 pm

JamesG (15:41:54) :
Thanks JamesG for that great explanation. I suspected that the 50% argument was a wild guess and likely bogus.

kuhnkat
May 13, 2009 5:22 pm

Nick Stokes,
One of many articles and studies acknowledging the huge increase in biomass over the last 20 years:
http://earthobservatory.nasa.gov/Features/GlobalGarden/
You also might want to spend a little time on the CO2Science site for all the PEER REVIEWED papers that you appear to be missing.
RW,
and exactly who was measuring the ratio of C13 in the oceans about 200 years ago??
HAHAHAHAHAHAHAHAHA
Oh yeah, paleo studies. Done by Mann and friends??
Let’s take this seriously for a moment. 200 years ago there is a SHARP DROP. Now, would you please explain to all of us, who was burning all the coal and petroleum that you would have us believe caused this sharp drop 200 years ago??
Please try to open your mind a tad and read some of the better submissions above that realistically bring into question the C13/Anthropogenic connection.

kuhnkat
May 13, 2009 5:45 pm

Carl Wolk,
you might want to look up the current CO2 data in cities. There are huge diurnal variations that exceed 500ppm peak in many of them.
What needs to be understood is that CO2 is NOT well mixed in the atmosphere. Around sources the measurements will regularly be much higher than the isolated measuring stations now in vogue. In fact, ALL those stations have strict standards that throw out measurements that are outside certain standards. This is to prevent mistakes in procedure or things like Volcano eruptions (see Mauna Loa), or odd wind patterns that bring CO2 saturated air from an inconvenient location (downtown anywhere) or a warming body of upwelling water or…, to the pristine measurement site.
Basically, those chemical measurements taken back then could not be duplicated now because the current levels would probably be much higher!!!!!! Why the warmers hyperventilate over them is pretty pitiful. It also shows how ugly they are when you consider that they are maligning some great scientists with their claims of poor practices and shoddy work just to try and shore up THEIR shoddy, misleading work.
Now, they can argue forever whether it is anthropogenic or natural, BUT, there is a heck of a lot more CO2 flux than is presented in their toy models.

kuhnkat
May 13, 2009 5:49 pm

Phil,
why should SST have a dependence on CO2??
Are you heating or cooling large amounts of CO2 and pumping it into the bottom of the ocean and letting it percolate up????

a jones
May 13, 2009 6:38 pm

Amazing what odd little nuggets one finds.
In the paper above “earth observatory” the authors report that plant growth in the tropics was stimulated by “less cloud over and higher solar radiation”.
If so I imagine the oceans in the tropics also warmed from the same effect.
Kindest Regards

Joel Shore
May 13, 2009 7:25 pm

JamesG says:

I’ve seen estimates of anthropogenic emissions from 7 Gt to 27 Gt all from seemingly reputable sources. Whether 7 or 27 though the assumed lost-in-sink percentage of that is always assumed to be 50%. If that seems ludicrous it’s because it is. The 50% is clearly just a dogma that doesn’t depend on any actual numbers.

Oh dear…What you probably have seen are estimates of 7 Gt carbon and estimates of 27 Gt of CO2. To convert between one and the other you have to multiply or divide by (44/12) since C has an atomic weight of 12 and CO2 has a molecular weight of 44. Your statement serves as a good example of how people here lead themselves astray and then mistakenly assume it is the scientists rather than themselves that are being dogmatic or incorrect.

So in order for for us to affect things, bearing in mind the huge likelihood of greater than 3% error in any of the flux numbers, we have to prove that the increase in the air is from man due to us disturbing the balance of nature. I find the argument surreal at this point: The system can’t cope with an extra 3%?

The point is that there is a lot of carbon that cycles between the oceans, atmosphere, and biosphere…But, as the ice core record shows, these fluxes roughly balance. (When they don’t, as when we are going in and out of glacial cycles, the rate of change of CO2 levels in the atmosphere is still much smaller than it has been over the last century.) We are rapidly liberating a store of carbon that has been locked away from the atmosphere, oceans, and biosphere for tens to hundreds of millions of years.

So no, it’s not obvious that the ocean is a net sink. Not unless you are either totally innumerate or just plain want to believe it.

In addition to the accounting (which is not as iffy as you thought once you realize the distinction between Gt of C and Gt of CO2), there is direct evidence that the ocean waters are becoming more saturated with carbonic acid as CO2 dissolves into them.

Joel Shore
May 13, 2009 7:31 pm

By the way, here is a summary of what is generally understood by scientists in regards to CO2, along with references into the literature: http://www.radix.net/~bobg/faqs/scq.CO2rise.html

Paul Vaughan
May 13, 2009 7:34 pm

kuhnkat (17:45:30) “It also shows how ugly they are when you consider that they are maligning some great scientists with their claims of poor practices and shoddy work just to try and shore up THEIR shoddy, misleading work.
I have seen many examples of such administrative practice first-hand. It is not science – it is politicsdisgraceful politics.

Nick Stokes
May 13, 2009 7:38 pm

JamesG,
There’s not much doubt about the amount of C burnt. carbon fuels are costly, and tracked by an army of accountants. The figures come from bodies like the EIA. You’re 27Gt is almost certainly data on CO2 produced, not C burnt. 7.5Gt C makes 27.5 Gt CO2. Burning is increasing rapidly, because of industrialisation in Asia. The latest EIA figure is 29195 Gt CO2 for 2006 (=7962 Gt C). There’s about another 5% from cement production.
The loss isn’t “assumed to be 50%”. Again, the Gt CO2 in the atmosphere is quite well known, so no assumption is needed. The IPCC AR4 has a FAQ on this topic (Chap 7), and they say 55%.

Mike Bryant
May 13, 2009 8:19 pm

Joel Shore,
I would like to congratulate you on your new calm and straightforward comments. I am sure I am not the only one to notice the “new” you. There is still no convincing evidence of the CO2=heating hypothesis, of course, let alone any reason to believe that there are catastrophes related to it. You are starting to sound like a scientist instead of an agenda-driven firebrand.
Thanks for your effort, it is appreciated,
Mike

Gerald Machnee
May 13, 2009 8:29 pm

RE: Joel Shore (19:31:24) :
**By the way, here is a summary of what is generally understood by scientists in regards to CO2, along with references into the literature: http://www.radix.net/~bobg/faqs/scq.CO2rise.html**
The person who summarized that is not an expert in the field by his own admission.
He essentially just noted the AGW point of view.
I would give more credence to Spencer, Pielke, CO2Science, etc.

Phil.
May 13, 2009 8:44 pm

Gerald Machnee (14:45:04) :
RE: Phil. (09:25:32) :
**His model has flaws anyway, for example the SST term should at least have a dependence on [CO2] as well.**
Where does that come from? An AGW model?

Routine Physical chemistry, related to Henry’s Law, rather than:
dCO2/dT ≃ 4.6*SST+0.1*Anthro (which in the form Spencer used it has arithmetic errors anyway) the first term should be of the form a(SST,[CO2]).

anna v
May 13, 2009 8:59 pm

oms (08:44:10) :
yet we observe that sea level is roughly the same everywhere. Hard to believe that all this water can flow around the world so quickly!
That is gravity that is keeping the levels constant. The consistency moves very very slowly and stays locally. Floods in Italy do not muddy the seas in Greece. There is no universal force, as gravity for water, around the globe for CO2. And AIRS data show that there is no direct mixing
http://airs.jpl.nasa.gov/story_archive/CO2_Increase_Sep2002-Jul2008/

anna v
May 13, 2009 9:18 pm

Steve Keohane (09:33:00) :
In the western hemisphere, it looks like Alaska and Canada are the big emmiters. This makes no sense to me, as things are just beginning to thaw from winter, from non-industrial, low-density habitation areas. The only thing I can imagine is a loss of snow and ice releasing CO2 from under the same. I suppose all the underwater and underground species have been respirating all winter, and that might be released.
Part of it is the surface heating of the waters too, snow and ice will have CO2 absorbed in them too released with the melt suddenly. Considering that ground measurements give much larger CO2 concentrations, that will be a factor too. Is there a biologist in the house? Do burgeoning flora emmitt more CO2?

Bill D
May 13, 2009 10:05 pm

All of the tundra ponds are emitters of CO2 and methane, because of the amount of organic matter that that drains into them. However, I doubt that the amount would be comparable to human sources in inhabited regions and the peaks should be in summer, when temperatures are higher. Unlike oceans, inorganic carbon in lakes is mainly controlled by dissolved organic carbon from the water shed rather than atmospheric sources.
Concerning human CO2 emissions, the numbers are precise, within plus or minus 2% of the total. A book on “The carbon cycle” is available on Google books. You can see detailed listings of all of the various sources. I don’t have the author handy, but found it by searching (in google scholar) under “ratio of fossil fuel consumption and atmospheric CO2 increase”

oms
May 13, 2009 10:31 pm

anna v (20:59:58) :

Floods in Italy do not muddy the seas in Greece. There is no universal force, as gravity for water, around the globe for CO2. And AIRS data show that there is no direct mixing.

Then it’s interesting that the equalizing cause for sea level (pressure gradient) also affects gases in a mixture. Without the floods in Italy muddying the seas in Greece…

Mike Bryant
May 13, 2009 10:51 pm

The role assigned to CO2 in the CAGW morality play is laughable. It’s as if the Royal Shakespeare Company had asked Adam Sandler to play the title role in King Lear.

Mike Bryant
May 14, 2009 12:26 am

anna v,
That AIRS animation makes it absolutely inconceivable that man can have anything to do with such massive swings.
http://airs.jpl.nasa.gov/story_archive/CO2_Increase_Sep2002-Jul2008/
Mike

Paul Vaughan
May 14, 2009 2:34 am

anna v (21:18:25) “Considering that ground measurements give much larger CO2 concentrations, that will be a factor too. Is there a biologist in the house? Do burgeoning flora emmitt more CO2?”
That would be soil organisms. I can’t give you any numbers (without researching them) – but I won’t hesitate to say the respiration peak should occur in summer (at high latitudes).

RW
May 14, 2009 2:35 am

“There is still no convincing evidence of the CO2=heating hypothesis, of course”
..except that the very existence of a greenhouse effect proves it beyond doubt, of course.
Robert Kral, from ages ago:
“the world has been both much warmer and much colder than it is now, before the emergence of humans and after. Since everything that we’re observing now is clearly within the bounds of natural variability, why is it necessary to invoke human causation? In fact, why is it not specious to do so before carefully eliminating all possible natural causes?”
This argument is a little bit like saying that if you were found dead in the middle of a road, and a car with a dent in its bonnet is stopped right by you, but you were less than 122 years old at the time of your death, then it would be reasonable to say that because your age was not outside the bounds of natural variability, there was no need to invoke human causation in your death. CO2 is a greenhouse gas, and its concentrations are rising sharply. Even if temperatures on Earth had varied in the past between absolute zero and 50,000K, that would not change these two facts, and it would not change the fact that increasing greenhouse gas concentrations are currently driving up temperatures.
For those who still seek to argue that all the CO2 is from ‘natural’ sources: please consider Occam’s Razor. We have these simple observations: 1. fossil fuel burning has released a vast amount of CO2 in the atmosphere; 2. fossil fuel CO2 has a lower 13C content than both atmospheric and oceanic CO2; 3. atmospheric CO2 concentrations began to rise when fossil fuel burning began to rise; 4. the 13C content of atmospheric CO2 began to drop when fossil fuel burning began to rise. You can see some nice figures here
No reasonable person looks at these facts and concludes anything other than fossil fuel burning is the cause of the rise in atmospheric CO2. You are instead trying to claim that all the fossil fuel CO2 is simply vanishing, that oceanic CO2 is somehow changing its isotopic composition of its own volition, and that this miraculously changed CO2 began appearing in the atmosphere just when fossil fuel use began to rise, but was completely unrelated.
Given that oceanic CO2 content is rising, you also need to believe in another, as yet unspecified source which is more than replenishing oceanic CO2 losses to the atmosphere, but is again somehow completely unrelated to the vast amount of CO2 released by fossil fuel burning. These arguments are physically impossible. You hardly need to be a scientist to see that. I have no idea what Roy Spencer’s motivation is, for publicising such nonsense, but the advancement of science is definitely not what he’s trying for here.

Nick Stokes
May 14, 2009 2:59 am

Kuhnkat,
OK, I did look on CO2Science. They featured this paper on global terrestrial carbon uptake in the 80’s and 90’s. They cited NEP values, which measure addition to global biomass, but not allowing for loss by burning or agricultural use. In other words an upper bound on C accumulation in the biosphere.
They gave the 90’s NEP figure at 1.36 Gt/yr. A rise from the 80’s, but as said, it is reduced by fire etc. Still, it isn’t going to absorb our 8+ Gt/yr fossil fuel emission.

anna v
May 14, 2009 5:28 am

RW (02:35:27) :
The isotope card has been burned since it was noted that part of the algae/ plankton prefer the C13 way. There is lots and lots of them in the oceans.
Sure people emit a puny 6 Gigatones to the atmospheres 750Gigatones, not even 1%, and within the noise of the system. http://en.wikipedia.org/wiki/Carbon_cycle
With the increase in temperatures and subsequent increase in CO2 the flora has flourished up by 30%. Notice that vegetation give 650 gigatons to the atmosphere, that is it is circulating in its respiration expiration and decay 100 times more than our puny 6 gigatons. It could clean its teeth with that. 30% increase would mean that the flora generates almost 200 gigatons, to our puny burning 6 extra. still noise to the signal.
So it is not that there is no effect from anthropogenic CO2. It is that it is too small and flooded by the flora and the ocean exhalations.
Unless you are not able to understand big numbers, ( one, two thee, many) this should be evident.

John
May 14, 2009 5:29 am

Why CO2 falls after the beginning of an ice age, rises after initial warming coming out of an ice age
A Wally Broecker article, based upon analysis of sediment cores, found that after the earth cools substantially after the initiation of an ice age, the drier and windier conditions off Patagonia cause about 50 times more dust from the southernmost part of South America to blow into the southern ocean. The dust has minute amounts of iron, which in fact does fertilize the ocean, enough to draw down tens of PPM of CO2, but over a time scale of several thousand years.
While some of the increase in CO2 drawdown ends up on the seabed in the form of diatomic/planktonic detritus, most of it occurs as an increase of dissolved CO2 in the ocean itself (the detritus gets processed by bacteria before it sinks very far, releasing the CO2 back into the water).
The process of CO2 drawdown takes so long because it requires that high CO2 waters only a few hundred meters or less from the surface near Antarctica are exported to deep waters, in a vertical “elevator” process that occurs in that part of the world. From these southern ocean deep waters, deep currents gradually redistribute the CO2-rich waters around the world.
Whenever this CO2 rich water gets to surface, it releases CO2 back to the atmosphere. So in order to get the full increase in oceanic CO2, the entire ocean must be saturated with the excess CO2. At this point, there is a steady state balance of input and output of CO2.
When the earth finally warms again, the input fertilization process slows back to the “normal” we see today — e.g., little natural wind blown iron fertilization off Patagonia — but the output process continues for a few thousand years until the tens of PPM of CO2 are all back in the atmosphere.
Fertilization does work, but it took thousands of years. To attempt to get something like 40 PPM sequestered in this manner, in say 100 years, would likely create hypoxic and anoxic dead zones because it would take place in one place, the southern ocean, where surface waters are exported to depth and there is a relative lack of iron. So I’ve become reluctantly convinced that iron fertilization isn’t the solution to too much atmospheric CO2 (yes, I agree, we don’t know how much is too much; if I didn’t agree, I wouldn’t be reading this blog). Whether we should fertilize the ocean is not the main topic of this post, however.
Thus one point of this post is to point out why CO2 levels rose and fell with the ice ages, and why there was a substantial lag of about 800 years before the CO2 dropped enough for us to measure it (going into the ice age) and rose by that much (going out). This doesn’t negate a cooling and warming role, as a positive feedback in each direction, but I don’t think Broecker or anyone else has tried to model how much of an additional feedback the changes in CO2 levels, subsequent to going into or out of an ice age, actually occurred.
The other point is to say that on the short time scales of decades that Roy Spencer analyzes, more CO2 gets into the ocean because with more in the atmosphere, the partial pressures of CO2 in air vs. CO2 in water cause something like half of the CO2 in the air to be dissolved in the oceans, even with slight increases in SSTs. I’m a bit surprised Spencer didn’t know that, he clearly knows his own field very well.

May 14, 2009 5:36 am

Mike Bryant (00:26:42) : That AIRS animation makes it absolutely inconceivable that man can have anything to do with such massive swings.
http://airs.jpl.nasa.gov/story_archive/CO2_Increase_Sep2002-Jul2008/

I reached a different conclusion. I thought it looked quite possible that we have a SYMBIOTIC process here: NH cities produce CO2 in the winter months, alongside the CO2 from winter forests, which is then guzzled by happy taiga and other temperate vegetation in the summer months. We know the biosphere has increased, and that the Sahel is greener.
However, Spencer’s evidence of a tiny proportion of human contribution to CO2 increase is important. What most people forget is the sheer size of the annual flux, the huge quantity of CO2 in the oceans, the temperature-dependent solubility of CO2, and the biosphere’s ability to grow, to take in any extra CO2, both as plants and as marine shells.

Gilbert
May 14, 2009 6:07 am

Carl Wolk (14:48:56) :
I am not an “AGW troll.” I find it disconcerting that if I doubt one argument proposed by one skeptical scientist, I am assumed to agree with AGW theory. I am what you would call a “skeptic,” but I think he is wrong on this issue. “Skeptics” have begun (especially on this blog) to leap onto any theory that would contradict AGW theory; this is not skepticism.
——
First, my apology. My troll statement was not aimed at you, but to all the warmers generally on this thread.
As for the alternate theories on this blog, I agree. It isn’t necessary to show what the source of climate change is. Only necessary to demonstrate that AGW isn’t it.
——
For Spencer’s proposed mechanism to make sense, we have to reject ice-core data in light of the medieval warm period. Instead we turn to Beck’s reconstruction. I find it odd that the chemical measurements show such rapid, and changing CO2 content, yet as soon as Mauna Loa begins to take measurements the slope turns very flat and does not change from a general linear rise. Are there any modern CO2 chemical measurements that we could use to compare against Mauna Loa data?
So really, this debate is focused on the wrong place. Spencer’s proposition is impossible if we accept ice-core data. It might be correct if we accept Beck’s chemical data.
——
I tend to reject the ice core data used by Callandar because it does not appear to be representative. There are also legitimate questions about the accuracy of ice cores. I think that the historical data from chemical analysis may only be useful to demonstrate the deficiencies in Callandars CO2 construction. At best we may only be able to say that we don’t know what the historical values really were, and consequently the claims that the current levels are higher than ever, are without merit. And I’m also suspicious of the Mauna Loa data.
As long as the government money is going into AGW research, it’s going to be difficult to do the kind if studies that will show what the reality is.
Also, while it appears counter intuitive, it is entirely possible for the co2 content in the atmosphere and the c02 content in the oceans to rise at the same time. The oceans are constantly absorbing and releasing CO2, and only a minor change in ocean temperature would change the balance.
While Dr. Spencers analysis may be a bit iffy, it did generate a lot of questions and it may actually be correct in a somewhat simplistic manner
I would suspect that we agree more than we disagree.

Phil.
May 14, 2009 6:43 am

anna v (05:28:07) :
RW (02:35:27) :
The isotope card has been burned since it was noted that part of the algae/ plankton prefer the C13 way. There is lots and lots of them in the oceans.
Sure people emit a puny 6 Gigatones to the atmospheres 750Gigatones, not even 1%, and within the noise of the system. http://en.wikipedia.org/wiki/Carbon_cycle
With the increase in temperatures and subsequent increase in CO2 the flora has flourished up by 30%. Notice that vegetation give 650 gigatons to the atmosphere, that is it is circulating in its respiration expiration and decay 100 times more than our puny 6 gigatons. It could clean its teeth with that. 30% increase would mean that the flora generates almost 200 gigatons, to our puny burning 6 extra. still noise to the signal.

Anna, you have misread the diagram, the black numbers are the amount stored and the purple numbers are the fluxes.

Pamela Gray
May 14, 2009 6:44 am

The warming trend (or any temperature trend for that matter) is a statistical number, not the data. For every spike and fall in the noise, weather events coincide that are directly tied to natural processes. Give me any spike or fall and I will send you scurrying to archived weather reports (jet stream data, oceanic conditions, weather fronts, etc) where you will find the explanation quite clear and easy to understand. If this can be done for EVERY spike and fall (which some say is weather), and it can, how can you jump to the conclusion that the statistical trend line (which some say is climate) that is created from these weather events is caused by something else (CO2, Sun, the goddess in the cave)? That is a leap even Evil Knievel would not have attempted. Folks, the weather noise creates the very artificial trend line, therefore the trend line is not climate. Furthermore, the trend line is too removed from cause and affect to be useful in ascribing any cause to such a noisy data set.

Phil.
May 14, 2009 6:48 am

[Phil. I’m growing weary of your attacking Dr. Spencer and others from the shadows without having the courage (as he does) to put your name to your words. So until you do, no more posts (about Spencer) from you. You are a scholar, and this sort of behaviour wouldn’t be tolerated in your own institution, in a professional critique of the work of other colleagues, or even in letters to administration, so I’m not going to give you a free ride to critique anonymously here. Science and scholars should be held to a higher standard. You say “Dr. Spencer’s math is wrong”, prove it and put your name and institution on it. I’ll even offer a guest post position for you. Otherwise don’t post again. – Anthony Watts]
UPDATE: Phil, your claim of anonymous review doesn’t hold water, because it is not written in private to a reviewer, your comments are published for the world to see. In peer review, the public does not see the critiques, only the reviewer. The decision and offer stands. You are welcome to have your own guest post here at WUWT, refuting Dr. Spencer, provided you have the courage to put your name on your work. – Anthony Watts

kim
May 14, 2009 8:35 am

While I don’t like Phil. worth a darn(he’s called me bad names) I think he should be allowed to continue to post. My objection to him is along the lines of his telling the truth, and nothing but the truth, but not the whole truth. You can rely on the specifics of what he says, but you cannot fundamentally rely on what he says because he will simply refrain from mentioning anything that goes against his point of view. Thus, he is fundamentally untrustworthy. I understand your point, Anthony, and agree with it, but still find much value in Phil.’s input. Glad I don’t have to decide.
============================================
REPLY: He’s welcome to post on other threads. My issue applies only to his claims about Spencer. We are so often beat up over the issue of peer review, and yet here is somebody who is part of a major academic institute of higher learning, fully willing to circumvent the traditional peer review process, and to do so publicly rather than privately, while claiming falsification from a position of anonymity. That won’t wash here. – Anthony

kim
May 14, 2009 8:36 am

Ah, Anthony, I see wisdom in your 8:21:30 post.
===============================

May 14, 2009 8:41 am

wattsupwiththat (08:21:30) :
It would be no different if you wrote a rebuttal letter to a journal. If you didn’t sign your name to it, they would not publish it. – Anthony Watts
I have a comment about the review process. I agree that the reviewer [if she/he so chooses] should be anonymous during the review process. After [and if] a paper has been accepted, the reviewers and their review should be made public. Some journals at least thank the reviewer publicly. But the whole exchange should be public [in the electronic supplement that almost all journals provide], so that one can see how thorough the review was, and by whom. This also helps the reviewer to get credit for the [unpaid] service provided, which can be a substantial amount of work in itself.

gary gulrud
May 14, 2009 9:06 am

“My objection to him is along the lines of his telling the truth, and nothing but the truth, but not the whole truth.”
I wonder if AGW apologists don’t consider the Good greater than one good, the Truth. Its like a difference between believing mankind basically good and ameliorable versus iniquitous and finite.

anna v
May 14, 2009 9:09 am

Phil. (06:43:38) :

Anna, you have misread the diagram, the black numbers are the amount stored and the purple numbers are the fluxes.

Possible.
Well,vegetation, 610, atmosphere 720 and anthropogenic 5.5 are all in black on my laptop screen. Are you saying that the 5.5 ( if it is purple) should be compared to what? 60 +60 +92? Not as small a percentage but still quite small.

Gerald Machnee
May 14, 2009 9:38 am

RE: RW (02:35:27) :
***“There is still no convincing evidence of the CO2=heating hypothesis, of course”
..except that the very existence of a greenhouse effect proves it beyond doubt, of course.***
The greenhouse effect keeps our temperature fairly steady or balanced on the earth. The question is: “does adding CO2 and other greenhouse gases change the temperature and by how much?”
1) NOBODY has to date MEASURED the amount or percentage that CO2 increases or has increased the global temperature. The “beyond all doubt” is a saying, but not a proof. One way to look at it is that if CO2 is the main factor in temperature increase, then the global temperature must increase EVERY year with the increase varying in amount due to natural causes. The temperature has not increased every year, so CO2 cannot be the dominant factor. The second question that should be addressed is Steve McIntyre’s one that IPCC has not addressed: 2) Where is the detailed engineering quality calculation of the effect that doubling the CO2 will have on the earth’s temperature?
You can check David Archibald’s paper as one reference of how the effect of greenhouse gasses decrease logarithmically.
***For those who still seek to argue that all the CO2 is from ‘natural’ sources: please consider Occam’s Razor. ***
The “skeptic” scientists are not saying that ALL the CO2 is from natural sources. Please check this.
***We have these simple observations: 1. fossil fuel burning has released a vast amount of CO2 in the atmosphere; 2. fossil fuel CO2 has a lower 13C content than both atmospheric and oceanic CO2; 3. atmospheric CO2 concentrations began to rise when fossil fuel burning began to rise; 4. the 13C content of atmospheric CO2 began to drop when fossil fuel burning began to rise. You can see some nice figures here***
Check the latest studies – The 13C is not only from fossil fuels. The increase in CO2 in the atmosphere is not all due to fossil fuels
***No reasonable person looks at these facts and concludes anything other than fossil fuel burning is the cause of the rise in atmospheric CO2. ***
What you are saying is that any reasonable person will conclude that fossil fuel burning is the only cause of the increase in CO2.
That is getting to be a tired old AGW alarmist line. Was fossil fuel burning the cause of the 1000+ ppm ages ago as well?
***I have no idea what Roy Spencer’s motivation is, for publicising such nonsense, but the advancement of science is definitely not what he’s trying for here.***
Sorry, but your criticisms are unfounded. Dr. Spencer has done a lot of good work.
He also stated that this paper was for discussion purposes.

bill
May 14, 2009 9:51 am

On another thread
http://wattsupwiththat.com/2009/05/10/solar-scientist-ken-tapping-no-sign-of-the-new-cycle-yet/#more-7754
anna v has been trying to convince me that the rise in co2 is being pushed by temperature with 2 different mechanisms
one with a response of 6months and a longer 800year deep ocean driven response.
I pointed out that the fast response would have been active in the entry/exit from ice age and therefore there should be a simultaneous (within the accuracy of the icecore record) change in CO2 with temperature as is she suggests now happening. But according to most [anti agw] the ice record shows this 800 year lag.
Anna then suggests that the fast response is a transient driven by change in temperature.
This in my view is not valid as the last 10 years have been cooling and the CO2 should therefore be dropping – the co2 record changes are detectable monthly.
Can anyone shed a light on the reason why the fast response CO2 increase is invisible in the ice core records please.

George E. Smith
May 14, 2009 10:14 am

Bill,
My understqanding is that the ice cores are not exactly like tree rings where each individual year can be isolated uniquely. So my guess is that the faster changes are being integrated by the nature of the resolution of the ice layers. The process Roy is asserting is beyond ice core resolution (I believe)
Pehaps somone “in the business” can elaborate on that.
George

George E. Smith
May 14, 2009 10:23 am

“”” RE: RW (02:35:27) :
***“There is still no convincing evidence of the CO2=heating hypothesis, of course”
..except that the very existence of a greenhouse effect proves it beyond doubt, of course.*** “””
Well RW you are a great problem solver. For a start the “greenhouse effect” isn’t really a greenhouse effect at all; green houses work via a different mechanism.
But more importantly; before you get to pointing fingers at CO2; you have to get past all the “greenhouse warming” that has already been done by the main atmospheric GHG component which is water vapor.
There’s very little warming energy left for CO2 to play with after water vapor gets through with the energy spectrum.
So NO ! the very existence of the greenhouse effect does not prove the existence of a CO2-warming link; doesn’t even hint at it, let alone prove it beyond a reasonable doubt.
And yes I do believe that CO2 molecules can and do absorb long wave infra red radiation; that does not equate to raising the global temperature.

May 14, 2009 10:51 am

Lucy Skywalker (12:41:52) :
Thank you very much – just when I thought I had an understanding of this issue!

Steve Keohane
May 14, 2009 10:55 am

bill (09:51:23)
THis may not answer your question, but the scale of temperature into and out of the glaciations is on the order of 10-12 deg. C. We haven’t had that amount of change yet, thus would not see CO2 response for that extreme a temperature shift. I know all the focus is on greenhouse effects and atmospheric composition, but it seems to me that the ocean temperature drives the atmospheric temperature due to the difference in mass. The greenhouse effect just slows the heat loss for the planet. IMO, the greenhouse effect is probably maxed out, and can’t become more of an isulator to allow temperature to climb more than another couple of degrees ever. I think we can be a lot cooler but not a lot warmer. The miniscule changes we have seen in the past century are lost in the noise level. By the time we start dropping 2-3 deg we’ll have a completely different world within a generation.
http://i40.tinypic.com/2pquhee.jpg

bill
May 14, 2009 11:51 am

George E. Smith (10:14:38) :
My understqanding is that the ice cores are not exactly like tree rings where each individual year can be isolated uniquely. So my guess is that the faster changes are being integrated by the nature of the resolution of the ice layers. The process Roy is asserting is beyond ice core resolution (I believe)

No – the entry into / out of an ice age should create a visible shift in the CO2 concentration easily within the measurement limits of ice cores
There should be a near instantaneous change in co2 when the temperature changes maintained at least for the time the temperature is changing or as I would expect a permanent step change until the temp returns to normal.
There is NONE – why?

Joel Shore
May 14, 2009 11:54 am

George E. Smith says:

But more importantly; before you get to pointing fingers at CO2; you have to get past all the “greenhouse warming” that has already been done by the main atmospheric GHG component which is water vapor.
There’s very little warming energy left for CO2 to play with after water vapor gets through with the energy spectrum.

George: Yes, those calculations have to be done but they already have been. And, it is such settled science that even Roy Spencer and Richard Lindzen accept the value for the radiative forcing of ~3.8 W/m^2 (give or take ~10%). [Where they part company with most of the rest of the scientific community is on the issue of how feedbacks operate to go from the forcing to the temperature change.]

Joel Shore
May 14, 2009 12:02 pm

Steve Keohane says:

THis may not answer your question, but the scale of temperature into and out of the glaciations is on the order of 10-12 deg. C.

Actually, the global temperature change was about half that. The numbers that you quote, presumably from the ice core graphs that you’ve seen, represent the temperature changes in the polar regions which evidence suggests are roughly double the globally-averaged temperature changes.

The greenhouse effect just slows the heat loss for the planet. IMO, the greenhouse effect is probably maxed out, and can’t become more of an isulator to allow temperature to climb more than another couple of degrees ever. I think we can be a lot cooler but not a lot warmer.

There is no evidence that I know of that this is the case. In fact, the dependence of temperature on concentration is expected to be approximately logarithmic…which is why people talk about how much change is produced by a doubling of concentration. (For a logarithmic function, the amount of change due to each doubling is a constant.)
Furthermore, going back millions of years, there are past climates that were considerably warmer than our current climate.

May 14, 2009 12:16 pm

George E. Smith (10:14:38) :
My understqanding is that the ice cores are not exactly like tree rings where each individual year can be isolated uniquely.
Pretty much they are, yes. Except near the bottom of the cores where the pressure has compressed the rings too much. For that part, a model of the compression with depth is used to get the age.
http://www.sciencelearn.org.nz/contexts/icy_ecosystems/sci_media/images/ice_core_showing_annual_rings

Paul Vaughan
May 14, 2009 12:50 pm

Further to George E. Smith (10:23:33) & Leif Svalgaard (12:16:17) who have responded to bill (09:51:23):
I’ve been following the interesting exchange between anna v & bill in the other WUWT thread. An exercise I think would help bill here:
Time-integrate monthly dCO2/dt at varying bandwidth (i.e. 1mo, 2mo, 3mo, 4mo, …). Note particularly what happens at multiples of 12. Consider related insight in conjunction with George’s & Leif’s comments.
Also, in reviewing anna v’s arguments, be careful when inferring-from-context whether she is referring to CO2 or dCO2/dt.
A final suggestion would be to question your own assertions about time-invariant best-lags. To gain related intuition, you might want to try some windowed time-integrated cross-correlation analyses.
I also recommend similar time-integration exercises for Pamela Gray due to the nature of comments appearing at Pamela Gray (06:44:40).
A related reference:
T.F.H. Allen & T.W. Hoekstra (1992). Toward a unified ecology. Columbia University Press, New York.
This book draws attention to the nature of very serious research errors stemming from inattentiveness to the influence of spatiotemporal heterogeneity on parameter estimates across spatiotemporal scale. A study that fails to investigate the influence of summary scale on parameter estimates in the presence of spatiotemporal heterogeneity is at-best incomplete and at-worst severely misleading.

maksimovich
May 14, 2009 1:07 pm

“What could be causing long-term warming of the oceans? My first choice for a mechanism would be a slight decrease in oceanic cloud cover. There is no way to rule this out observationally because our measurements of global cloud cover over the last 50 to 100 years are nowhere near good enough.”
Indeed Albedo is to climate science, in what an egg is to theology.

anna v
May 14, 2009 1:09 pm

Lets see. The six month lag shown in the plots above, is rate of change of CO2 per year and temperature anomaly per year. Neither have the scale of the effect there, and there is not obvious way to know if the scale of the effect is the same when the absolute temperature is -30C as when it is 30C. We can only guess. There might be a logarithmic dependence, or an exponential one for all we know.
I wonder if one took the measured values in the antarctic of temperatures and CO2 whether the effect of the plots above would be seen? Certainly antarctic CO2 has a smaller amplitude change due to seasonal variations.
My guess: Ice core records come from regions where there is ice, i.e. below 0C where there is no flora, so only the cold ocean bubbling mechanism will dominate for shorterm effects. How much in ppm C02 comes out when going from -30C to -10C? It might still follow temperature, but be too small to count. One would have to wait for the deep ocean rich in CO2 turnover to get an effect.

Gerald Machnee
May 14, 2009 2:33 pm

Joel Shore (12:02:13) :
**There is no evidence that I know of that this is the case. In fact, the dependence of temperature on concentration is expected to be approximately logarithmic…which is why people talk about how much change is produced by a doubling of concentration. (For a logarithmic function, the amount of change due to each doubling is a constant.)**
“The amount of change due to each doubling is a constant”
What does this mean? When you check the graph the change decreases significantly with each doubling.
Check slide 22 of David Archibald’s paper at:
http://www.davidarchibald.info/papers/Solar%20Cycle%2024%20-%20Implications%20for%20the%20Unites%20States%20David%20Archibald%20March%202008.pdf

bill
May 14, 2009 2:46 pm

anna v
I showed some plots of Greenland ice cores on the other thread. You pointed out the CO2 AIRS animations
http://svs.gsfc.nasa.gov/vis/a000000/a003500/a003562/
Greenland is experiencing more or less the same range of changes of CO2 as the rest of the NH but for shorter times. So CO2 from these cores can reflect the temperate zone levels.
When you talk about temperatures of -30C this is not the temperature of the whole globe and vegetation would still be in abundance further south.
Paul Vaughan (12:50:17) :
Time-integrate monthly dCO2/dt at varying bandwidth (i.e. 1mo, 2mo, 3mo, 4mo, …). Note particularly what happens at multiples of 12. Consider related insight in conjunction with George’s & Leif’s comments.

I take it that you have done this and could share you results and conclusions? I would have thought that the accurate record is insufficienltly long to obtain much useful info – but prove me wrong.
I’m not sure what you are saying about leif and george’s comments. There are many ice cores, some have good resolution early on gradually getting worse as shear and compression take their toll. At end of first iceage the granularity is about 150years. Should show up rises in co2 similar to the current

George E. Smith
May 14, 2009 3:26 pm

“”” bill (11:51:36) :
George E. Smith (10:14:38) :
My understqanding is that the ice cores are not exactly like tree rings where each individual year can be isolated uniquely. So my guess is that the faster changes are being integrated by the nature of the resolution of the ice layers. The process Roy is asserting is beyond ice core resolution (I believe)
No – the entry into / out of an ice age should create a visible shift in the CO2 concentration easily within the measurement limits of ice cores
There should be a near instantaneous change in co2 when the temperature changes maintained at least for the time the temperature is changing or as I would expect a permanent step change until the temp returns to normal.
There is NONE – why? “””
I thought we were talking about resolving the six months or less time lag that Dr Spencer was talking about.
But if Leif Says that each single year can be positivlely identified (or implies that) well that’s good enough for me. All the tv programs I have seen on ice cores seemed to suggest that wasn’t that clear; but then that’s TV for you. I’ve got a Scientific American Article from fairly recent years that shows two Antarctic ice cores that over the last 1000 years show CO2 and temperature data going in exactly the opposite directions in the two cores.
So much for ice cores as a global indicator. It was in an article in which the author claimed that man started global warming 8000 years ago and stopped an ice age in its tracks.

George E. Smith
May 14, 2009 3:39 pm

Speaking of CO2 in the oceans; my CO2 expert at Scripps, says that only about 15 of dissolved CO2 is in the from of CO2, and that most appears as either HCO3- or CO3- – ions.
So a question for the Physical Chemists (which excuses me) what part of that mix, of CO2/CO3- -/HCO3- is in equilibrium with 385 ppm of CO2 in the atmosphere under the direction of Henry’s Law (and I do mean under equilibrium conditions) recognizing that real world may not be equilibrium.
I’m wondering whether Henry’s Law sets the CO2 gas in solution of whether it sets the total including the ionic species; or something else entirely.
I presume that when the water freezes, that all three species are largely excluded from the ice, but I wonder how much of that must then exit to the atmosphere.
If the wild 18 ppm swing of CO2 in the arctic is biological as in permafrost peat bogs outgassing; why would those sources then take up the CO2 again, and wouldn’t those bogs give off CO2 while in the molten state, whereas the ocean gives up CO2 while freezing.
Enquiring minds want to know.
George

bill
May 14, 2009 3:45 pm

Ice core data gives about 40 year temp resolution and as said above 150 year CO2 at 12000 years ago.
These are the published data. Perhaps yearly info is available.
The general statement in this blog is that there is a 800year(ish) lag of CO2 from temperature hence there is no way CO2 controls temperature. This article suggest that the lag is 6 months. This disparity needs an explanation.

Nick Stokes
May 14, 2009 4:26 pm

George,
I expect your shift key failed, and you meant 1%, which is about right. As I recall, it’s about 1% CO2, 88% HCO3- and 11% CO3–.
And yes, it’s the 1% that is in equilibrium with gas CO2. But there are linked equilibria. So if some extra CO2 dissolves, then the equilibrium of CO2 with HCO3- shifts to make more HCO3-, and there’s even a bit more CO3–. This is buffering, and the amount of dissolved CO2 stays at just over 1%.
In fact, the CO3– is in equilibrium with solid CaCO3, including shells, corals, etc, and that equilibrium can shift too, dissolving some CaCO3.

Paul Vaughan
May 14, 2009 5:29 pm

Re: anna v (13:09:22)
You’ve raised a lot of interesting points. I thank you, Dr. Spencer, Phil., and others (including those who have posted links to data) for causing me to take a far more careful look at CO2 than I have previously.

After taking a first exploratory look at some of the data, one important thing that seems to be escaping the (core) focus of the discussion is the substantial north-south asymmetry of Earth.

A comment I would offer to several who have posted:
Misrepresenting Dr. Spencer’s presentation is a questionable tactic. In particular, many have seriously misrepresented the meaning of the 90/10 split; the discussion barely even attempted to focus on what it really means. Also, many have failed in their interpretations to acknowledge the proportion of the variation explained by the demonstrative-model put forward to stimulate exploration, discussion, & learning.
I hope Phil. will take up Anthony’s offer. [Note: I found Phil.’s comments at Phil. (20:44:36) & Phil. (06:43:38) instrumental in this discussion.]
Anthony: Thank you for hosting this discussion. These discussions cause me to dig deeper – & in areas that might not regularly cross my mind.
– – –
Re: Leif Svalgaard (08:41:22)
I appreciate your concerns, but for balance I will comment on the downside:
That would leave the system wide-open to abuse by administrative-types who substitute overtime-hours for true talent. I have had more than my share of dealings with such folks who have unyieldingly-discordant domination instincts – and I know they would seize such an easy opportunity to deliberately engineer crippling inefficiency into the system — they simply cannot help themselves – and since their tactics can undermine the sustainable defense of civilization, critical stalemating effort needs to be invested in the maintenance of balance.
Perhaps there is a middle-ground approach that does not necessarily become yet another opportunity for “adminabalism” (administration+ballistic+cannibalism). We are naive if we let our guard down, thinking the science establishment is immune to:
1) ideologically-driven system-self-destruct political appointees & affiliates (on a mission to tear down institutions organized around values perceived as a political threat).
2) other (perhaps less-malicious) entities intent on governing which tools (such as low-confidence computer models) are employed to manage culture.
Most honest researchers I know are already nearly-overwhelmed, as stringent administrative controls creep deeper & deeper into the research arena. I would argue that the efficient-research solution is more liberation, not more control. Bright folks will have more time to cover more ground, spot research deficiencies, and innovate if they spend less time tied up with infighting & formal administrative processes that suck time, resources, & mental freshness.
I understand, appreciate, & respect the concern you have expressed; I add this comment with balance in mind.
– – –
bill (14:46:30) “I take it that you have done this and could share you results and conclusions?”
Based on the pattern in your comments, you really need to do these calculations for yourself. If you don’t have 10 minutes to do them in Excel, you can use the webpage mentioned upthread by others to gain insight in mere seconds:
http://www.woodfortrees.org/plot/esrl-co2/isolate:2/mean:1/scale:1/from:1990/to:2000
Adjust the “mean (samples)” “Value” upwards in increments of 1 (month) and notice what happens as you roll through multiples of 12 (i.e. 12, 24, 36, …). This should help you discern the manner in which samples integrated to annual (& subharmonic) resolution obscure seasonal signals.
A little more:
If you take the time to look at the Antarctica (“South Pole”) series [perhaps you have already done this, but perhaps without exploring time-integration], you will notice a 6 month phase shift. If you additionally look at the other sites, you will notice striking geographic variation in both amplitude & seasonal pattern.
Final (more general) note:
Beware the hazards of drawing conclusions without investigating the sensitivity of parameter estimates to scale & localization in the presence of spatiotemporal heterogeneity.

May 14, 2009 5:40 pm

bill (15:45:22) :
Ice core data gives about 40 year temp resolution and as said above 150 year CO2 at 12000 years ago.
I don’t know about temps, but the time resolution for 10Be [and for the annual rings] is one year…

Pamela Gray
May 14, 2009 6:03 pm

Leif, I spit Coors Light on my puter screen!!!! Your statement is either (or both) the understatement of the year, or the overstatement of the year! It reminds me of the question, “Who is buried in Grant’s tomb?”

Paul Vaughan
May 14, 2009 6:11 pm

Re: Nick Stokes (16:26:17)
Thank you for these comments (on linked chemical equilibria), which make the picture a whole lot more interesting.
Could you (or anyone else) comment on the role of wind variations (& related surface roughness) in hemispheric, regional, local, & microspatial variations in these equilibria? – and perhaps also comment on how this varies across temperature gradients?
Maybe someone knows of a link to a really good primer?

May 14, 2009 6:12 pm

Paul Vaughan (17:29:55) :
I appreciate your concerns, but for balance I will comment on the downside:
No balance is needed, nor wanted. If the review process is public and transparent, the system cannot be abused.

Paul Vaughan
May 14, 2009 7:03 pm

Leif Svalgaard (18:12:43)
“If the review process is public and transparent, the system cannot be abused.”

The “abuse” will come in the tedious time-consumption – i.e. scientists getting caught hostage by the process. Since each scientist’s time is finite, this will bite into time for innovation.
However, clusters of bright minds will naturally engineer means of balancing cumbersome administrative duties (“red tape”) with research objectives.
Leif Svalgaard (18:12:43) “No balance is needed, nor wanted.”

May 14, 2009 8:16 pm

Paul Vaughan (19:03:44) :
“If the review process is public and transparent, the system cannot be abused.”
The “abuse” will come in the tedious time-consumption – i.e. scientists getting caught hostage by the process.

Not at all. I regularly review papers by other scientists. An example is my review of Dikpati’s prediction of a very large cycle 24: http://www.leif.org/research/Dikpati%20Referee%20Report.pdf
Not tedious, but necessary, and at times rewarding because the review forces one to focus on the idea [and even learn a bit].

Steve Keohane
May 14, 2009 8:33 pm

Joel Shore (12:02:13)
Depending upon what you look at, the earth has never been more than 5-10 deg C warmer than it is now in the past 6 X 10^8 years. In the past 450K years it hasn’t been over 2-3 degrees above present. We have had 5000 ppm of CO2 and the temperature has never run away. We have only one source of heat, the sun. Our climate system varies by retaining more or less of that heat by whatever mechanism, and we certainly have very few facts about how it does that. What we don’t know about climate is far more than we do. I have been looking at this for 48 years, and without some new measured, real-world correlated facts you aren’t going to convince me otherwise.

Nick Stokes
May 14, 2009 8:55 pm

Paul Vaughan (and George Smith)
Here is a good set of notes on dissolved carbonate chemistry. The last page (sec 6.2) is quite relevant here.

Jeremy
May 14, 2009 8:57 pm

@E.M. Smith
“We’ve reach “Peak Fish” some decade or two ago. (As of now some 30% of all fish eaten is aquacultured so nobody cared much about the Peak Fish crisis…) The fastest we can harvest them from the ocean. Do you think that maybe hauling billions of pound of fish out of the ocean might reduce the quantity of carbonate pellets the fish in the ocean can excrete?
Do you think that might leave more CO2 in the ocean to outgas?”
Do you think the “dead zones” we have occasionally noticed off the coasts of continents might be symptoms of this?

Paul Vaughan
May 14, 2009 11:57 pm

Realism:
Steve Keohane (20:33:40) “What we don’t know about climate is far more than we do.”
Very refreshing to see non-partisan honesty.

May 15, 2009 1:06 am

George E Smith,
I’m not formally an expert but here’s my understanding to help your questions FWIW
(1) the balance of CO2, HCO3-, and CO3– is a whole equilibrium chain; changing one affects all; the net balance is that CO2 partitions between water and air at 15ºC approximately 50:1 by Henry’s Law; this proportion changes a tiny bit as the solubility decreases with increasing temp; but with the size of the oceans, a tiny change of temperature magnifies into a huge outgassing.
(2) the NH seasonal “wild swing” of CO2 is the effect of the temperate forests: Canada and the Russian taiga. In winter there is net exhaling of CO2 from rotting-down processes.
My chief sources here for understanding CO2 are:
Glassman http://www.rocketscientistsjournal.com/2006/10/co2_acquittal.html
Floor Anthoni http://www.seafriends.org.nz/issues/global/acid2.htm#carbon_situation (he explained why ocean acidification is more alarmist nonsense – because of the ever-present Ca++ ions, ready to make shells)
Segalstad http://www.greenworldtrust.org.uk/Science/Scientific/Segalstad.htm
and I’ve done my own page of notes on CO2 http://www.greenworldtrust.org.uk/Science/Scientific/CO2-flux.htm
Hope this helps

RW
May 15, 2009 1:11 am

Gerald Machnee:
“if CO2 is the main factor in temperature increase, then the global temperature must increase EVERY year”
Not true at all. Why would CO2 stop weather?
“Check the latest studies – The 13C is not only from fossil fuels”
Link to those ‘latest studies’ please.
George E. Smith:
“And yes I do believe that CO2 molecules can and do absorb long wave infra red radiation; that does not equate to raising the global temperature.”
That is a horrible self-contradiction. Whatever absorbs more radiation, gets warmer. There is more CO2, hence more absorption, hence…
anna v:
“The isotope card has been burned since it was noted that part of the algae/ plankton prefer the C13 way. There is lots and lots of them in the oceans.”
Do you believe that the algae and the plankton suddenly started going mental about 200 years ago, after hundreds of thousands of years of not doing very much? Why would they have done that?

Roy W. Spencer
May 15, 2009 4:57 am

Sorry I have not been able to respond…I was in Stockholm yesterday and Helsinki today. I quickly scanned the various comments that have been made, and would like to clarify….
The two coefficients in the simple model I used have different units…in fact, the anthropogenic one is unitless (just a fraction). I think someone pointed out that since the observed atmospheric rise is 50% of the anthropogenic emissions, then my “10% anthropogenic” fit is really 20%, which is true…I thought of this shortly after posting but was too busy traveling to do anything about it. Similarly, the “20% anthropogenic” fit is then really 40% anthropogenic.
In case anyone missed it, the main point of the post was not to advance a new and complete explanation of the connections between CO2 and temperature. It was to show there is very clear evidence that temperature — either directly or indirectly (through biological activity) does impact the atmospheric concentration of CO2…at least on interannual time scales. Now, since El Nino has been more prevalent in the last 30 years, isn’t it possible that *some* of the warming and CO2 increase during that time is natural?
It is a perfectly legitimate scientific hypothesis, and the fact that it is avoided like the plague illustrates how political climate science has become.
-Roy

anna v
May 15, 2009 5:12 am

Do you believe that the algae and the plankton suddenly started going mental about 200 years ago, after hundreds of thousands of years of not doing very much? Why would they have done that?
Please give a link for the hundreds of thousands of years of not doing very much.
200 years ago the temperatures started rising because of getting out of the little ice age, and the flora responded accordingly. One can imagine many reasons why a C13 rich strain might flower better with heat.
The argument about fossil fuel origin of C13 can hold only if there are no other sources of C13 in the game.

bill
May 15, 2009 5:25 am

Roy W. Spencer (04:57:10) :
… It was to show there is very clear evidence that temperature — either directly or indirectly (through biological activity) does impact the atmospheric concentration of CO2…at least on interannual time scales.

But then doesn’t this lead on to my points that if temperature is pushing CO2 AND it is doing it with a 6 month lag it SHOULD appear in the ice core records as effectively zero lag between CO2 and Temperature. An 800(approx) lag is usually quoted?
Whilst biologic conditions during the iceage will be different (although still active in warmer latitudes) the entry into the icae age from temperatures (and presumably similar vegitation) that is similar to today should show only a 6 month lag:
entry into iceage CO2 falls 4000 years after temp starts to drop
http://img12.imageshack.us/img12/9952/iceage100200kbq5.jpg
exit from ice age (last) shows nearly simulateous co2 and temperature (which is pushing which?)
http://img11.imageshack.us/img11/6826/iceage040kkq1.jpg

anna v
May 15, 2009 5:40 am

Dr Spencer, ( or anybody else who has access to the data)
It would be really interesting if you could make the same plots of CO2 changes and temperature anomalies from the data of the antarctic only. i.e. antarctic temperature anomalies versus CO2 yearly changes.
It is interesting that with such a simple model a first order “explanation” can be found.

Joel Shore
May 15, 2009 6:49 am

Gerald Machnee says:

“The amount of change due to each doubling is a constant”
What does this mean? When you check the graph the change decreases significantly with each doubling.
Check slide 22 of David Archibald’s paper at:
http://www.davidarchibald.info/papers/Solar%20Cycle%2024%20-%20Implications%20for%20the%20Unites%20States%20David%20Archibald%20March%202008.pdf

It means exactly what it says: For a function like y = A + log(x), each doubling of x produces the same amount of change in y. So, going from 280ppm to 560ppm produced the same amount of change in y as going from 140ppm to 280ppm. This is in contrast to a linear function where the same amount of change in x produces the same amount of change in y…In particular, going from 280ppm to 560ppm would produce the same amount of change in y as going from 0ppm to 280ppm.
I don’t think what I said is in contradiction to what Archibald shows although I haven’t tried to look in detail at the quantitative values that he shows.
RW says:

Gerald Machnee:
“if CO2 is the main factor in temperature increase, then the global temperature must increase EVERY year”
Not true at all. Why would CO2 stop weather?

Indeed…and a simple way to see this is to consider the seasonal cycle in a place such as here in Rochester. I think everyone would agree that it is a very significant feature of the climate here…and yet as we go from winter to summer, we do not see a steady temperature increase each day. I haven’t looked at the actual data but my guess would be, for example, that week-long periods in the spring with negative temperature trends are not at all uncommon.

May 15, 2009 7:01 am

To understand the minuscule effect of even doubling atmospheric CO2, check out the graph on Page 22 of David Archibald’s paper: click
That Page 22 graph is another nail in the coffin of the repeatedly discredited, falsified and ridiculous notion that CO2 = AGW.

Gerald Machnee
May 15, 2009 7:26 am

RE Joel Shore (06:49:08) :
*** Gerald Machnee:
“if CO2 is the main factor in temperature increase, then the global temperature must increase EVERY year”
Not true at all. Why would CO2 stop weather?
Indeed…and a simple way to see this is to consider the seasonal cycle in a place such as here in Rochester. I think everyone would agree that it is a very significant feature of the climate here…and yet as we go from winter to summer, we do not see a steady temperature increase each day. I haven’t looked at the actual data but my guess would be, for example, that week-long periods in the spring with negative temperature trends are not at all uncommon.***
Please read what I said, “Then the global temperature must increase EVERY year.”
I was not talking about daily or seasonal variations or local at your house!! I said GLOBAL and YEAR!

Gerald Machnee
May 15, 2009 7:38 am

RE Joel Shore (06:49:08) :
***It means exactly what it says: For a function like y = A + log(x), each doubling of x produces the same amount of change in y. So, going from 280ppm to 560ppm produced the same amount of change in y as going from 140ppm to 280ppm. This is in contrast to a linear function where the same amount of change in x produces the same amount of change in y…In particular, going from 280ppm to 560ppm would produce the same amount of change in y as going from 0ppm to 280ppm.***
No.
I think you should review the mathematics. Each successive doubling has a decreased effect. Only the first doubling of CO2 had a significant result on the temperature. any doubling now is a fraction of a degree. Please check slide 22.

Roy W. Spencer
May 15, 2009 7:53 am

Bill:
I suspect that CO2 changes on all time scales…seasonal, interannual, decadal, centennial, etc. The time-smoothing inherent in the ice core record from long ago means you will only see the longest time scale relationships in the ice cores. I also suspect the different time scales correspond to different physical (or biological) mechanisms.
In my blog posting I’m showing a clear interannual relationship, and hypothesizing that it might also extend to decadal and longer time scales. If it does, then some significant part of the observed CO2 increase in the last century is likely natural. Since mankind emits more than is needed to explain the observed increase, this also means the “missing sink” of carbon dioxide is bigger than is currently appreciated.

bill
May 15, 2009 8:17 am

Roy W. Spencer (07:53:15) :
The time-smoothing inherent in the ice core record from long ago means you will only see the longest time scale relationships in the ice cores.

Are you also suggesting that the fast response is transient? Could you please explain why because I just cannot see why that would be. Thanks
If transient then after 30 years of increase are we now due for an unexpected fall?

Richard Sharpe
May 15, 2009 8:25 am

Gerald MacNhee says, in responding to Joel Shore:

I think you should review the mathematics. Each successive doubling has a decreased effect. Only the first doubling of CO2 had a significant result on the temperature. any doubling now is a fraction of a degree. Please check slide 22.

Y = A + log2(X): A = 0. At X = 1, Y = 0, at, at X = 2, Y = 1, at X = 4, Y = 2, at X = 8, Y = 3!
Thus, as Joel says, each doubling of X produces a linear change in Y, a unit increase in Y. Note, I chose log2(X) to make the numbers easier.
However, for Y = A + nX, where A = 0, and N = 1, each doubling in X produces a doubling in Y.
I think what you actually meant to say when you said:

Each successive doubling has a decreased effect

Is: Each successive increase has a decreased effect.
However, I don’t want to put words in your mouth.
To achieve what you said would require a function with history or scaling it with some factor that gets smaller the further along the X axis you go.

Richard M
May 15, 2009 8:46 am

Joel Shore:
“I haven’t looked at the actual data but my guess would be, for example, that week-long periods in the spring with negative temperature trends are not at all uncommon.”
And these are all explainable by weather systems (primarily the jet stream movement). Exactly how to do you explain the lack of increasing temps for the last few years. Please provide the exact mechanism. Thanks.

Gerald Machnee
May 15, 2009 9:49 am

RE: Richard Sharpe (08:25:38) :
**I think what you actually meant to say when you said:
Each successive doubling has a decreased effect
Is: Each successive increase has a decreased effect.
However, I don’t want to put words in your mouth.
To achieve what you said would require a function with history or scaling it with some factor that gets smaller the further along the X axis you go.**
I think that what I said is correct. Anyway, I will not quibble over wording. I have not worked with logs (either type) for a while. That is why I referred to David Archibald’s chart on slide 22 of his presentation. Now that is not his originally, but he used the calculations from MODTRAN facility in Chicago.
Your calculations above are still linear in result.
The MODTRAN calculations show that the first 20 ppm increase in CO2 gave a temperature increase of about 1.55 deg C. the next 20 ppm give an increase of about 0.3 deg C. The next 20 ppm give an increase of less than 0.2 deg C. So by the time the ppm reached 280, we had achieved most of the temperature increase due to CO2.
There are also others who talk about the logarithmic effect, but too many are ignoring it.

George E. Smith
May 15, 2009 10:27 am

“”” RW (01:11:44) :
Gerald Machnee:
“if CO2 is the main factor in temperature increase, then the global temperature must increase EVERY year”
Not true at all. Why would CO2 stop weather?
“Check the latest studies – The 13C is not only from fossil fuels”
Link to those ‘latest studies’ please.
George E. Smith:
“And yes I do believe that CO2 molecules can and do absorb long wave infra red radiation; that does not equate to raising the global temperature.”
That is a horrible self-contradiction. Whatever absorbs more radiation, gets warmer. There is more CO2, hence more absorption, hence…
“””
Sorry RW; not contradictory at all. It certainly might raise a local temperature; specifically the temperature of the atmosphere where the CO2 is but that in turn sets in motion other effects; some of which lead to increases in cloud cover (globally); and increases in cloud cover lead to cooling (locally); but the overall global effect may be quite negligible. And in fact we know from measured data that increases in CO2 have not caused any significant increases in global temperature; which incidently we don’t even have any reliable way of measuring.
So we have had a 37.5 % increase in atmospheric CO2 since the end of the 19th century; which continues apace to this day, and essentially no credible evidence of a significant temperature increase.
Sure some temperature anomaly data sets suggest that those data sets have recorded an increase; but they don’t reflect any scientific measure of a global temperature.
And prior to around 1980, when oceanic buoys first started to measure both air and water temperature; we now know that all that data taken from 70% of the earth’s surface; namely the oceans, is useless, since water and atmospheric temperatures aren’t correlated; so the air temperatures cannot be reconstructed from the water temperatures that were historically measured.
When the computer climate models properly include the full physical effects of the most important GHG namely water; then come and tell us that CO2 increases cause GLOBAL warming.
Right now we appear to have global cooling; yet we still have increasing CO2 in the atmosphere; obviously something else is seeing to it that the effect of the CO2 increase is completely squashed.

George E. Smith
May 15, 2009 10:53 am

“”” Nick Stokes (20:55:14) :
Paul Vaughan (and George Smith)
Here is a good set of notes on dissolved carbonate chemistry. The last page (sec 6.2) is quite relevant here. “””
Hey Nick; so you want to put this old codger to work relearning some chemistry ?
That is a very nice piece of relevent information; more than I ever expected to get my hands on. And yes I am going to have to hit the books to freshen up what chemistry I learned in High school; but I can at least read it now.
And yes like a good Sherlock Holmes you unmasked my lazy shift thumb; pretty soon I could get to just remember that (5) is Bunglese for percent(%).
Thanks for the help; I’m determined to become very friendly; even intimate with the ocean/atmosphere carbon exchange; although I sometimes wonder why; because I’m pretty darn certain that CO2 doesn’t have much to do with the earth’s temperature; well at least as far as causing it; not with all that water around to veto the CO2.
Thanks for the help.

oms
May 15, 2009 11:05 am

Gerald Machnee (09:49:26) :

Your calculations above are still linear in result.
The MODTRAN calculations show that the first 20 ppm increase in CO2 gave a temperature increase of about 1.55 deg C. the next 20 ppm give an increase of about 0.3 deg C. The next 20 ppm give an increase of less than 0.2 deg C. So by the time the ppm reached 280, we had achieved most of the temperature increase due to CO2.

Gerald, also keep in mind that the log relationship for radiative forcing is between the mixing ratio and the forcing. The first 20 ppm will affect the mixing ratio much more than the last 20 ppm.

Richard Sharpe
May 15, 2009 11:18 am

Gerald MacNhee says:

The MODTRAN calculations show that the first 20 ppm increase in CO2 gave a temperature increase of about 1.55 deg C. the next 20 ppm give an increase of about 0.3 deg C. The next 20 ppm give an increase of less than 0.2 deg C. So by the time the ppm reached 280, we had achieved most of the temperature increase due to CO2.
There are also others who talk about the logarithmic effect, but too many are ignoring it.

What you have just described is the increase in the output of a function decreases for every linear increase in the input.
X + 20ppm ==> Y + 1.55C
X + 20ppm + 20ppm ==> Y + 1.55C + 0.3C
X + 20ppm + 20ppm + 20ppm ==> Y + 1.55C + 0.3C + 0.2C
So, at some point we will get to:
X + X ppm ===> Y + aC (here a is a constant).
X + X + X + X ppm ===> Y + 2aC
This is what a logarithmic function does.

George E. Smith
May 15, 2009 11:18 am

“”” Nick Stokes (16:26:17) : “””
Nick, I noticed the book chapter referred to “hydrated” CO2, rather than CO2.
I’m guessing that they are implying that the 1% CO2 is actually in some complex of the CO2n(H2O) variety. Those crazy water molecules do have a habit of grabbing onto other things.
Almost makes me wish I had studied more chemistry.
And yes I understand how the CO2 (aqua)/CO2 (atm) Henry’s law equilibrium will “trickle down” to the HCO3- and CO3- – concentrations as well (roughly); and I’m not surprised that the calcium carbonate shells will also play a role. It’s a wonder we can ever get anything straight with all these interractions.
Most immediaterly, I’m trying to figure out; if freezing sea water ejects the CO2/HCO3-/CO3–species from the solid, into the water, how much extra CO2 should be emitted into the atmosphere for each cubic metre of new ice formed; and how that compares with the 18 ppm change over the arctic annual cycle. It seems to me the amount should be pretty large, but maybe not so when compared to the land based seasonal emissions (tundra and permafrost bogs)
George

Paul Vaughan
May 15, 2009 1:08 pm

Suggested exercise for bill, motivated by bill (05:25:02):
1) Create a sinusoidal wave – call it wave A.
2) Create a second sinusoidal wave (wave B) with the same period but half-a-cycle out-of-phase with wave A.
3) Apply a moving-average of bandwidth 1 cycle to both waves A & B.
4) Note the cross-correlation best-lag for cycle-integrated-A with cycle-integrated-B.
5) Create a longer wave of period p where p is much greater than 1 (i.e. much greater than the period of waves A & B) – call it wave C.
6) Translate wave C in time by some amount h where h is less than p – call the result D.
7) Add wave C to (unsmoothed) wave A – call the result wave E.
8) Add wave D to (unsmoothed) wave B – call the result wave F.
9) Apply a moving-average of bandwidth 1 cycle (i.e. a moving-average with bandwidth equal to the period of waves A & B) to both waves E & F.
10) Note the cross-correlation best-lag for A&B-cycle-integrated-E with A&B-cycle-integrated-F.
11) Repeat steps 1 to 10 introducing random amplitude variations.
12) Repeat steps 5 to 10 introducing random phase variations.
13) Explore steps 11 & 12 in concert.
Questions:
a) Do you understand that if temporal sampling resolution is 1 year, seasonal variations will be smoothed over?
b) What was your main insight from the exercise I suggested earlier? [See Paul Vaughan (17:29:55) – latter portion, addressed to bill (14:46:30).]
– – –
Re: Lucy Skywalker (01:06:41)
Can you share the standard explanation that is offered for the winter dCO2/dt dip at Alert (in northern Canada)? [Contextual note for anyone following along: The main dip at that location occurs in summer, but there is a smaller, narrower (but striking & substantial) dip in winter.]

Paul Vaughan
May 15, 2009 1:15 pm

Roy W. Spencer (04:57:10) “[…] It is a perfectly legitimate scientific hypothesis, and the fact that it is avoided like the plague illustrates how political climate science has become.”
Thank you for your comments Dr. Spencer.
Can you point out some publications related to your above presentation? Although I have not been down that branch of the literature, I am left with the impression that it might not exist – and if that is the case, I plan to launch an investigation.
By the way: I never found CO2 to be a very interesting variable to study before seeing your WUWT post (above) and the resulting discussion.
– – –
anna v (05:40:30) “[…] anybody else who has access to the data […]”
Clarification:
The data are available publicly in text-format via:
http://cdiac.ornl.gov/trends/co2/sio-keel.html
ftp://ftp.cmdl.noaa.gov/ccg/co2/in-situ/

Gerald Machnee
May 15, 2009 1:20 pm

RE: Richard Sharpe (11:18:01) :
Are you saying that:
**What you have just described is the increase in the output of a function decreases for every linear increase in the input.
X + 20ppm ==> Y + 1.55C
X + 20ppm + 20ppm ==> Y + 1.55C + 0.3C
X + 20ppm + 20ppm + 20ppm ==> Y + 1.55C + 0.3C + 0.2C
So, at some point we will get to:
X + X ppm ===> Y + aC (here a is a constant).
X + X + X + X ppm ===> Y + 2aC
This is what a logarithmic function does.**
is the same as below?
Y = A + log2(X): A = 0. At X = 1, Y = 0, at, at X = 2, Y = 1, at X = 4, Y = 2, at X = 8, Y = 3!

Joel Shore
May 15, 2009 1:52 pm

Gerald Machnee:

The MODTRAN calculations show that the first 20 ppm increase in CO2 gave a temperature increase of about 1.55 deg C. the next 20 ppm give an increase of about 0.3 deg C. The next 20 ppm give an increase of less than 0.2 deg C. So by the time the ppm reached 280, we had achieved most of the temperature increase due to CO2.

Actually, the technically-correct statement is that MODTRAN gives a certain value for the radiative forcing, which Archibald has then converted to a temperature rise by assuming a climate sensitivity of 0.1°C per watt/m2. This is well lower than the generally-accepted range of ~0.5-1.0°C per watt/m2. (Even in the absence of feedbacks, it would be ~0.25°C per watt/m2, so 0.1°C per watt/m2 assumes strong negative feedbacks.

There are also others who talk about the logarithmic effect, but too many are ignoring it.

I don’t know who you speak of ignoring it. As I noted above, the whole reason that scientists talk about the climate sensitivity to a doubling of CO2, rather than an increase by some constant increment, is because of this logarithmic dependence. However, it is also worth noting that the difference between a logarithmic and linear function is only very noticeable if the change in concentration is a large fraction of the initial concentration. So, when we go from 300ppm to 320ppm, the increase is almost as great as going from 280ppm to 300ppm. (For example, if the 280 to 300ppm caused an increase of 0.3°C then the 300 to 320ppm would cause an increase of ~0.28°C.)

Gerald Machnee
May 15, 2009 2:39 pm

RE: Joel Shore (13:52:18) :
Gerald Machnee:
**Actually, the technically-correct statement is that MODTRAN gives a certain value for the radiative forcing, which Archibald has then converted to a temperature rise by assuming a climate sensitivity of 0.1°C per watt/m2. This is well lower than the generally-accepted range of ~0.5-1.0°C per watt/m2. (Even in the absence of feedbacks, it would be ~0.25°C per watt/m2, so 0.1°C per watt/m2 assumes strong negative feedbacks.**
Archibald’s figure is a calculation based on what he wrote.
The “generally-accepted range” is based on feedbacks which are assumed or modelled, and which are being questioned now. More study is required to decide if the feedbacks are negative or positive.
**I don’t know who you speak of ignoring it. As I noted above, the whole reason that scientists talk about the climate sensitivity to a doubling of CO2, rather than an increase by some constant increment, is because of this logarithmic dependence. However, it is also worth noting that the difference between a logarithmic and linear function is only very noticeable if the change in concentration is a large fraction of the initial concentration. So, when we go from 300ppm to 320ppm, the increase is almost as great as going from 280ppm to 300ppm. (For example, if the 280 to 300ppm caused an increase of 0.3°C then the 300 to 320ppm would cause an increase of ~0.28°C.)**
Ignoring – the mainstream does not even discuss it.
You note that when you go from 300 to 320 the increase is the same as from 280 to 300. That is correct. However, your figures of 0.3 and 0.28 are much too large and are based on too high a positive feedback..
As I noted in another post, Steve McIntyre has an open question to IPCC or anyone else to provide a detailed engineering quality derivation of the change or sensitivty that a doubling of CO2 will produce. To date there have been no responses other than quoting IPCC which is based on Charney(1979) and lacks the detail.

RW
May 15, 2009 3:13 pm

Roy Spencer:
“isn’t it possible that *some* of the warming and CO2 increase during that time is natural?”
Of course. It’s not just possible but certain. But natural mechanisms can only account for an extremely small part of the CO2 rise. Once again, it’s really very very simple – humans have burned a lot of fossil fuels, and this has resulted in the emission of a lot of CO2; fossil fuels have a lower 13C content than either oceanic or atmospheric CO2; both oceanic and atmospheric CO2 is going up, and both oceanic and atmospheric 13C content is going down; and these changes have been dramatic and have happened exactly synchronously with rising CO2 emissions from fossil fuels. Your ‘models’ cannot account for the changes in isotopic ratios; they do not even match the observed rise in CO2 itself. If a model doesn’t reproduce the data, then either the model is wrong, or the data is wrong. Which would you say it is, in this case?
“It is a perfectly legitimate scientific hypothesis, and the fact that it is avoided like the plague illustrates how political climate science has become.”
The hypothesis that some of the temperature and CO2 increases are not due to humans is not even a hypothesis – it’s obvious and well known. The suggestion that ‘natural’ factors are dominant is not a perfectly legitimate scientific hypothesis, because it’s contradicted by the observations. No-one is avoiding anything like the plague.
anna v:
“200 years ago the temperatures started rising because of getting out of the little ice age”
Rising temperatures do not explain rising temperatures.
“and the flora responded accordingly. One can imagine many reasons why a C13 rich strain might flower better with heat.”
Can one? If one imagines what you imagine, one has to imagine that present day temperatures are vastly hotter than they were at any time in the past 1000 years. Otherwise, how can one explain the data?
“The argument about fossil fuel origin of C13 can hold only if there are no other sources of C13 in the game.”
Incorrect. 13C is found in all sources of CO2. It is the proportion that matters. As you see from my link, the 13C proportion of atmospheric CO2 began dropping sharply, about 200 years ago. Fossil fuel CO2 has been emitted in sufficient quantity, and has the right 13C content to explain the observations. What more is there to say?
George E. Smith:
“And in fact we know from measured data that increases in CO2 have not caused any significant increases in global temperature; which incidently we don’t even have any reliable way of measuring.”
You contradicted yourself before, and you’re contradicting yourself now. What can you possibly know from measured data that measures things you believe can’t be reliably measured?

bill
May 15, 2009 4:01 pm

Paul Vaughan (13:08:57) :
You now have me totally confused!
If you are trying to tell me that sampling data at the yearly rate will remove any seasonal data? If so that is obvious even to me!
As to the previous reference to woodfortrees I do not even know what isolate samples (first box) is !! (it appears to be a derivative type of function)
If you told me what you are trying to prove it could help.
All on this blog I think would agree that global temperature is a difficult thing to obtain. Even good data such as central england temperature is subject to weather and so will not represent the temperature of England let alone the world.
CO2 is similar – how do you get a global average from only a few stations with incomplete records.
CO2 from ice cores is a differnt matter – is it an average or spot measurement. (it takes 30 years to be sealed into the ice)
So looking at derivatives of CO2 or temperature on a montly series does not proove much in my mind.
IF it is expected that CO2 affect the temperature on a monthly basis I think it will be very unlikely that this will appear in any comparison plots – there is just too much noise.
Most plots I make take an average of jan temps/co2 over a reference period and then subtract the current jan value to give a deviation from the mean – (this is repeated for each month).
Doing it this way tends to remove the seasonal differences. before doing any sort of 12 month running average.

George E. Smith
May 15, 2009 4:15 pm

And again RW you are wrong.
The increases in CO2 we can and have measured; nobody disputes that; so that takes care of that.
But increases or any changes in global temperature (which I said we can’t reliably measure anyway) have not risen to the point where they can be reliably measured.
We don’t even try to measure the true mean global (surface (or lower troposphere)) temperatures, since that would require an astronomical number of simultaneous measurements to satisfy the rules of sampled data systems; so we settle for a rudimentary observation of “anomalies” which aren’t the same as global temperature measurments.
Yes we have a lot of anecdotal observations of local temperature changes (for short periods of time) such as glaciers retreating (which could be precipitation changes rather than temperature changes); but any accurate sampling of the complete global surface temperature map in compliance with the Nyquist criterion would require orders of magnitude more samples to reflect the minute claimed temperature changes that people infer from anomalies; like a few millidegrees out of a global temprature range of around 150 deg C.
By all accounts the recent CO2 changes are unprecedented in the last 3/4 million years; so we are told; yet the expected temperature changes that models predict can’t poke themselves above the noise level.
So I stand by my statement (within the usual 3:1 fudge factor that accompanies all climate modelling)

May 15, 2009 4:32 pm

Roy W. Spencer (07:53:15) :

Bill:
I suspect that CO2 changes on all time scales…seasonal, interannual, decadal, centennial, etc. The time-smoothing inherent in the ice core record from long ago means you will only see the longest time scale relationships in the ice cores. I also suspect the different time scales correspond to different physical (or biological) mechanisms.
In my blog posting I’m showing a clear interannual relationship, and hypothesizing that it might also extend to decadal and longer time scales. If it does, then some significant part of the observed CO2 increase in the last century is likely natural. Since mankind emits more than is needed to explain the observed increase, this also means the “missing sink” of carbon dioxide is bigger than is currently appreciated.

Here’s a graph [by Bill Illis] showing a 5-month lag: click. There are lots of different cycles where CO2 lags temperature. The 800± year lag is only one of them.
The problem, though, is the fact that CO2 forcing has been endlessly hyped, and its effect has been hugely overestimated. To get some perspective of what the CO2=AGW crowd is trying to sell, Dr. Spencer has a chart that shows atmospheric CO2 in percentage terms, using the correct y-axis: click. This chart is much more honest about CO2 levels that the steeply rising Mauna Loa graph that the alarmists regularly trot out.
The climate isn’t bothered a bit by a desirable increase in this benign trace gas. In fact, the Earth seems to like it: click
And the human contribution of CO2 is very small compared to what nature emits: click.
Finally, I would like to commend Dr. Spencer for admitting he made a mistake. Everyone makes mistakes. It is to the credit of scientific skeptics that when mistakes are made they are corrected. The CO2=AGW side will not stand up and admit they are mistaken about CO2 causing runaway global warming, despite a monumental tower of evidence discrediting that claim. WUWT alone contains reams of information showing that the CO2 scare is wildly overblown.

Joel Shore
May 15, 2009 6:49 pm

Gerald Machnee says:

As I noted in another post, Steve McIntyre has an open question to IPCC or anyone else to provide a detailed engineering quality derivation of the change or sensitivty that a doubling of CO2 will produce. To date there have been no responses other than quoting IPCC which is based on Charney(1979) and lacks the detail.

The IPCC number is based on much more than the original Charney estimate. It is based on a lot of observational data, such as the difference in temperature between the last glacial maximum (LGM) and now, the response of the global temperature to the Mt Pinatubo eruption, and the 20th century temperature record (although the latter does not provide very strong constraints because of the uncertainty in the climate forcing).
In addition to this is the fact that the climate models containing our current understanding of all of the relevant physics seem to converge to the same range of values as these observational data give … and while there are certainly uncertainties with regards to clouds, noone has yet demonstrated a model with a significant negative cloud feedback that can successfully reproduce the basics of the current climatology as well as the current models can.
What McIntyre’s gimmick amounts to is a statement that the climate sensitivity is not easy to calculate in any direct and simple way. Well, yes, that is true. But that hardly means that we should ignore the best evidence from the observational data and from modeling of the climate system in favor of hopes that it is much lower.

May 15, 2009 7:03 pm

Joel Shore:

“What McIntyre’s gimmick amounts to…”

Have to call you on that one, Joel. I doubt if you’re brave enough to go on Climate Audit and call Steve McIntyre’s position a “gimmick” to his face. But I’d be interested in seeing you you try.
REPLY: What Smokey said. Joel, please be my guest, go to CA and make the same claim. And while I’m at it let me add that MciNtyre does things. He investigates, analyses, publishes, makes FOI requests to agancies that aren’t cooperative in sharing data, and suffers fools gladly. Other than whine and complain about others here, what have you done of merit? – Anthony

oms
May 15, 2009 10:06 pm

George E. Smith (16:15:32) :

We don’t even try to measure the true mean global (surface (or lower troposphere)) temperatures, since that would require an astronomical number of simultaneous measurements to satisfy the rules of sampled data systems; so we settle for a rudimentary observation of “anomalies” which aren’t the same as global temperature measurments.

Astronomical is exactly what satellite remote sensing is about. We care so much about the sparse ground stations because they’ve been around for much, much longer (hence are important for climate trends).
Anomalies usually mean the value in relation to a reference mean value. It’s a lot easier to look at small differences in an anomaly map than to visually compare two maps side by side of T0(x,y) and T1(x,y). You can also try to remove some absolute offset this way. Anomalies aren’t inherently different than the absolute values with relation to the sampling grid.

… any accurate sampling of the complete global surface temperature map in compliance with the Nyquist criterion would require orders of magnitude more samples to reflect the minute claimed temperature changes that people infer from anomalies

This does not seem to be a correct application of the Nyquist theorem. Is there some clear evidence of spatial frequency information in the surface temperatures that would bias the temperature integrated over the footprint of a microwave satellite?

Paul Vaughan
May 15, 2009 11:34 pm

Anthony [replying to Smokey (19:03:41)]: “What Smokey said. Joel, please be my guest, go to CA and make the same claim. And while I’m at it let me add that MciNtyre does things. He investigates, analyses, publishes, makes FOI requests to agancies that aren’t cooperative in sharing data, and suffers fools gladly. Other than whine and complain about others here, what have you done of merit?”
Comments:
1) I like reading what Joel has to say – it helps me learn the “standard arguments”.
However,
2) I back Smokey & Anthony regarding the “gimmick” attack — here’s part of why: I’ve done these engineering calculations – they are rigorous — and engineers use common sense like I’ve rarely seen in the 6 other disciplines I’ve visited during the past 2 decades.
– –
Joel Shore (18:49:35) “[…] and while there are certainly uncertainties with regards to clouds, noone has yet demonstrated a model with a significant negative cloud feedback that can successfully reproduce the basics of the current climatology as well as the current models can.”
Interesting.

RW
May 16, 2009 4:59 am

George E. Smith:
“But increases or any changes in global temperature (which I said we can’t reliably measure anyway) have not risen to the point where they can be reliably measured.”
If you don’t think global temperatures can be measured reliably, then logically you have no idea whether they have risen, fallen or stayed the same.
Your presumptuous ‘we’ does not include people actually doing the measurements – such as Roy Spencer, who seems to find no problem with the concept of reliable measurements of global temperatures, and reports an upward trend of +0.13&dec;C/decade since 1979.

Gerald Machnee
May 16, 2009 5:56 am

RE: Joel Shore (18:49:35) :
Gerald Machnee says:
**The IPCC number is based on much more than the original Charney estimate. It is based on a lot of observational data, such as the difference in temperature between the last glacial maximum (LGM) and now, the response of the global temperature to the Mt Pinatubo eruption, and the 20th century temperature record (although the latter does not provide very strong constraints because of the uncertainty in the climate forcing).**
They make assumptions which are not working too well now in the temperature forecasts.
**In addition to this is the fact that the climate models containing our current understanding of all of the relevant physics seem to converge to the same range of values as these observational data give … and while there are certainly uncertainties with regards to clouds, noone has yet demonstrated a model with a significant negative cloud feedback that can successfully reproduce the basics of the current climatology as well as the current models can.**
The current models are not better, but have been massaged to reflect the current results. The cloud and moisture is not adequately modelled. The positive feedbacks are assumed not measured. The errors are starting to show, but the modellers are using 30 years or so to verify and mask the problem.
**What McIntyre’s gimmick amounts to is a statement that the climate sensitivity is not easy to calculate in any direct and simple way. Well, yes, that is true. But that hardly means that we should ignore the best evidence from the observational data and from modeling of the climate system in favor of hopes that it is much lower.**
That is not a gimmick but a legitimate question. The IPCC has their head buried in the sand. McIntyre asked for more information when he was doing reviews and the Chair Susan Solomak responded byt telling him he would be kicked off the panel if he asked such questions. That is how they get their consensus. One on the senior people at IPCC, a British Meteorologist conveniently “lost or discarded” his notes which are supposed to be archived.
If climate sensitivity is nor easy to calculate maybe they should allocate some funds to that instead of using the same tired data to claim the “hockey stick” is still correct.
What best evidence are you referring to??

bill
May 16, 2009 6:40 am

Smokey (16:32:24) :
Here’s a graph [by Bill Illis] showing a 5-month lag: click. There are lots of different cycles where CO2 lags temperature. The 800± year lag is only one of them

I’ve just realised how these lags are derived thanks to Paul Vaughan.
People are looking at monthly derived figures and differentiating them. They are then picking up the small seasonal variation of CO2. Doing similar things with temperature then pointing out a 6 month lag. Temperature variation is seasonally most predominant in NH and so an average global temp will show perturbations relating to NH seasons. See http://cdiac.ornl.gov/trends/co2/sio-keel.html the further south (in general) you go the less the seasonal variation 15 ppm in canada (alert) and 0 (ish) in NZ
South Pole, Antarctica 2ppm
New Zealand Baring Head ~1ppm
Kermadec Islands ~1ppm
Cape Matatula, Samoa ~1ppm
Christmas Island 4ppm
Cape Kumukahi 11ppm
Mauna Loa, Hawaii 9ppm
Baja California Sur, Mexico ~8ppm
La Jolla Pier 13ppm
Barrow, Alaska 20ppm
Alert, NWT, Canada 15ppm
A seasonal variation will affect CO2, with a Lag as is to be expected. BUT this is not the continual rise in CO2 shown by all measuring stations referenced in Pauls post
http://cdiac.ornl.gov/trends/co2/sio-keel.html
It is interesting to note that Antarctica has a greater seasonal variation than NZ. perhaps one needs to check the phase relation ship of CO2 between the various locations. Is Nz’s seasonally produced CO2 nulling the NH change. Is antarctica showing a SH or NH variation. Is SH CO2 seasonal changes 6 months antiphase with NH?
So I would expect there to be a lag between differntial of T and differential of CO2 on seasonal scale. BUT there is no way( ?) of determining the delay +ve or -ve between the slow rise of CO2 and Temperature.
Bill Illis’s graph does not state if the plot of CO2 is differential or standard. I would expect the former

Ron Weinrick
May 16, 2009 6:52 am

Very clear, and passes the common sense test. Thank you, and may cooler heads prevail. I realize it is beyond the scope of the essay, but I’m interested in what may cause changes in cloud cover. Are there cyclical inputs over large time scales (years) which affect the percentage of cloud cover?

bill
May 16, 2009 7:28 am

It’s interesting doing a comparison of a few of the CO2 levels from CDIAC:
http://img190.imageshack.us/img190/4250/co2x4.jpg
Antarctica = SH CO2 seasonal changes
NZ = mix of SH(dominant) and NH
Xmas Is and Mauna Loa NH

Pofarmer
May 16, 2009 7:45 am

Somewhat OT.
But, whatever happened to the group that was showing that CO2 was maybe not so well mixed in the Atmosphere?

bill
May 16, 2009 7:47 am

Should have plotted a few more northerly points:
http://img190.imageshack.us/img190/1068/co2x7.jpg

Roy W. Spencer
May 16, 2009 8:01 am

My graphs were all based on anomalies from the average seasonal cycle.

Roy W. Spencer
May 16, 2009 8:05 am

regarding the realism of models that have tried negative cloud feedback and could not replicate the average behavior of the climate system (which I have never heard before):
1) I doubt that any one has tried that hard, and
2) For global warming we are interested in the feedbacks…not replicating the average behavior of the climate system…which all of the models have difficulty doing anyway.
The CERES team leaders (Wielicki, and now Norman Loeb) have both mentioned that they can not get the modelers to do any more than cursory, basic comparisons with the satellite radiation budget data. I can’t help but wonder why…

bill
May 16, 2009 8:58 am

The weird thing about these NH plots is the dip. It does not look biological it looks as if a switch has been flipped (or full wave rectified – for the electronic people).
at the beginnin of july it is racing down by the beginning of august it is going north at the same ppm rate.
Is there daily data for CO2 – does this show an even spikier spike?

bill
May 16, 2009 9:41 am

Another strange thing is that from 1978 to 2007 the plot of barrow has exactly the same shape – the peak is in may the min is in August
the fall from may to august is 14.55ppm and in 2007 is 17.22
Please note that my comment above (bill (08:58:20) : Your comment is awaiting moderation) is a month out 2000.0 = january / 2000.5 = july
You would have thought that if temperature were causing CO2 to change then the sink would beginearlier each year and the dip would change by less (or more) than the 20% it does now. In fact with warming seas shouldn’t this dip be getting less not more as less CO2 gets absorbed.
Or is the flora. But then wouldnt the dip start earlier than May? And wouldn’t the dip be more rounded and continue into October?
Hhhhhhhhmmmmmm!!!! Strange!

Paul Vaughan
May 16, 2009 2:37 pm

bill, I see you’ve posted a multi-site CO2 timeplot:
http://img190.imageshack.us/img190/1068/co2x7.jpg
Have you looked at an analogous plot for dCO2/dt? It is [a great deal] more telling.
For example, note the Alert winter dip [which I mentioned at Paul Vaughan (13:08:57), responding to Lucy Skywalker (01:06:41)].
You indicated curiosity regarding spikiness at daily resolution. I would push that to sub-daily; 3 hour resolution should be sufficient. Without the finer resolution, it would be difficult (perhaps impossible) to explore the nuances of diurnal & polar-day/night interaction with polar (& latitudinal more generally) continental distribution – as seasons pass. (Think of alternate continental & north-south symmetry/asymmetry configurations to get a handle on what we should be digging for in analyses.)
Thanks to Dr. Spencer & WUWT, I see how ABSOLUTELY RIDICULOUS it is to claim Mauna Loa CO2 represents global CO2 (without applying appropriate heavy qualification – in the form of paragraphs – [not just adjectives]) — stunningly ridiculous. [Before this thread I had been trusting in the “well-mixed” dogma – but, fortunately, another layer of innocence & ignorance has been ripped away in the pursuit of true wisdom.]
bill, your graph illustrates exactly how folks came to the flawed conclusion; they IGNORED the fundamental importance of spatiotemporal variation – perhaps accidentally, but it’s a very serious error, so they need to get past it.
[Enlightened] ecologists & physical geographers have been railing against this very serious error for nearly 3 decades now. [See the literature for an absolutely nauseating number of papers hammering this important & painful lesson (which still does not get through to everyone, despite the intense effort).] The efforts have been motivated by an acute awareness that scientific literature is plagued with erroneous analyses, based on FALSE assumptions (for example about uniformity & symmetry).
We should also be listening to the common sense of engineers, who are forced (by the nature of their work) to remain practical. They are not (so easily) deluded into excessively-linear abstraction by notions that means are of dominating importance; rather, they have a healthy appreciation for – for example – minima, maxima, & ranges. Statisticians can work with the latter summaries no problem, but it is up to those who are consulting with them (who have subject-area expertise) to stress the importance of not making false assumptions – for example about independence & distribution-tails.
Statistical consulting work easily slips deep into communication failure. A skilled consultant knows what questions to ask, but the consultee must also take some responsibility – the ethics of not just truthful, but also full disclosure of relevant details apply.
It remains clear that many bright folks have some very serious catching up to do when it comes to sampling & data analysis in the presence of multi-scale spatiotemporal heterogeneity. In the meantime, these bright folks must NOT be allowed to influence policy with FLAWED analyses and MISLEADING summaries. (Their errors may be innocent, but the consequences?… I could tell you stories…)

maksimovich
May 16, 2009 2:52 pm

Roy W. Spencer (08:05:55) :
regarding the realism of models that have tried negative cloud feedback and could not replicate the average behavior of the climate system (which I have never heard before):
The absence of “theory” is a limiting quality eg Ramanathan 2008
“It is remarkable that general circulation climate models (GCMs) are able to explain
the observed temperature variations during the last century solely through variations in greenhouse gases, volcanoes and solar constant. This implies that the cloud contribution to the planetary albedo due to feedbacks with natural and forced climate changes has not changed during the last 100 years by more than ±0.3%; i.e, the cloud forcing has remained constant within ±1 Wm–2. If indeed, the global cloud properties and their influence on the albedo are this stable (as asserted by GCMs), scientists need to validate this prediction and develop a theory to account for the stability.”
Om the other hand the other UNEP scientific assessment committee (ozone) has a different perspective.eg.
“The Sun’s output is not constant over time, and solar UV-C radiation changes significantly over the 11-year solar cycle. This UV-C does not penetrate as far as the Earth’s surface, but changes in UV-C cause ozone changes of ~3%. Counter- intuitively, the UV-B received at the surface is therefore expected to be a minimum when the solar output is a maximum. However, there may be other climatic impacts of solar variability as well (e.g. changes in cloud cover), …
Cloud Effects
Cloud effects are important. The mean attenuation of UV-B by clouds is typically in the range 15-30%. There have been improvements in the measurement of clouds from automated imagers at the Earth’s surface. Progress has been made relating these cloud images to satellite-derived cloud patterns and to the UV radiation received at the Earth’s surface.37 There is evidence for long-term changes in cloud in some regions, as discussed later….
Biospherical Feedbacks
Other feedbacks can involve the biosphere (Chapters 4 and 5).9, 84, 85 For example, increasing UV can reduce the productivity of oceanic phytoplankton. This can produce two feedbacks. Firstly, it reduces the oceanic sink for carbon in atmosphere (production of carbonates which fall to the sea floor). Secondly, it can influence the production of dimethyl sulphide (DMS), which is an important source of condensation nuclei.85 This, in turn affects cloud-droplet size, cloud reflectivity, and hence planetary albedo.

bill
May 16, 2009 3:36 pm

Paul Vaughan (14:37:04) :
http://img190.imageshack.us/img190/1068/co2×7.jpg
Have you looked at an analogous plot for dCO2/dt? It is [a great deal] more telling.

No
You indicated curiosity regarding spikiness at daily resolution. I would push that to sub-daily; 3 hour resolution should be sufficient. Without the finer resolution, it would be difficult (perhaps impossible) to explore the nuances of diurnal & polar-day/night interaction …
methinks you are getting carried away!
I see how ABSOLUTELY RIDICULOUS it is to claim Mauna Loa CO2 represents global CO2 … stunningly ridiculous.
You are looking at the minutiae, not at the important stuff which is over a longer period. I find it reassuring that the average CO2 for such diverse places is actually very similar antarctica is within a few ppm of actric.
bill, your graph illustrates exactly how folks came to the flawed conclusion; they IGNORED the fundamental importance of spatiotemporal variation – perhaps accidentally, but it’s a very serious error, so they need to get past it.
So what effect are you proposing that the spatiotemporal variation will have on peoples views?
I am proposintg that the rate of change which Roy Spencer has plotted is not relevant neither to man nor beast, as the change is occurring because of flora or sea fauna and is negated a couple of months later. Each cycle is getting to higher levels of CO2 and is showing no reductions.
..these bright folks must NOT be allowed to influence policy with FLAWED analyses and MISLEADING summaries.
VERY true neither should the anti-AGW’s be allowed endanger the future. What is needed is a computer with no agenda, infinite knowledge, capable of weighing up all current climate data. –
(DEEP THOUGHT: The Great Hyperlobic Omnicognate Neutron-Wrangler can talk all four legs off an Arcturan Mega-Donkey but only I can persuade it to go for a walk afterwards.)

Just Want Truth...
May 16, 2009 4:38 pm

This from Piers Corbyn should be taken in to account also :
“Extra plant transpiration-cooling negates all extra co2 heating (i.e. 3.8 W/m2 – 3.8 W/m2 = 0).”
~~Piers Corbyn,
-solar physisist,
-ARCS, FRAS, FRMetS,
-WeatherAction.com, the Long Range Forecasters,
-developer, SWT (Solar Weather Technique),
-Award of AMEME Hopley Shield 2008
PDF page 7
http://junkscience.com/mar08/What_Does-Does_Not_Drive_Climate_Change.pdf

Just Want Truth...
May 16, 2009 4:45 pm

Mr. Spencer,
If you have time, do you have any thoughts on Miklos Zagoni’s finding that there can be no runaway greenhouse effect that is outlined in this video :

Paul Vaughan
May 16, 2009 5:18 pm

bill (06:40:56) “[…] a Lag as is to be expected.”
It is important to note here (for anyone following along) that the lags we are seeing in these data seem [far] more likely to be related to anti-phase than to delayed-response (although we cannot be absolutely certain based just on these data alone).
From what I can see there are 2 (main) cycles of differing amplitudes – & they are in anti-phase. This comes as no surprise, as this is what is seen in many other geophysical time series (on our north-south asymmetric Earth).
– – –
maksimovich (14:52:40) “[…] Ramanathan 2008 […] “This implies that the cloud contribution to the planetary albedo due to feedbacks with natural and forced climate changes has not changed during the last 100 years” […]”
Note the use of the word “implies”; this is another example of the flawed logic that many applied above (in this thread) in interpreting the 90/10 split in Dr. Spencer’s demonstrative-presentation. It stems from a deep misunderstanding of decomposition (& shared variance).

Paul Vaughan
May 16, 2009 6:29 pm

bill (15:36:33) “methinks you are getting carried away!”
How much photosynthesis do you think happens at night (relative to during the day)? ..and in the polar regions during the polar-day versus during the polar-night (i.e. summer vs. winter)?

bill (15:36:33) “What is needed is a computer […]”
Computers are certainly useful tools, but no computer is going to be able to substitute satisfactorily for sound human judgement in the foreseeable future.

bill (15:36:33) “You are looking at the minutiae, not at the important stuff […] I am proposintg that the rate of change which Roy Spencer has plotted is not relevant neither to man nor beast […] VERY true neither should the anti-AGW’s be allowed endanger the future.”
Do you think a boreal forest carbon-modeler would agree with your suggestion that we ignore seasonal variations?
I think you might seriously misunderstand my motivation (& the motivation of others).
My interest is in understanding nature, including nature’s nuances. As indicated above, prior to this thread I did not find CO2 to be a very interesting variable to study because all it shows (at annual resolution – & upon superficial inspection) in the modern 1958+ records is dull, monotone increase (which leaves little over which to puzzle).
I’m neither “anti-AGW” nor “pro-AGW”. I’m a student of nature [which includes humans], an advocate of careful analysis, and a proponent of balance.

bill (15:36:33) “So what effect are you proposing that the spatiotemporal variation will have on peoples views?”
The take-home message for the general public here might be something like this:
Thinking of CO2 equilibria-chain balances only in terms of a “global annual average” is far too simple.

Final comments:
No one (sensible) is trying to say CO2 is not increasing; (sensible) people are saying, “Let’s understand nature, including water.” I see no sensible reason for introducing biases about what spatiotemporal scales are worth understanding. We can draw no definitive conclusions until we understand the whole picture (if that’s even possible).

bill
May 16, 2009 7:12 pm

Paul Vaughan (18:29:02) :
Thanks for the response.
I admire your interest in these things.
I think that CO2 is measured at a 2 weekly rate so you will find it difficult to find shorter period with long term records.
I would still like to know how sharp the NH dip is and what feature of the ecosystem can respond in such a rapid way!
bill

Paul Vaughan
May 16, 2009 8:46 pm

bill (19:12:15)
“I think that CO2 is measured at a 2 weekly rate so you will find it difficult to find shorter period with long term records.
I would still like to know how sharp the NH dip is and what feature of the ecosystem can respond in such a rapid way!”

All you have to do to see the dip is difference the series (& then plot). [It is as simple as d(i+1) = CO2(i+1) – CO2(i). There are advantages to this approach (over making assumptions about annual structure, which will mess up attempts to go after subtle nuances).]
I too was hoping someone would share the “standard” 2-sentence overview of the Alert winter dip – to tide me over until I have time to venture into another massive branch of the research jungle.
I rarely (fully) trust any conclusions I see in any publication without performing analyses myself. Researchers have all sorts of formal training in (abstract, theoretical) statistics (including heavily-mathematical statistics, complete with loads of proofs & derivations), but the education system – which has other priorities – lacks focus on fundamental concepts, data analysis, and the application of sound judgement in data analysis.
My guess is that there will be plenty of datasets & publications, including a good number focused on short timescales. Publicly-available text-format data-websites are (arguably) essential in a knowledge society. Although I’m confident shorter timescale datasets exist, I suspect you’re right that they will not be long-term and I anticipate tediously-inefficient bureaucratic access-hoops (in most/many cases).

bill (19:12:15) “I admire your interest in these things.”
Likewise – & same regarding co-participants in the evolving confluence of science & democracy that occurs in these valuable forums.

Paul Vaughan
May 17, 2009 8:51 pm

Misleading the Public? – Or Just an “Accident”?
If one googles “carbon dioxide wiki”, one is led to:
Wikipedia – Carbon dioxide
http://en.wikipedia.org/wiki/Carbon_dioxide

Focusing further on CO2 “in the Earth’s atmosphere” within that article leads to:
Wikipedia – Carbon dioxide in the Earth’s atmosphere
http://en.wikipedia.org/wiki/Carbon_dioxide_in_the_Earth%27s_atmosphere

The caption of the first graph one sees there links to:
Wikipedia – Keeling Curve
http://en.wikipedia.org/wiki/Keeling_curve

If one digs around for a link to “data” on that page, one finds a link to CO2 data labeled:
“Globally averaged marine surface monthly mean data.”
but the link actually leads to the Mauna Loa Observatory (elevation more than 3km) data:
ftp://ftp.cmdl.noaa.gov/ccg/co2/trends/co2_mm_mlo.txt

…and note that nowhere on that webpage can one find “mauna loa” — that clue is found elsewhere in the directory – by adjusting the url …which is also how one can find the “globally-averaged” data.
Maybe it was just a linking accident? – even if so, the error is well worth pointing out, particularly considering the insights (into spatial variation) that arose in this thread.
– – –
Worthy of note for albedo/cloud enthusiasts:
“[…] a decline in the coccolithophores may have secondary effects on climate change, by decreasing the earth’s albedo via their effects on oceanic cloud cover.”
Wikipedia – Ocean Acidification
http://en.wikipedia.org/wiki/Ocean_acidification
[a rather “alarmist” article – well worth assessing for political bias]

Paul Vaughan
May 17, 2009 11:00 pm

Wikipedia – Carbon sink
http://en.wikipedia.org/wiki/Carbon_dioxide_sinks
“Because the effect of periodic small scale phytoplankton blooms on ocean ecosystems is unclear, more studies would be helpful. Phytoplankton have a complex effect on cloud formation via the release of substances such as dimethyl sulfide (DMS) that are converted to sulfate aerosols in the atmosphere, providing cloud condensation nuclei, or CCN. But the effect of small scale plankton blooms on overall DMS production is unknown.”

Paul Vaughan
May 18, 2009 2:24 am
maksimovich
May 18, 2009 3:43 pm

Paul Vaughan (20:51:52) :
Worthy of note for albedo/cloud enthusiasts:
“[…] a decline in the coccolithophores may have secondary effects on climate change, by decreasing the earth’s albedo via their effects on oceanic cloud cover.”
Wikipedia – Ocean Acidificationhttp://en.wikipedia.org/wiki/Ocean_acidification
[a rather “alarmist” article – well worth assessing for political bias]
GLOBAL BIOGEOCHEMICAL CYCLES, VOL. 15, No. 2, PAGES 507-516, JUNE 2001
Decreasing marine biogenic calcification: A negative feedback on rising atmospheric pCo2
Ingrid Zondervan, Richard E. Zeebe1, Björn Rost, and Ulf Riebesell
Alfred Wegener Institute for Polar and Marine Research Bremerhaven, Germany
Abstract. In laboratory experiments with the coccolithophore species Emiliania huxleyi and Gephyrocapsa oceanica, the ratio of particulate inorganic carbon (PIC) to particulate organic carbon (POC) production decreased with increasing CO2 concentration ([CO2]). This was due to both reduced PIC and enhanced POC production at elevated [CO2]. Carbon dioxide concentrations covered a range from a preindustrial level to a value predicted for 2100 according to a “business as usual” anthropogenic CO2 emission scenario. The laboratory results were used to employ a model in which the immediate effect of a decrease in global marine calcification relative to POC production on the potential capacity for oceanic CO2 uptake was simulated. Assuming that overall marine biogenic calcification shows a similar response as obtained for E. huxleyi or G. oceanica in the present study, the model reveals a negative feedback on increasing atmospheric CO2 concentrations owing to a decrease in the PIC/POC ratio
Paul Vaughan (23:00:02)
Because the effect of periodic small scale phytoplankton blooms on ocean ecosystems is unclear, more studies would be helpful. Phytoplankton have a complex effect on cloud formation via the release of substances such as dimethyl sulfide (DMS) that are converted to sulfate aerosols in the atmosphere, providing cloud condensation nuclei, or CCN. But the effect of small scale plankton blooms on overall DMS production is unknown.”
Variability of atmospheric dimethylsulphide over the southern Indian
Ocean due to changes in ultraviolet radiation
D. R. Kniveton,1 M. C. Todd,2 J. Sciare,3 and N. Mihalopoulos4
Received 13 January 2003; revised 16 May 2003; accepted 11 August 2003; published 10 October 2003.
[1] Dimethylsulphide (DMS) is a climatically important component of global
biogeochemical cycles, through its role in the sulphur cycle. Changes in ultraviolet
radiation (UV) exhibit both positive and negative forcings on the dynamics of production and turnover of DMS and its precursor dimethylsulphoniopropionate (DMSP). In this study we investigate the net forcing of UV on atmospheric DMS. The work is based on a 10-year record of observed DMS at Amsterdam Island in the southern Indian Ocean, and satellite-based retrievals of surface UVand photosynthetically active radiation (PAR).The results show an inverse relationship between UV radiation and atmospheric DMS associated with extreme changes (defined as the greatest 5%n daily UV, independent of changes in wind speed, sea surface temperature, and PAR.
Testing the relationship between the solar radiation dose and surface DMS
concentrations using high resolution in situ data
C. J. Miles, T. G. Bell, and T. M. Lenton 2009
Abstract
We tested the recently proposed, strong positive relationship between dimethylsulphide (DMS) concentrations and the solar radiation dose (SRD) received into the surface ocean. We utilised in situ daily data sampled concurrently with DMS concentrations 5 from the Atlantic Meridional Transect (AMT) programme for the component variables of the SRD; mixed layer depth (MLD), surface insolation (I0) and a light attenuation coefficient (k), to calculate SRDin situ. We find a significant correlation (_=0.53) but the slope of the relationship is approximately half that previously proposed. The correlation is improved (_=0.76) by replacing the in situ data with an estimated I0 (which assumes 10 a constant 50% removal of the top of atmosphere value; 0.5×TOA), a MLD climatology and a fixed value for k following a previously described methodology. Equally significant, but non-linear relationships are also found between DMS and both in situ MLD (_=0.73) and the estimated I0 (_=0.76) alone. The DMS data shows an interesting relationship to an approximated UV attenuation depth profile. Using a cloud adjusted, 15 satellite climatology of surface UVA irradiance to calculate a UV radiation dose (UVRD)provides an equivalent correlation (_=0.73) to DMS. With this data, MLD appears the dominant control upon DMS concentrations and remains a useful shorthand to prediction without fully resolving the biological processes involved. However, the implied
relationship between incident solar/ultraviolet radiation dose and sea surface DMS con20 centrations (modulated by MLD) is critical for closing a climate feedback loop.
Also recommend Paul Crutzen.
Crutzen PJ 2002. Analysis of the Gaia hypothesis as
a model for climate/biosphere interactions. GAIA
2/2002, 96–103.
Also an interesting article here
(PhysOrg.com) — Groundbreaking Victoria University research shows that ocean acidification may have no negative effect on tropical corals and local sea anemones – in fact it may improve photosynthesis.
http://www.physorg.com/news161877580.html

May 19, 2009 10:47 am

Just back from a nice trip to Prague, Linz (Austria) and Southern Germany, saw this discussion now. Took me halve a day to wade through the comments… A lot of interest anyway!
I have made a lot of comments on the origin of the CO2 rise in the past, including Dr. Spencer’s different attempts to look at nature as cause…
Again a new round, where Dr. Spencer used this simple model:
delta[CO2]/delta[t] = a*SST + b*Anthro
There are fundamental problems with this model:
The first one is that there is no time limit in the equation for a change in sea surface temperature. That is the same problem as with the previous discussion with Frank Lansner at this blog: if the sea surface temperature goes up, the model assumes that the increase of CO2 per year remains the same over time. That is certainly not the case, as a new equilibrium between ocean temperature and vegetation at one side and CO2 levels at the other side will be reached within a limited amount of time for the fast processes (ocean solubility and vegetation growth) and after longer times for the slow processes (ice sheet/vegetation area, -deep- ocean flows). The real formula should have a term related to the change in temperature, not the relative temperature, as the overall change in CO2 is related to the change in temperature…
The second fundamental problem is that the formula shows the short term relationship between dCO2 and dT, that is the variability around the trend, but that doesn’t need to have any relationship with the trend itself, if the trend is caused by something else than temperature. Temperature then is only causing the “noise” around the trend. And eventually a small part of the trend, if there is a temperature difference between the start and the end of the period of interest. On the other side, the emissions cause only a small part of the variability around the trend, as most of the variability is temperature related.
I have used an alternative “simple model”:
dCO2 = 3*dT + 0.55*emissions
where CO2 and emissions are in ppmv (1 ppmv CO2 = 2.1 GtC emissions) and 3*dT is the factor 3 ppmv/°C which fits for short term changes, up to 8 ppmv/°C for very long term temperature influence on CO2 levels (glacials – interglacials, MWP-LIA temperature change) and dT is the temperature difference between begin and end of the period…
If we plot that formula for the short term variability (dCO2/dt), then we see the following:
http://www.ferdinand-engelbeen.be/klimaat/klim_img/egbn_trend.jpg
which is a reasonable fit to the variability around the trend.
Pieter Tans of NOAA, responsible for the CO2 data of several stations, including Mauna Loa, made a better fit by including precipitation, which influences plant growth (see the second halve of the pdf):
http://esrl.noaa.gov/gmd/co2conference/pdfs/tans.pdf
Some different view if we look at the trends themselves. Have a look at the trends of temperature and accumulated emissions vs. CO2 levels:
http://www.ferdinand-engelbeen.be/klimaat/klim_img/temp_co2_acc_1900_2004.jpg
The CO2 levels until 1960 are from different ice cores (Law Dome, best resolution 8 years), after 1959 from Mauna Loa. As can be seen, even in cooling periods (1945-1975) the CO2 levels increase at about 55% ratio with the accumulated emissions…
Even more informative, the correlation trends 1900-2004 between accumulated emissions and atmosphere:
http://www.ferdinand-engelbeen.be/klimaat/klim_img/acc_co2_1900_2004.jpg
Compare that to the temperature – CO2 correlation trend:
http://www.ferdinand-engelbeen.be/klimaat/klim_img/temp_co2_1900_2004.jpg
where a large change in temperature has little influence on CO2 increase…
Main conclusion of this all: temperature has a fast (and slow) influence on CO2 levels, but that is limited to 3ppmv/°C for short term changes, up to 8 ppmv/°C for very long term changes. Thus while temperature is mainly responsible for the variability around the trend, the emissions are responsible for the bulk of the trend itself…

May 19, 2009 11:20 am

Paul Vaughan 16-05-2009 (20:46:46) :
My guess is that there will be plenty of datasets & publications, including a good number focused on short timescales. Publicly-available text-format data-websites are (arguably) essential in a knowledge society. Although I’m confident shorter timescale datasets exist, I suspect you’re right that they will not be long-term and I anticipate tediously-inefficient bureaucratic access-hoops (in most/many cases).
The data of one-hour raw averages of 40 minutes of 10-second measurements (without any filtering) are available at:
ftp://ftp.cmdl.noaa.gov/ccg/co2/in-situ/
for four base stations, including Barrow, from the start year (1973 for Barrow) up to 2008…
A nice explanation of the procedures followed at Mauna Loa and other baseline stations is at:
http://www.esrl.noaa.gov/gmd/ccgg/about/co2_measurements.html
About the dip of Barrow and other far north stations: most of the time the ocean near Barrow is frozen. When the temperature increases, two main things happen: tundra grows as crazy and the ocean ice sheet disappears. The first can be seen in the increase of d13C level and both as a dip in the CO2 levels (the stil cold ocean absorbs a lot of CO2). The increase in winter is more difficult to explain, but CO2 may be transported in from other places.
Most baseline stations are in the Pacific at sea level and don’t show such a huge seasonal variability. But if you look at the variability of Schauinsland (Black Forest, southern Germany), at 1200 m when measured with sufficient wind speed and above the inversion layer, the seasonal variation is even larger than at Barrow:
http://cdiac.ornl.gov/trends/co2/graphics/schauinsland.gif
The Ferrell air moving cells may transport the CO2 levels polewards…
Hope this helped…

bill
May 20, 2009 10:28 am

Paul Vaughan (02:24:30) :
Thanks for the info Looking at the “daily” data – not daily! – the minimum of the CO2 dip occurs within about +- 1 week f4rom the start of the northern records to present.
Ferdinand Engelbeen (10:47:54) :
The daily data dip in CO2 is very sharp. It goes from negative to positive slope within a couple of weeks. The slope to max co2 is very similar to to min co2 in this dip.
Is this really possible from a biological process and sea ice? It’s acting as if a CO2 sink / source switch has been thrown. No AGW agenda on this point! I just would like to know how the mass of the atmosphere can react this quickly.

May 20, 2009 3:52 pm

bill 20-05-2009, 10:28:24 :
Both the start and end of the growing season and the ice breakup and reformation are quite rapid processes in the high north… Usually transformed from complely frozen sea to open sea (at least near the coast) and from snow covered frozen land to growing leafs and flowers within a few weeks…
The atmospheric mixing is relative fast: days to weeks at the same altitude, with some Noth-South gradient (and a steep gradient near the equator, caused by the ITCZ), weeks to months for different altitudes. But as plant growth and ocean absorption are rather continuous processes when the temperature is high enough and ice is melted, there will be a continuous difference between the places with (relative) huge vegetation growth and high absorption and the rest of the atmosphere.
The other way out is more difficult to understand, as once all is frozen, there is no real source of CO2, or there should still be a lot of bacterial life decomposing the organics on/in the (permafrost!) soils.
Alternative is that the fast changes come from elsewhere and are brought in by air circulation, where the change from polar highs in winter to more southernly air masses could be the cause…
That the dip is mainly from vegetation growth can be seen in the d13C changes: http://cdiac.ornl.gov/trends/co2/allison-csiro/graphics/alc_c13co2.jpg here for Alert (NW Territories, Canada)
Looking at the different graphs of CO2 and CO2C13 (=d13C) at the NOAA website, it seems that the behaviour of CO2 and d13C at Barrow and Hohenpeisenberg (Germany, 1,000 m) is practically the same, see: http://www.esrl.noaa.gov/gmd/ccgg/iadv/
It looks like that instream at Barrow of already mixed air from the south is the main cause of the variation…

bill
May 20, 2009 4:46 pm

Ferdinand Engelbeen (15:52:44) :
If the change over from -ve co2 slope to +ve slope were to take many weeks I would agree that vegetation/Ice could be the cause but using your ref for hourly data I plotted this for Barrow:
http://img132.imageshack.us/img132/7442/barrowhourlyco2.jpg
http://img190.imageshack.us/img190/1068/co2x7.jpg
Despite the change in temperature between 1975 and 2008 the minimum has barely changed. And the slope switch occurs in a metter of a couple of weeks
I still find the speed of response amazing for such a massive system!
Vegetation will affect the CO2 in a continual way for many weeks as spring approaches from the south. By the time Barrow unfreezes it will be growing vigorously a few hundred km further south.

Paul Vaughan
May 20, 2009 10:36 pm

Re: Ferdinand Engelbeen (10:47:54), (11:20:28), & (15:52:44)
Thank you for providing the very helpful comments & links.

Clarification:
It is not the summer dip at Alert (northern Canada), but rather the winter dip (in dCO2/dt) that is of particular interest.
I have been digging around to try to figure it out. Anything I could offer at this stage would be premature — I will refrain from speculative comment at this time.
Insights from anyone presently more-knowledgeable are certainly welcome.
– – –
Regarding “daily” CO2 time series, there is a noteworthy mystery:
The data labeled “daily” at
http://scrippsco2.ucsd.edu/data/data.html
…are NOT daily.
Fortunately, Ferdinand has pointed out truly daily CO2 time series:
ftp://ftp.cmdl.noaa.gov/ccg/co2/in-situ/
I briefly became concerned that a lot of the data had been estimated (interpolated – possibly using untenable assumptions about annual structure) after noting the long & variable-length gaps in the Scripps “daily” [but not daily] data.
I am very thankful that Ferdinand has commented. However, I will be uneasy about all CO2 data until someone sheds some light on what is up with the Scripps website’s misleading “daily” links.

bill (16:46:57) “Despite the change in temperature between 1975 and 2008 the minimum has barely changed. And the slope switch occurs in a metter of a couple of weeks
I still find the speed of response amazing for such a massive system!”

I imagine you are speaking of the seasonal timing of the minimum. If so, keep in mind that at the latitude of Alert, there is basically only polar-night (winter) & polar-day (summer). Think about the very low angle of the sun as it circles the sky [staying above the horizon (aside from mountain-shadows)] in the “evening” of the polar-day (i.e. approaching Sept. 21).

Just Want Truth...
May 23, 2009 2:37 pm

Just Want Truth… (16:45:32) :
CORRECTION
Ferenc Miskolczi not Miklós Zágoni.